Chuyên đề bất đẳng thức THCS Đề tài tốt nghiệp

101 375 0
Chuyên đề bất đẳng thức THCS  Đề tài tốt nghiệp

Đang tải... (xem toàn văn)

Tài liệu hạn chế xem trước, để xem đầy đủ mời bạn chọn Tải xuống

Thông tin tài liệu

Bất đẳng thức là một chuyên đề khó. Tài liệu này hệ thống rất nhiều phương pháp chứng minh bất đẳng thức rất dễ hiểu, kèm các ví dụ và bài tập thực hành. Đây là đề tài tốt nghiệp CĐSP của tác giả. Quý thầy cô và các bạn có thể tải về và nghiên cứu.

§Ò tµi: Chøng minh bÊt ®¼ng thøc vµ c¸c øng dông a. ®Æt vÊn ®Ò 1. lý do chän ®Ò tµi To¸n häc lµ mét khoa häc tù nhiªn, to¸n häc ra ®êi tõ rÊt sím nh»m ®¸p øng nhu cÇu ®o ®¹c ruéng ®Êt vµ x©y dùng nhµ cöa. Cµng ngµy x· héi loµi ngêi cµng tiÕn dÇn lªn ë møc ®é cao h¬n vµ ®Õn nay ®ang ®ang ë tr×nh ®é cao nhÊt tõ mµ loµi ngêi cha tõng cã. Do ®ã to¸n häc cñng kh«ng n»m ngoµi quy luËt ph¸t triÓn tõ s¬ khai ®Õn hiÖn ®¹i. To¸n häc nghiªn cøu rÊt nhiÒu, rÊt ®a d¹ng vµ phong phó. Trong ®ã c¸c bµi to¸n vÒ bÊt ®¼ng thøc lµ nh÷ng bµi to¸n khã , ®Ó gi¶i ®îc c¸c bµi to¸n vÒ bÊt ®¼ng thøc, bªn c¹nh viÖc n¾m v÷ng kh¸i niÖm vµ c¸c tÝnh chÊt c¬ b¶n cña bÊt ®¼ng, cßn ph¶i n¾m ®îc c¸c ph¬ng ph¸p chøng minh bÊt ®¼ng thøc. Cã nhiÒu ph¬ng ph¸p ®Ó chøng minh bÊt ®¼ng vµ ta ph¶i c¨n cø vµo ®Æc thï cña mçi bµi to¸n mµ sö dông ph¬ng ph¸p cho phï hîp. Mçi bµi to¸n chøng minh bÊt ®¼ng thøc cã thÓ ¸p dông ®îc nhiÒu ph¬ng ph¸p gi¶i kh¸c nhau , còng cã bµi ph¶i phèi hîp nhiÒu ph¬ng ph¸p mét c¸ch hîp lÝ míi gi¶i ®îc. Bµi to¸n chøng minh bÊt ®¼ng thøc ®îc vËn dông nhiÒu vµo c¸c d¹ng bµi to¸n gi¶i vµ biÖn luËn ph¬ng tr×nh, bÊt ph¬ng tr×nh, hÖ ph¬ng tr×nh ®Æc biÖt , t×m gi¸ trÞ lín nhÊt , nhá nhÊt cña biÓu thøc ...vµ ®Ò thi häc sinh giái huyÖn, thµnh phè, tuyÓn sinh vµo líp 10 thêng cã bµi to¸n bÊt ®¼ng thøc, trong khi ®ã s¸ch gi¸o khoa phæ th«ng l¹i tr×nh bµy...V× vËy häc sinh cÇn thiÕt ph¶i n¾m ®îc nh÷ng kiÕn thøc c¬ b¶n vÒ bÊt ®¼ng thøc . Trong thùc tÕ ë trêng THCS , häc sinh gÆp nhiÒu khã kh¨n khi gi¶i c¸c bµi to¸n liªn quan vÒ bÊt ®¼ng thøc , v× c¸c bµi to¸n chøng minh bÊt ®¼ng thøc thêng kh«ng cã c¸ch gi¶i mÉu, kh«ng theo mét ph¬ng ph¸p nhÊt ®Þnh nªn häc sinh kh«ng x¸c ®Þnh ®îc híng gi¶i bµi to¸n . MÆt kh¸c v× nhËn thøc cña häc sinh THCS cßn cã nhiÒu h¹n chÕ vµ kh¶ n¨ng t duy cha tèt do ®ã häc sinh cßn lóng tóng nhiÒu vµ kh«ng biÕt vËn dông kiÕn thøc vµo gi¶i c¸c d¹ng bµi tËp kh¸c . Trong néi dung cña ®Ò tµi nµy xin ®îc tËp trung giíi thiÖu c¸c tÝnh chÊt c¬ b¶n, mét sè ph¬ng ph¸p hay ®îc sö dông khi chøng minh bÊt ®¼ng 1 Sinh viªn: NguyÔn Xu©n L ¬ng Líp C§SP To¸n – Tin K48 §Ò tµi: Chøng minh bÊt ®¼ng thøc vµ c¸c øng dông thøc nh : dïng ®Þnh nghÜa , biÕn ®æi t¬ng ®¬ng , dïng c¸c bÊt ®¼ng thøc ®· biÕt , ph¬ng ph¸p ph¶n chøng, tam tøc bËc hai …., mét sè bµi tËp vËn dông vµ c¸c øng dông cña bÊt ®¼ng thøc nh»m gióp häc sinh bít lóng tóng khi gÆp c¸c bµi to¸n vÒ chøng minh hay vËn dông bÊt ®¼ng thøc , gióp häc sinh cã thÓ tù ®Þnh híng ®îc ph¬ng ph¸p chøng minh, gi¶i c¸c bµi to¸n liªn quan vµ høng thó h¬n khi häc vÒ bÊt ®¼ng thøc nãi riªng vµ bé m«n To¸n nãi chung . Qua ®Ò tµi (mét sè ph¬ng ph¸p chøng minh bÊt ®¼ng thøc vµ øng dông cña bÊt ®¼ng thøc ) t«i muèn gióp häc häc sinh cã thªm mét sè ph¬ng ph¸p chøng minh bÊt ®¼ng thøc ®ã lµ lý do t«i chän ®Ì tµi nµy, khi nghiªn cøu kh«ng tr¸nh khái nh÷ng sai sot m¸c ph¶i rÊt mong ®îc sù gãp ý cña c¸c thµy c« gi¸o, c¸c b¹n ®Ó ®Ò tµi ®îc hoµn thiÖn h¬n, t«i xin ch©n thµnh c¶m ¬n! 2. NhiÖm vô nghiªn cøu. - kü n¨ng gi¶i c¸c bµi to¸n chøng mih bÊt ®¼ng thøc - kü n¨ng vËn dông bÊt ®¼ng thøc ®Ó gi¶i c¸c bµi to¸n: T×m gi¸ trÞ lín nhÊt-nhá nhÊt, gi¶i hÖ ph¬ng tr×nh, ph¬ng tr×nh nghiÖm nguyªn, ph¬ng tr×nh v« tØ. 3. ®èi tîng nghiªn cøu. - Häc sinh trung häc c¬ së - C¸c ph¬ng ph¸p chøng minh bÊt ®¼ng thøc vµ øng dông cña nã. 4- Ph¬ng ph¸p nghiªn cøu : Qua qu¸ tr×nh häc tËp tõ tríc ®Õn nay, tham kh¶o tµi liÖu, thu thËp tµi liÖu, ®óc rót, tæng kÕt kinh nghiÖm, kiÓm tra kÕt qu¶ kiÓm tra chÊt lîng häc sinh, nghiªn cøu hå s¬ gi¶ng d¹y, ®iÒu tra trùc tiÕp th«ng qua c¸c giê häc, thÓ hiÖn trªn nhiÒu ®èi tîng häc sinh kh¸c nhau : Häc sinh giái, kh¸ vµ häc sinh trung b×nh vÒ m«n To¸n 5. ph¹m vi nghiªn cøu. Giíi h¹n ë phÇn chøng minh bÊt ®¼ng thøc vµ c¸c øng dông cña bÊt ®¼ng thøc ë ch¬ng tr×nh to¸n trung häc c¬ së b. GI¶I QUYÕT VÊN §Ò. Sinh viªn: NguyÔn Xu©n L ¬ng Líp C§SP To¸n – Tin K48 2 §Ò tµi: Chøng minh bÊt ®¼ng thøc vµ c¸c øng dông PhÇn I. C¬ së lý luËn. §Ó gi¶i ®îc bµi to¸n ®ßi hái mæi ngêi ph¶i ®äc kü bµi to¸n xem bµi to¸n yªu cÇu c¸i g×, ph¶i sö dông nh÷ng ph¬ng ph¸p nµo ®Ó gi¶i, ®· gÆp bµi to¸n nµo ®· gi¶i cã d¹ng t¬ng tù nh bµi to¸n ®ã hay kh«ng ®Ó tõ ®ã cã thÓ t×m ra c¸ch gi¶i. §èi víi häc sinh trung häc c¬ së viÖc vËn dông khiÕn thøc lý thuyÕt, nhËn d¹ng bµi to¸n ®Ó t×m ra c¸ch gi¶i cha ®îc rÌn luyÖn nhiÒu ®«i lóc tr×nh bµy vÊn ®Ò nµy cßn s¬ sµi. Khi nghiªn cøu vÒ bÊt ®¼ng thøc ta thÊy r»ng nã thËt sù cã t¸c dông rÌn luyÖn vµ ph¸t huy kh¶ n¨ng t duy ®Ó gi¶i to¸n kh«ng chØ riªng g× bÊt ®¼ng thøc mµ cßn gi¶i c¸c d¹ng to¸n kh¸c bëi muèn gi¶i ®îc nã ®ßi hái ph¶i thËt sù cã mét kiÕn thøc to¸n häc rÊt lín. Ph¬ng ph¸p ®Ó gi¶i c¸c bµi to¸n bÊt ®¼ng thøc kh«ng ë ®©u xa x«i ngoµi ch¬ng tr×nh cña c¸c em häc sinh trung häc c¬ së. Nhng viÖc c¸c em vËn dông nã nh thÕ nµo ®ã lµ vÊn ®Ò cèt lái. Muèn lµm ®îc ®iÒu ®ã ®ßi hái häc sinh ph¶i thËt sù n¾m v÷ng kiÕn thøc, ph¶i cã lËp luËn l«gic, xÐt ®Çy ®ñ c¸c mÆt kh¸c nhau cña bµi to¸n, nhËn d¹ng ®îc bµi to¸n. §Æc biÖt c¸c häc sinh kh¸ giái ph¶i linh ho¹t, s¸ng t¹o kh«ng chØ gi¶i ®îc bµi to¸n mµ cßn ph¶i kh¸i qu¸t ®îc d¹ng cña nã ®Ó ®ua ra ph¬ng ph¸p chung cho c¸c bµi to¸n kh¸c tu¬ng tù. Khi gi¶ng d¹y cho häc sinh c¸c gi¸o viªn ph¶i rÌn luyÖn cho c¸c em n¾m ch¾c phÇn lý thuyÕt, ®a ra c¸c vÝ dô minh ho¹ cô thÓ, c¸c bµi tËp vËn dông, nªn chó ý t¹o cho c¸c em c¸ch nh×n nhËn mét bµi to¸n ®Ó gi¶i kh«ng nªn gi¶i t¾t, lµm t¾t t¹o cho häc sinh khã hiÓu thËm chÝ kh«ng h×nh thµnh ®îc l«gic cña to¸n häc. Thêi lîng ch¬ng tr×nh dµnh cho bÊt ®¼ng thøc ë phæ th«ng c¬ së lµ h¹n chÕ. Do ®ã viÖc häc tËp vµ vËn dông thµnh thao cho c¸c em sÎ khã kh¨n ®ãi víi c¸c em cã häc lùc trung b×nh, kh¸. PHÇN 2. néi dung cña ®Ò tµi. i> c¸c kiÕn thøc cÇn lu ý. 1) §Þnh nghÜa bÊt ®¼ng thøc + a nhá h¬n b , kÝ hiÖu a < b + a lín h¬n b , kÝ hiÖu a > b , + a nhá h¬n hoÆc b»ng b , kÝ hiÖu a ≤ b, + a lín h¬n hoÆc b»ng b , kÝ hiÖu a ≥ b , 2) m«t sè tÝnh chÊt cña bÊt ®¼ng thøc: a) NÕu a > b vµ b > c th× a > c (tÝnh chÊt b¾c cÇu) b) NÕu a > b vµ c bÊt k× th× a + c > b + c Tøc lµ: Khi céng vµo 2 vÕ cña bÊt ®¼ng thøc víi cïng mét sè bÊt k× th× bÊt ®¼ng thøc kh«ng ®æi chiÒu. c) NÕu a > b + c th× a − b > c Sinh viªn: NguyÔn Xu©n L ¬ng Líp C§SP To¸n – Tin K48 3 §Ò tµi: Chøng minh bÊt ®¼ng thøc vµ c¸c øng dông Tøc lµ: Ta cã thÓ chuyÓn mét sè h¹ng cña bÊt ®¼ng thøc tõ vÕ nµy sang vÕ kia vµ ph¶i ®æi dÊu sè h¹ng ®ã. d) NÕu a > b vµ c > d th× a + c > b + d Tøc lµ: NÕu céng vÕ víi vÕ cña 2 bÊt ®¼ng thøc cïng chiÒu ta ®îc mét bÊt ®¼ng thøc cïng chiÒu. Chó ý: Kh«ng ®îc céng vÕ víi vÕ cña 2 bÊt ®¼ng thøc ngîc chiÒu e) NÕu a > b vµ c < d th× a − c > b − d Tøc lµ: NÕu trõ vÕ víi vÕ cña 2 bÊt ®¼ng thøc ngîc chiÒu ta ®îc mét bÊt ®¼ng thøc cïng chiÒu víi bÊt ®¼ng thøc bÞ trõ. Chó ý: Kh«ng ®îc trõ vÕ víi vÕ cña 2 bÊt ®¼ng thøc cïng chiÒu. f) NÕu a > b vµ c > 0 th× ac > bc NÕu a > b vµ c < 0 th× ac < bc Tøc lµ: Nh©n 2 vÕ cña mét bÊt ®¼ng thøc víi cung mét sè d¬ng thf bÊt ®¼ng thøc kh«ng ®æi chiÒu Nh©n 2 vÕ cña mét bÊt ®¼ng thøc víi cïng mét sè ©m th× bÊt ®¼ng thøc ®æi chiÒu. g) NÕu a > b > 0 vµ c > d > 0 th× ac > bd Tøc lµ: NÕu ta nh©n vÕ víi vÕ hai bÊt ®¼ng thøc cïng chiÒu cã c¸c vÕ ®Òu d¬ng th× ta ®îc mét bÊt ®¼ng thøc cung chiÒu. Chó ý: Kh«ng ®îc nh©n vÕ víi vÕ cña hai bÊt ®¼ng thøc ngîc chiÒu. h) NÕu a > b > 0 th× 1 1 > >0 b a Tøc lµ: NÕu nh©n 2 vÕ cña bÊt ®¼ng thøc ®Òu d¬ng th× phÐp lÊy nghÞch ®¶o dæi chiÒu cña bÊt ®¼ng thøc. k) NÕu a > b > 0 vµ n nguyªn dong th× a n > b n NÕu a > b vµ n nguyªn dong th× a n + > b n +1 3. Mét sè bÊt ®¼ng thøc th«ng dông 2 + A ≥ 0( A = 0 ⇔ A = 0); A = A 2 + A ≤ B ⇔ − B ≤ A ≤ B (B ≥ 0) A ≥ B + A ≥B⇔   A ≤ −B + A + B ≤ A + B . DÊu “=” x¶y ra khi vµ chØ khi A, B Cïng dÊu + A − B ≤ A − B . DÊu “=” x¶y ra khi vµ chØ khi A ≥ B ≥ 0 hoÆc A≤B≤0 Sinh viªn: NguyÔn Xu©n L ¬ng Líp C§SP To¸n – Tin K48 4 §Ò tµi: Chøng minh bÊt ®¼ng thøc vµ c¸c øng dông + A > B ⇔ A 2 > B2 + a 2 ≥ 0 (a 2 = 0 ⇔ a = 0) + a 2 + b 2 ≥ 2ab . (DÊu “=” x¶y ra khi vµ chØ khi a = b ) + a b + ≥ 2 (Víi a, b cïng dÊu) b a Chó ý: §Ó chøng minh mét bÊt ®¼ng thøc cã nhiÒu c¸ch, tuú thuéc vµo tõng d¹ng cña bµi to¸n. Sau ®©y lµ mét sè c¸ch thêng dïng. II> c¸c ph¬ng ph¸p chøng minh bÊt ®¼ng thøc. 1. P¬ng ph¸p sö dông ®Þnh nghÜa §Ó chøng minh A ≥ B (hoÆc A > B ) ta chøng minh A − B ≥ 0 (hoÆc A − B > 0 ) - Lu ý : A2 ≥ 0 víi mäi A ; dÊu '' = '' x¶y ra khi A = 0 . - VÝ dô : Bµi to¸n 1.1. Chøng minh bÊt ®¼ng thøc C«si ®èi víi hai sè thùc kh«ng ©m ( cßn gäi lµ bÊt ®¼ng thøc ¥clit ) a + b ≥ ab a,b ∈ R* 2 DÊu “ = “ x¶y ra khi vµ chØ khi a = b ThËt vËy, a + b ≥ ab ⇔ a + b − 2 ab ≥ 0 ⇔ ( a − b)2 ≥ 0 2 Víi mäi a,b ≥ 0. DÊu “ = “ x¶y ra khi vµ chØ khi a = b . Bµi to¸n 1.2. 2 2 2   Chøng minh a + b + c ≥  a + b + c ÷ víi mäi sè thùc a, b, c 3 3   2 Ph©n tÝch: §©y lµ mét ®¼ng thøc kh¸ quen thuéc, ta cã thÓ gi¶i b»ng c¸ch xÐt hiÖu vÕ tr¸i vµ vÕ ph¶i. Lêi gi¶i: XÐt hiÖu a 2 + b 2 + c2 3  a + b + c  3a 2 + 3b2 + 3c2 − (a + b + c)2 = − 3 ÷ 9  (a − b)2 + (b − c)2 + (c − a)2 2 = 9 Sinh viªn: NguyÔn Xu©n L ¬ng Líp C§SP To¸n – Tin K48 ≥0 5 §Ò tµi: Chøng minh bÊt ®¼ng thøc vµ c¸c øng dông a 2 + b 2 + c2 ≥  a + b + c  VËy  3 3 ÷  DÊu “=” x¶y ra ⇔ a = b = c 2 2 2 2  a +b+c  a + b + c Do ®ã ≥ 3 3 ÷  2 Khai th¸c bµi to¸n: - B»ng ph¬ng ph¸p xÐt dÊu cña hiÖu A − B ta xÐt ®îc sù ®óng ®¾n cña bÊt ®¼ng thøc A ≥ B . §Ó ý r»ng víi 2 sè thùc bÊt k× u, v ta cñng cã: u 2 + v2 ≥  u + v  2  2 ÷ 2   - t¬ng tù nh chøng minh trªn ta cã thÓ chøng minh bµi to¸n sau Bµi to¸n 1.3 . Víi mäi sè : x, y, z chøng minh r»ng : x2 + y2 + z2 +3 ≥ 2(x + y + z) Lêi gi¶i: Ta xÐt hiÖu : H = x2 + y2 + z2 +3 - 2( x + y + z) = x2 + y2 + z2 +3 - 2x - 2y - 2z = (x2 - 2x + 1) + (y2 - 2y + 1) + (z2 - 2z + 1) = (x - 1)2 + (y - 1)2 + (z - 1)2 Do (x - 1)2 ≥ 0 víi mäi x (y - 1)2 ≥ 0 víi mäi y (z - 1)2 ≥ 0 víi mäi z => H ≥ 0 víi mäi x, y, z Hay x2 + y2 + z2 +3 ≥ 2(x + y + z) víi mäi x, y, z . DÊu b»ng x¶y ra x = y = z = 1. Khai th¸c bµi to¸n: T¬ng tù ta cã thÓ chøng minh bµi to¸n sau: Cho a, b, c, d, e lµ c¸c sè thùc : Chøng minh r»ng : a2 + b2 + c2 + d2 + e2 ≥ a(b + c + d + e) Bµi to¸n 1.4. Chøng minh r»ng: a + b ≥ 2 víi mäi a, b cïng dÊu b a Sinh viªn: NguyÔn Xu©n L ¬ng Líp C§SP To¸n – Tin K48 6 §Ò tµi: Chøng minh bÊt ®¼ng thøc vµ c¸c øng dông Lêi gi¶i: 2 2 a + b − 2ab (a − b) Ta cã: a + b − 2 = = b a 2 ab a, b cïng dÊu ⇒ ab 2 (a − b) ≥0 ab > 0 ⇒ ab VËy a + b ≥ 2 dÊu “=” x¶y ra khi vµ chØ khi a − b = 0 hay a = b b a Khai th¸c bµi to¸n: 1.4.1 Chøng minh t¬ng tù nh trªn ta cã thÓ chøng minh ®îc bµi To¸n sau Chøng minh r»ng víi mäi x tho¶ m·n 1 ≤ x ≤ 5, ta cã : 5 - x + x − 1 ≥ 2. Híng dÈn: 5- x + x −1 ≥ 2 ⇔ ( ) 5 - x + x − 1 ≥ 2 ≥ 4 ⇔ 4 + 2 ( 5 − x )( x − 1) ≥ 4 2  x = 5   ⇔ 2 ( 5 − x )( x − 1) ≥ 0  § óng dÊu b»ng khi   x = 1    1.4.2 Chøng minh bÊt ®¼ng thøc: ab + bc + ca < c víi a ,b lµ c¹nh 2 2 gãc vu«ng cña tam gi¸c ABC, cßn c lµ c¹nh huyÒn. Híng dÈn: Ta cã : ab + bc + ca < 2.c2 hay ab + bc + ca < a2 + b2 + c2 XÐt: a2 + b2 + c2 – ab – bc – ca = 1 2a 2 + 2b 2 + 2c 2 − 2ab − 2bc − 2ca ) = ( 2 1 2 ( a − b ) + (b − c) 2 + (c − a)2 > 0 2 ( ) Bµi to¸n 1.5. Chøng minh r»ng nÕu a.b ≥ 1 th×: Ph©n tÝch: 1 + 1 ≥ 2 . 1+ a 2 1+ b2 1+ ab Sinh viªn: NguyÔn Xu©n L ¬ng Líp C§SP To¸n – Tin K48 7 §Ò tµi: Chøng minh bÊt ®¼ng thøc vµ c¸c øng dông Cñng cã thÓ xÐt hiÖu 2 vÕ th× míi sö dông ®îc gi¶ thiÕt a.b ≥ 1 ( ⇔ ab −1 ≥ 0 ) Lêi gi¶i: XÐt hiÖu: 1 + 1 − 2 = 1 − 1 + 1 − 1 1+ a 2 1+ b2 1+ ab 1+ a 2 1+ ab 1+ b2 1+ ab 2 −1) ≥ 0 = (b − a) (ab 2 (1+ ab)(1+ a )(1+ b2 ) Khai th¸c bµi to¸n: - Víi 3 sè d¬ng a, b, c mµ abc ≥ 1 , bÊt ®¼ng thøc sau ®óng hay sai? Chóng ta cã thÓ ph¸t triÓn bµi to¸n tæng qu¸t hay kh«ng? NÕu ®îc, h·y ph¸t biÓu bµi to¸n tæng qu¸t. 1 + 1 + 1 ≥ 3 1+ a 2 1+ b2 1+ c2 1+ abc - Víi 2 sè x, y mµ x + y ≥ 0 ta cã: 1 + 1 ≥ 2 y x 1 + 4 1 + 4 1 + 2x + y 2. Ph¬ng ph¸p biÕn ®æi t¬ng ®¬ng - §Ó chøng minh A ≥ B ta biÕn ®æi t¬ng ®¬ng A ≥ B ⇔ …⇔ C ≥ D trong ®ã bÊt ®¼ng thøc cuèi cïng C ≥ D lµ mét bÊt ®¼ng thøc hiÓn nhiªn ®óng hoÆc lµ bÊt ®¼ng thøc ®¬n gi¶n h¬n bÊt ®¼ng thøc A ≥ B . Sau khi kh¼ng ®Þnh ®îc tÝnh ®óng ®¾n cña bÊt®¼ng thøc C ≥ D ta kÕt luËn bÊt ®¼ng thøc A ≥ B ®óng - Mét sè h»ng ®¼ng thøc thêng dïng : (A+B)2=A2+2AB+B2 (A-B)2=A2-2AB+B2 (A+B+C)2=A2+B2+C2+2AB+2AC+2BC (A+B)3=A3+3A2B+3AB2+B3 (A-B)3=A3-3A2B+3AB2-B3 Bµi to¸n 2.1. Chøng minh r»ng ∀a, b, c, d ∈ R th× a 2 + b2 + c2 + d 2 + e2 ≥ a(b +c +d +e) Sinh viªn: NguyÔn Xu©n L ¬ng Líp C§SP To¸n – Tin K48 8 §Ò tµi: Chøng minh bÊt ®¼ng thøc vµ c¸c øng dông Lêi gi¶i. BÊt ®¼ng thøc ®ang xÐt t¬ng ®¬ng víi bÊ ®¼ng thøc sau: (nh©n hai vÕ víi 4, chuyÓn vÕ) (a 2 − 4ab + 4b2 ) + (a 2 − 4ac + 4c2 ) + (a 2 − 4ad + 4d 2 ) +(a 2 − 4ae + 4e2 ) ≥ 0 ⇔ (a − 2b)2 + (a − 2c)2 + (a − 2d)2 + (a − 2e)2 ≥ 0 lµ h»ng ®óng . Bµi to¸n 2.2. Cho a, b, c lµ c¸c sè thùc. Chøng minh r»ng: Lêi gi¶i: a 2 + b2 +1 ≥ ab + a + b BÊt ®¼ng thøc a 2 + b2 +1 ≥ ab + a + b ⇔ (a 2 + b 2 +1) − 2(ab + a + b) ≥ 0 ⇔ (a 2 − 2ab + b2 ) + (a 2 − 2a +1) + (b 2 − 2b +1) ≥ 0 ⇔ (a − b)2 + (a −1)2 + (b −1)2 ≥ 0 ®óng ⇒ §iÒu cÇn chøng minh Khai th¸c bµi to¸n: T¬ng tù nh bµi to¸n trªn h·y chøng minh bÊt ®¼ng thøc sau: Cho a, b, c lµ c¸c sè thùc. Chøng minh r»ng: a + b + c ≥ 2 1 + 1 + 1  bc ca ab  a b c ÷ Bµi to¸n 2.3. ∀x, y chøng minh r»ng Lêi gi¶i: Ta cã: x 4 + y4 ≥ xy3 + x 3 y VËy Bµi to¸n 2.4. x 4 + y4 − xy3 − yx 3 = x 3 (x − y) − y3 (x − y) 2   = (x − y)2  (x + y )2 + 3y  ≥ 0 2 4   x 4 + y4 ≥ xy3 + x 3 y 3 3 3 Chøng minh r»ng a + b + c − 3abc ≥ 0 (1) Lêi gi¶i. a +b+c Sinh viªn: NguyÔn Xu©n L ¬ng Líp C§SP To¸n – Tin K48 9 §Ò tµi: Chøng minh bÊt ®¼ng thøc vµ c¸c øng dông Ta cã: (1) ( a + b + c) ( a ⇔ 2 + b2 + c2 − ab − ac − bc ) ≥0 a +b+c 1 ⇔ (a − b)2 + (b − c)2 + (c − a)2 ≥ 0 2 (2) (2) ®óng ⇒ (1) ®óng Bµi to¸n 2.5. Chøng minh r»ng a 2 (1+ b2 ) + b2 (1+ c2 ) + c2 (1 + a 2 ) ≥ 6abc (1) Lêi gi¶i: (1) ⇔ (a − bc)2 + (b − ac)2 + (c − ab)2 ≥ 0 (2) ®óng ⇒ (1) ®óng Khai h¸c bµi to¸n: T¬ng tù nh trªn ta cã thÓ chøng minh bµi to¸n sau 2.5.1 (2) Cho a > 0; b > 0 vµ a 3 + b 3 = a − b. Chøng minh r»ng: Híng dÈn: a 2 + b 2 + ab < 1. a 3 + b 3 = a - b ⇔( a 3 - b 3 ) ( a 2 + b 2 + ab ) = ( a - b ) ( a 2 + b 2 + ab ) a3 - b3 ⇔( a - b ) ( a + b + ab ) = a - b ⇔a + b + ab = 3 a + b3 a3 - b3 2 2 VËy a + b + ab < 1 ⇔ 3 < 1 ⇔a 3 - b 3 < a 3 + b 3 ⇔0 < b 3 3 a +b 3 3 2 2 3 3 2 2 2.5.2 Chøng minh víi mäi sè dong a, b, c ta lu«n cã : ( a 2 + b 2 b 2 + c2 c2 + a 2 3 a 2 + b 2 + c2 + + ≤ a+b b+c c+a a+b+c Híng dÈn: ) Sinh viªn: NguyÔn Xu©n L ¬ng Líp C§SP To¸n – Tin K48 10 §Ò tµi: Chøng minh bÊt ®¼ng thøc vµ c¸c øng dông  a 2 + b 2 b 2 + c2 c2 + a 2   ≤ 3 a 2 + b 2 + c 2 B§ T ⇔ ( a + b + c )  + + b+c c+a   a+b c a 2 + b 2 a b 2 + c2 b c2 + a 2 ⇔ + + ≤ a 2 + b 2 + c2 a+b b+c c+a 2 2 2 ac( c − a ) bc( c − b ) ab( b − a ) ⇔ + + ≥ 0( § óng ) ( a + b )( b + c ) ( a + b )( a + c ) ( c + a )( b + c) 3. Ph¬ng ph¸p quy n¹p to¸n häc - KiÕn thøc : §Ó chøng minh mét bÊt ®¼ng thøc ®óng víi n > 1 b»ng ph¬ng ph¸p quy n¹p to¸n häc , ta tiÕn hµnh : + KiÓm tra bÊt ®¼ng thøc ®óng víi n = 1 (n = n0) + Gi¶ sö bÊt ®¼ng thøc ®óng víi n = k > 1 (k > n0) + Chøng minh bÊt ®¼ng thøc ®óng víi n = k + 1 + KÕt luËn bÊt ®¼ng thøc ®óng víi n > 1 (n > n0) Chó ý: Khi chøng minh bÊt ®¼ng thøc cã n sè (n ∈N) Th× ta nªn chó ý sö dông ph¬ng ph¸p quy n¹p to¸n häc - VÝ dô : Bµi to¸n 3.1. Chøng minh bÊt ®¼ng thøc C«si trong trêng hîp tæng qu¸t. Víi a1 , a 2 ... a n ∈ R n , n ≥ 2 th× ( ( ) ( ) ( ) ) a1 + a 2... + a n ≥ n a .a ...a 1 2 n n Lêi gi¶i: Ta dïng ph¬ng ph¸p quy n¹p theo n : • Víi n =2 bÊt ®¼ng thøc ®¶ ®îc chøng minh ë 1. (bÊt ®¼ng thøc ¥clit) • §Ó chøng minh bÊt ®¼ng thøc tæng qu¸t, trø¬c hÕt ta h·y xÐt vµi bÊt ®¼ng thøc phô. NÕu x1, x 2 ∈ R + th× x1 < x 2 ⇔ x1n −1 < x 2n −1 . VËy ∀x1, x 2 ∈ R + th× ta lu«n cã (chuyÓn mét bé phËn sang vÕ ph¶i, ta ®îc) (x1n −1 − x 2n −1)(x1 − x 2 ) ≥ 0 x1n + x 2n ≥ x1x 2n −1 + x 2 x1n −1. LÊy n sè thùc kh«ng ©m x1,x 2...x n ∈ R + , viÕt c¸c bÊt ®¼ng Sinh viªn: NguyÔn Xu©n L ¬ng Líp C§SP To¸n – Tin K48 11 §Ò tµi: Chøng minh bÊt ®¼ng thøc vµ c¸c øng dông thøc t¬ng øng råi céng l¹i ta ®îc: (x1n + x 2 n ) + (x1n + x 3 n ) + ... + (x1n + x n n ) + +(x 2 n + x 3n ) + ... + (x 2 n + x n n ) + ... + (x n −1n + x n n ) ≥ (x1 x 2 n −1 + x 2 x 1n −1 ) + +(x1 x 3n −1 + x 3 x1n −1 ) + ... + (x1x n n −1 + x n x 1n −1 ) + ... n −1 n −1 x + x x ) (*) −1 nn n n −n1 n (n −n1)(x + x + ... + x ) ≥ 1 2 n n −1 n −1 n −1 x1 (x 2 + x 3 + ... + x n ) + (**) + x 2 (x1n −1 + x3n −1 + ... + x n n −1 ) + Tõ ®ã:+(x + x n (x1n − 2 + x 2n −1 + ... + x n −1n −1 ) B©y giê theo gi¶ thiÕt quy n¹p, ta thõa nhËn r»ng ®èi víi n −1 sè thùc kh«ng ©m bÊt k× , trung b×nh céng kh«ng nhá h¬n trung b×nh nh©n cña chóng. ThÕ th× nãi riªng ta cã: x 2n −1 + x3n −1 + ... + x n n −1 ≥ (n −1)x 2x 3...x n x1n −1 + x3n −1 + ... + x n n −1 ≥ (n −1)x1x 3...x n ………………………………………………… x1n −1 + x 2n −1 + ... + x n −1n −1 ≥ (n −1)x 2 x 2...x n −1 Sö dông c¸c bÊt ®¼ng thøc nµy, ta cã thÓ t¨ng cêng c¸c bÊt ®¼ng Thøc ( ** ) (n −1)(x1n + x 2 n + ... + x n n ) ≥ n(n −1)x1x 2...x n ) Trong hÖ thøc nµy ®Æt x1n = a1,x 2 n = a 2 ,...x n n = a n ta ®îc a1 + a 2... + a n ≥ n a .a ...a ( ®pcm ) 1 2 n n Trong tÊt c¶ qu¸ tr×nh lý luËn trªn, dÊu “=” x¶y ra khi vµ chØ khi x1 = x 2 = ... = x n tøc lµ khi vµ chØ khi a1 = a 2 = ... = a n Trong lý thuyÕt ®¶ cã mét sè bÊt ®¼ng thøc ®îc chóng minh b»ng ph¬ng ph¸p quy n¹p (bÊt ®¼ng thøc C«si, Becnuli, . . .) Sau ®©y ta xÐt mét sè bµi to¸n kh¸c. Bµi to¸n 3.2. Sinh viªn: NguyÔn Xu©n L ¬ng Líp C§SP To¸n – Tin K48 12 §Ò tµi: Chøng minh bÊt ®¼ng thøc vµ c¸c øng dông 2 2 2 Tæng qu¸t cña bÊt ®¼ng thøc u + v ≥  u + v ÷ 2 2   Cho a, b lµ hai sè d¬ng, chøng minh r»ng víi mäi sè tù nhiªn n ≥ 2 Ta cã: u n + vn ≥  u + v  n  2 ÷ 2   Ph©n tÝch: ViÖc xÐt hiÖu trùc tiÕp kh«ng ®¹t ®îc kÕt qu¶ v× vËy chóng ta cã thÓ nghÜ ®Õn c¸ch sö dông ph¬ng ph¸p quy n¹p. Lêi gi¶i: Víi n = 2 ta cã: a +b ≥ a+b   ÷ 2 2 2 2 (b»ng c¸ch xÐt hiÖu). 2  Gi¶ sö bÊt ®¼ng thøc ®óng víi n = k , tøc lµ k a k + bk ≥  a + b   ÷ 2  2   Ta ph¶i chøng minh bÊt ®¼ng thøc cñng ®óng víi n = k +1, tøc lµ ThËt vËy, k +1 a k +1 + bk +1 ≥  a + b   ÷ 2  2  k k +1 a k + b k ≥  a + b  ⇔ a + b . a k + bk ≥  a + b   ÷  ÷ 2 2 2  2   2  Ta chøng minh: ⇔ ⇔ ⇔ ⇔ a k +1 + bk +1 ≥ a + b . a k + bk 2 2 2 a k +1 + bk +1 ≥ abk + a k b a k +1 − a k b + bk +1 − abk ≥ 0 (a k − bk )(a − b) ≥ 0 (a − b)2 (a k + a k −2b + ... + ab2−k + bk −1) ≥ 0 (®óng) Khai th¸c bµi to¸n: a) Bµi to¸n vÈn ®óng trong trêng hîp a ≥ 0; b ≥ 0 Sinh viªn: NguyÔn Xu©n L ¬ng Líp C§SP To¸n – Tin K48 13 §Ò tµi: Chøng minh bÊt ®¼ng thøc vµ c¸c øng dông n n b) Víi a + b = 2 ta cã a + b ≥ 1 Bµi to¸n 3.3. 2 ∀n ∈ N , n >1, chøng minh r¼ng: 1 + 1 + ... + 1 > 13 n +1 n + 2 2n 24 Lêi gi¶i: Víi n = 2 tacã VT = 1 + 1 = 7 = 14 > 13 = VP 3 4 12 24 24 Gi¶ sö bÊt ®¼ng thøc ®óng víi n , nghÜa lµ ta cã: 1 + 1 + ... + 1 > 13 n +1 n + 2 2n 24 Ta ph¶i chøng minh bÊt ®¼ng thøc ®óng víi n +1 , nghÜa lµ ph¶i chøng minh: Ta cã 1 + 1 + ... + 1 > 13 n +1 n + 2 2(n +1) 24 VT = 1 + ... + 1 + ( 1 + 1 − 1 ) n +1 2n 2n +1 2n + 2 n +1 1 = 1 + ... + 1 + > 13 = VP n +1 2n (2n +1)(2n + 2) 24 ⇒ BÊt ®¼ng thøc ®óng víi n + 1 KÕt luËn : bÊt ®¼ng thøc ®óng víi ∀n ∈ N , n >1. T¬ng tù nh trªn ta cã thÓ chøng minh c¸c bÊt ®¼ng thøc sau 1) Cho a,b,c lµ 3 c¹nh cña mét tam gi¸c vu«ng víi c lµ c¹nh huyÒn . Chøng minh r»ng: a 2n + b2n ≤ c2n ∀n ∈ N 2) ∀n ∈ N , Chøng minh r»ng: 22n + 2 > 2n + 5 3) ∀n ∈ N , n >1, chøng minh r¼ng: 1 + 1 + ... + 1 < 2 − 1 n 12 22 n2 Bµi to¸n 3.4. Chøng minh r»ng víi a,b,c,d,e ∈ ( 0;1) th× ( 1 − a ) ( 1 − b) ( 1 − c) ( 1 − d) ( 1 − e) > 1 − a − b − c − d − e Vµ h·y chøng minh mét kÕt qu¶ tæng qu¸t h¬n kÕt qu¶ cña bµi to¸n trªn. Sinh viªn: NguyÔn Xu©n L ¬ng Líp C§SP To¸n – Tin K48 14 §Ò tµi: Chøng minh bÊt ®¼ng thøc vµ c¸c øng dông lêi gi¶i: Ta sÎ chøng minh kÕt qu¶ tæng qu¸t sau ®©y Víi a1 ,a 2 ,...,a n ∈ ( 0;1) ( n ≥ 2 ) ⇒ ( 1 − a1 ) ( 1 − a 2 ) ...( 1 − a n ) > 1 − a 1 − a 2 − ... − a n Chøng minh b»ng quy n¹p to¸n häc theo n - Víi n = 2 ⇒ ( 1 − a 1 ) ( 1 − a 2 ) = 1 − a 1 + a 1a 2 > 1 − a 1 − a 2 - Gi¶ sö k¼ng ®Þnh ®ón víi n = k , ta sÎ chøng minh kh¼ng ®Þnh cñng ®óng víi n = k + 1 Do kh¼ng ®Þnh ®óng víi n = k ⇒ ( 1 − a1 ) ( 1 − a 2 ) ...( 1 − a k ) > 1 − a1 − a 2 − ... − a k Víi 0 < 1 ⇒ 1 − a k −1 > 0 ⇒ ( 1 − a ) ( 1 − a ) ...( 1 − a ) ( 1 − a ) > ( 1 − a 1 Mµ vÕ ph¶i b»ng 2 k +1 k 1 − a 2 − ... − a k ) ( 1 − a k +1 ) 1 − a1 − a 2 − ... − a k − a k +1 + ( a1 + a 2 + ... + a k ) a k +1 1 4 42 4 43 >0 > 1 − a1 − a 2 − ... − a k − a k +1 ⇒ ( 1 − a1 ) ( 1 − a 2 ) ... ≥ ( 1 − a k +1 ) > 1 − a1 − a 2 − ... − a k +1 VËy kh¼ng ®Þnh ®óng víi ∀n > 2 4. Ph¬ng ph¸p tam thøc bËc hai a) C¸c tÝnh chÊt cña tam thøc bËc hai th¬ng dïng trong bÊt ®¼ng thøc *. F(x) = ax 2 + bx + c ∀x ∈ R (a ≠ 0) a > 0 F(x) ≥ 0 ⇔  ∆ ≤ 0  *. a ≤ x ≤ b ⇔ (x − a)(x − b) ≤ 0 *. F(x) = ax 2 + bx + c ≥ 4ac-b 2 4a ∀x ∈ R (a > 0) b) Ph¬ng ph¸p. *> Ph¬ng ph¸p 1: §Ó chøng minh bÊt ®¼ng thøc M > N ta biÕn ®æi Sinh viªn: NguyÔn Xu©n L ¬ng Líp C§SP To¸n – Tin K48 15 §Ò tµi: Chøng minh bÊt ®¼ng thøc vµ c¸c øng dông M > N ⇔ B2 − 4AC ≤ 0 (A > 0) XÐt tam thøc F(x) = Ax 2 + Bx + C ta chØ cÇn chøng minh F(x) ≥ 0 ∀x ∈ R *> Ph¬ng ph¸p 2: §Ó chøng minh bÊt ®¼ng thøc M > N ta biÕn ®æi M > N ⇔ B2 − 4AC ≥ 0 . XÐt tam thøc F(x) = Ax 2 + Bx + C Ta chØ cÇn chøng minh: ∃x 0 / aF(x 0 ) ≤ 0 *> Ph¬ng ph¸p 3: §Ó chøng minh bÊt ®¼ng thøc M > N ta biÕn ®æi M > N ⇔ Ax 2 ± Bx + C ≥ 0 ∀x vµ chØ cÇn chøng minh: B2 − 4AC ≤ 0  A > 0 Bµi to¸n 4.1. Cho a, b lµ c¸c sè tho¶ m¶n ®iÒu kiÖn a 2 − a + 2b + 4b2 − 4ab ≤ 0 Chøng minh r»ng 0 ≤ a − 2b ≤ 1 (1) Ph©n tÝch §Ó ý r»ng bÊt ph¬ng tr×nh bËc hai at 2 + bt + c ≤ 0 (a > 0) ⇔ t1 < t < t 2 trong ®ã t1 , t 2 lµ c¸c nghiÖm cña tam thøc at 2 + bt + c ta cã lêi gi¶i sau. Lêi gi¶i: (1) ⇔ a 2 − 4ab + 4b2 − (a − 2b) ≤ 0 ⇔ (a − 2b)2 − (a − 2b) ≤ 0 §Æt t = a − 2b ⇒ t 2 − t ≤ 0 ⇔ 0 ≤ t ≤ 1 ⇔ 0 ≤ a − 2b ≤ 1 Khai th¸c bµi to¸n: Ta ®· dïng ®Þnh lý vÒ dÊu tam thøc bËc hai ®Ó gi¶i bµi to¸n nµy. nªn ta cã thÓ gi¶i c¸c bµi to¸n sau b»ng mét ph¬ng ph¸p kh¸c ®¬n gi¶n: T×m gi¸ trÞ lín nhÊt (bÐ nhÊt) nÕu cã cña c¸c biÓu thøc: Sinh viªn: NguyÔn Xu©n L ¬ng Líp C§SP To¸n – Tin K48 16 §Ò tµi: Chøng minh bÊt ®¼ng thøc vµ c¸c øng dông 2 + 2x + 2003 x y= x2 y = −x 2 + 3x − 2 C¨n cø vµo ®Æc ®iÓm Parabol y = a.x 2 + bx + c víi a > 0 ( a < 0 ) quay  −b , −∆ ÷ lµ ®iÓm cã tung ®é  2a 4a  bÒ lâm lªn trªn (xuèng díi), do ®ã ®Ønh S   bÐ nhÊt (lín nhÊt), ta cã thÓ thªm mét c¸ch t×m gi¸ trÞ lín nhÊt (bÐ nhÊt) cña c¸c bÓu thøc cã d¹ng y = a.x 2 + bx + c ( a ≠ 0 ) Bµi to¸n 4.2. ∀x, y ∈ R , chøng minh bÊt ®¼ng thøc sau: x 2 y4 + 2(x 2 + 2)y2 + 4xy + x 2 ≥ 4xy3 (1) Lêi gi¶i: (1) ⇔ (y2 +1)2 x 2 + 4y(1− y2 )x + 4y2 ≥ 0 F(x) = (y2 +1)2 x 2 + 4y(1− y2 )x + 4y2 ∆' = 4y2 (1− y2 )2 − 4y2 (y2 +1)2 ∆' = −16y2  '  f (x) ≥ 0 ∆ ≤ 0 ⇒    ∀y ∈ R ∀x, y ∈ R   Bµi to¸n 4.3. Víi a,b,c,d ∈ R , chøng minh bÊt ®¼ng thøc sau: (1) (a + b + c + d)2 > 8(ac + bd) . Lêi gi¶i: (1) XÐt tam thøc  a 2 + 2(b − 3c + d)a + (b + c + d)2 − 8bd > 0 ⇔   ∀a ∈ R;b < c < d  F(a) = a 2 + 2(b − 3c + d)a + (b + c + d)2 − 8bd ∆' = (b − 3c + d)2 − (b + c + d)2 = 8bd ∆' = 8(c − b)(c − d) ∆' < 0 ⇒ F(a) > 0 ∀a ∈ R Sinh viªn: NguyÔn Xu©n L ¬ng Líp C§SP To¸n – Tin K48 17 §Ò tµi: Chøng minh bÊt ®¼ng thøc vµ c¸c øng dông VËy (1) ®óng Bµi to¸n 4.4. Chøng minh bÊt ®¼ng thøc C«si – Bunhiac«pski. Cho n cÆp sè thùc bÊt k× a i , bi , i =1,...,n. thÕ th× (a1b1 + a 2b2 + ... + a n bn ) ≤ (a12 + a 22 + ... + a n 2 )(b12 + b22 + ... + bn 2 ) DÊu “=” x¶y ra khi vµ chØ khi tån t¹i sè k ∈ R sao cho b1 = ka1 , b2 = ka 2 , …, bn = ka n Lêi gi¶i: Víi ∀x ∈ R ta cã: (a1x − b1)2 ≥ 0 ………………. Tõ ®ã suy ra: (a n x − bn )2 ≥ 0 a12 x 2 − 2a1b1x + b12 ≥ 0 ………………………….. a n 2 x 2 − 2a n bn x + bn 2 ≥ 0 Céng vÕ víi vÕ ta ®îc (a12 + a 22 + ... + a n n )x 2 − 2(a1b1 + a 2b2 + ... + a n bn )x +(b12 + b22 + ... + bn n ) ≥ 0 VÕ tr¸i lµ mét tam thøc bËc hai f ( x ) = Ax 2 − 2B'x + C víi A ≥ 0 Vµ f (x) ≥ 0 ∀x ∈ R nªn nÕu A > 0 th× 2 ∆' = B' − AC = (a1b1 + a 2b2 + ... + a n bn ) + (a12 + a 22 + ... + a n 2 )(b12 + b22 + ... + bn 2 ) ≤ 0 Vµ thu ®ù¬c bÊt ®¼ng thøc cÇn chøng minh. Cßn nÕu A = 0 th× a1 = a 2 = ... = a n khi ®ã bÊt ®¼ng thøc cÇn chøng minh lµ hiÓn nhiªn. Cuèi cïng ta thÊy dÊu “=” trong bÊt ®¼ng thøc x¶y ra khi vµ chØ khi ∆' = 0 ⇔ a1x = b1 = ... = a n x = bn ⇔ b1 = ka1,...,bn = ka n Víi k ∈ R . Khai th¸c bµi to¸n: T¬ng tù nh bµi to¸n trªn ta cã thÓ chøng minh c¸c bÊt ®¼ng thøc sau: 1) 5x 2 + 3y2 + 4xy − 2x + 8y + 9 ≥ 0 ∀x, y ∈ R 2) 3y2 + x 2 + 2xy + 2x + 6y + 4 ≥ 1 ∀x, y ∈ R Sinh viªn: NguyÔn Xu©n L ¬ng Líp C§SP To¸n – Tin K48 18 §Ò tµi: Chøng minh bÊt ®¼ng thøc vµ c¸c øng dông 3) (x + y)2 − xy +1 ≥ (x + y) 3 Bµi to¸n 4.5. Cho a.b ≠ 0 Chøng minh r»ng: a 2 b2 a b + − 3  + ÷+ 4 ≥ 0 b2 a 2 b a Lêi gi¶i: a b §Æt x = + b a BÊt ®¼ng thøc trë thµnh: ta cã : a 2 b2 x = 2 + 2 +2 b a 2 2 a b ⇒ + 2 = x2 − 2 2 b a 2 x 2 − 2 − 3x + 4 ≥ 0 ⇔ x 2 − 3x + 2 ≥ 0 ⇔ ( x − 1) ( x − 2 ) ≥ 0 NÕu ab < 0 Th× ta cã a 2 + 2ab + b 2 ≥ 0 ⇒ a 2 + b 2 ≥ −2ab Chia c¶ hai vÕ cho ab ta ®îc a 2 + b2 VËy x ≤ −2 ≤ −2 ab Trong c¶ hai trêng hîp th× ( x − 1) ( x − 2 ) ≥ 0 DÊu ®¼ng thøc x¶y ra khi a = b 5. Ph¬ng ph¸p sö dông bÊt ®¼ng thøc Cauchy #>Víi hai sè a, b ≥ 0 ta lu«n cã: a + b ≥ ab 2 DÊu “=” x¶y ra khi vµ chØ khi a = b Chøng minh: C¸ch1: (Ph¬ng ph¸p biÕn ®æi t¬ng ®¬ng) 2 a+b a+b ≥ a.b ⇔  ≥ ab ⇔ (a − b) 2 ≥ 0 B®t hiÓn nhiªn ÷ 2  2  ®óng. Sinh viªn: NguyÔn Xu©n L ¬ng Líp C§SP To¸n – Tin K48 19 §Ò tµi: Chøng minh bÊt ®¼ng thøc vµ c¸c øng dông §¼ng thøc x¶y ra ⇔ a = b . C¸ch 2: (Ph¬ng ph¸p h×nh häc) + NÕu a = 0 hoÆc b = 0 th× B§T hiÓn nhiªn ®óng. + NÕu a > 0 vµ b > 0 th× ta ®Æt: HA = a , HB = b ( H×nh vÏ ) a+b = OI ≥ HC = HA.HB = a.b 2 §¼ng thøc x¶y ra ⇔ HC = OI ⇔ H ≡ O ⇔ a = b ⇒ H A O #> D¹ng tæng qu¸t cña bÊt d¼ng thøc Cauchy Cho a1,a 2 ,...,a n ≥ 0 I C a1 + a 2 + ... + a n ≥ n a a ...a 1 2 n n DÊu “=” x¶y ra khi vµ chØ khi a1 = a 2 = ... = a n Ta cã: Chøng minh b»ng ph¬ng ph¸p quy n¹p Bµi to¸n 5.1. 1 1 +1≥9. ÷ a b c Cho a,b,c > 0.Chøng minh ( a + b +c )  + Ph©n tÝch: VÕ tr¸i chøa a,b,c > 0 vµ c¸c nghÞch ®¶o cña chóng.v× vËy ta nghÜ ®Õn viÖc dïng bÊt d½ng thøc c«si. Lêi gi¶i: C¸ch 1 ¸p dông bÊt ®¼ng thøc C«si cho c¸c bé 3 sè a,b,c vµ 1 , 1 , 1 a b c ta cã: a + b + c ≥ 3 3 abc (1). 1 + 1 + 1 ≥ 33 1 (2) a b c abc 1 1 1 b  Nh©n tõng vÕ cña (1) vµ(2)ta ®ù¬c: ( a + b + c )  + + ÷≥ 9 (®pcm) a c  C¸ch 2: ( a + b + c )  1a + 1b + 1c ÷ = 3 +  ba + ac ÷+  ac + ac ÷ +  bc + bc ÷ ≥ 3 + 2 + 2 + 2 = 9   DÊu “=”x¶y ra ⇔ a = b = c         Khai th¸c bµi to¸n: Sinh viªn: NguyÔn Xu©n L ¬ng Líp C§SP To¸n – Tin K48 20 B §Ò tµi: Chøng minh bÊt ®¼ng thøc vµ c¸c øng dông T¬ng tù nh trªn ta cã thÓ chøng minh ®îc bÊt ®¼ng thøc sau Cho a, b,c > 0 vµ a + b + c + d = 1 Chøng minh r»ng a+b+c + b+c+d + b+d+a + c+d+a ≤ 2 3 Bµi to¸n 5.2. Cho a, b, c > 0 vµ a + b + c ≤ 1. Chøng minh r»ng Lêi gi¶i: 1 1 1 + + ≥9 a 2 + 2bc b 2 + 2ca c 2 + 2ab 1 1 1 + + a 2 + 2bc b 2 + 2ca c 2 + 2ab 1 1 1 2  ≥ ( a + b + c)  2 + 2 + 2 ÷  a + 2bc b + 2ca c + 2ab  = ( a 2 + 2bc ) + ( b 2 + 2ca ) + ( c 2 + 2ab )  . 1 1 1   + +  2 ÷≥ 9 2 2  a + 2bc b + 2ca c + 2ab  Khai th¸c bµi to¸n. Chøng minh t¬ng tù nh trªn ta cã thÓ chøng minh ®îc c¸c bµi to¸n sau 5.2.1. Chøng minh r»ng víi mäi a,b > 0 tho¶ m·n a + b = 1 ta cã 5.2.2 1 1 + 2 ≥6 ab a + b 2 Chøng minh r»ng víi mäi a, b > 0 tho¶ m·n : a.b = 1, ta cã: 1 1 2 + + ≥3 a b a+b Híng dÈn: Sinh viªn: NguyÔn Xu©n L ¬ng Líp C§SP To¸n – Tin K48 21 §Ò tµi: Chøng minh bÊt ®¼ng thøc vµ c¸c øng dông 1 1 2 2 a+b a+b 2 + + = ( a + b) + = + + a b a+b a+b 2 2 a+b a+b a+b 2  = + + ÷≥ ab + 2 = 3 2 a+b  2 Bµi to¸n 5.3. 1+1≥ 4 x y x+y Cho x,y >0, chøng minh (1) Ph©n tÝch : Do x,y >0 nªn bÊt ®¼ng thøc (1)cã thÓ suy tõ bÊt ®¼ng thøc C«si hoÆc trùc tiÕp xÐt hiÖu. Lêi gi¶i : C¸ch 1: Sö dông bÊt ®¼ng thøc C«sicho hai sè d¬ng: x + y ≥ 2 xy ⇔ x + y 2 ≥ 4xy ( C¸ch hai : ) ⇔ x+y ≥ 4 . xy x + y XÐt hiÖu cña hai vÕ. Khai th¸c bµi to¸n: BÊt ®¼ng thøc trªn cã liªn quan ®Õn viªc “céng mÉu” nªn cã thÓ sö dông ®Ó chøng minh bÊt ®¼ng thc sau: Bµi to¸n 5.3.1 Cho a, b, c lµ ®é dµi 3 c¹nh cña mét tam gi¸c, chøng minh r»ng: 1 + 1 + 1 ≥ 2  1 + 1 + 1  . Trong ®ã p= a + b + c . p − a p − b p − c  a b c ÷ 2 Bµi to¸n 5.3.2. Cho a > 0;b > 0 , chøng minh r»ng 2 a + 3 b ≥ 5 5 ab Híng dÈn ¸p dông bÊt ®¼ng thøc C«si cho c¸c sè a , a , 3 b , ⇒ Bµi to¸n 5.4. 3 b, 3 b a + a + 3 b + 3 b + 3 b ≥ 5 5 ab 2 a + 3 b ≥ 5 5 ab Cho a,b,c lµ ®é dµi 3 c¹nh cña mét tam gi¸c Chøng minh r»ng: a + b + c ≥ 3. b+c−a a +c−b b+a −c Sinh viªn: NguyÔn Xu©n L ¬ng Líp C§SP To¸n – Tin K48 22 §Ò tµi: Chøng minh bÊt ®¼ng thøc vµ c¸c øng dông Lêi gi¶i: C¸ch 1: NhËn xÐt: Do a,b,c lµ ®é dµi 3 c¹nh cña mét tam gi¸c nªn ta cã a + b − c > 0; a + c − b > 0; b + c − a > 0. ¸p dông bÊt ®¼ng thøc C«si cho c¸c cÆp sè d¬ng: (a + b − c)(a + c − b) ≤ a + b − c + a + c − b = a 2 (a +c-b)(b +c-a) ≤ c (b +c-a)(b +a -c) ≤ b (1) (2) (3) §Ó ý r»ng c¶ 2 vÕ cña c¸c bÊt ®¼ng thøc (1) (2) (3) lµ c¸c sè d¬ng vµ ba bÊt ®¼ng thøc nµy cïng chiÒu, nh©n tõng vÕ cña chóng ta ®îc (a + b − c)(a + c − b)(b + c − a) ≤ abc. Trë l¹i bµi to¸n: a + b + c ≥ 33 abc b+c−a a +c−b b+a −c (b + c − a)(a + c − b)(a + b − c) ≥ 3 3 abc abc C¸ch2: §Æt x = b + c − a;y = a + c − b;z = a + b − c , khi ®ã x, y,z > 0 vµ a = y + z ,b = x + z ,c = x + y . 2 2 2 VÕ tr¸i: a + b + c = 1 ( y+z + x +z + x + y) b+c−a a +c−b b+a −c 2 x y z 1 x y x z y z 1 = ( + + + + + ) ≥ (2 + 2 + 2) = 3 2 y x z x z y 2 x y y+ x =2   x z DÊu “=” x¶y ra ⇔  + = 2 ⇔ x = y = z ⇔ a = b = c. z x y z  + =2  z y Sinh viªn: NguyÔn Xu©n L ¬ng Líp C§SP To¸n – Tin K48 23 §Ò tµi: Chøng minh bÊt ®¼ng thøc vµ c¸c øng dông Khai th¸c bµi to¸n: Trong bµi to¸n trªn chóng ta ®· sö dông Èn phô hoÆc dïng bÊt ®¼ng thøc C«si ®Ó gi¶i. Sö dông c¸ch thøc trªn, h¶y gi¶i bµi to¸n sau: 1) Cho a,b,c > 0 vµ a + b + c + d = 1 Chøng minh r»ng a +b+c + b+c+d + b+d+a + c+d+a ≤ 2 3 2) Cho a,b,c,d > 0 , Chøng minh r»ng: a) (1+ a)(1+ b)(1+ c) ≥ (1+ 3 abc ) b) (a + b)(c + d) + (a + c)(b + d) + + (a + d)(b + c) ≥ 6 4 abcd Bµi to¸n 5.5. Cho x1,x 2 ,...,x n ∈  0;1 , chøng minh r»ng: (1+ x1 + ... + x n )2 ≥ 4(x12 + x 22 + ... + x n 2 ) Lêi gi¶i: Theo C«si ta cã: (1+ x1 + ... + x n )2 ≥ 4(x1 + x 2 + ... + x n ) ≥ 4(x12 + x 22 + ... + x n 2 ) xi ≥ xi2 Do 6. Ph¬ng ph¸p sö dông BÊt ®¼ng thøc Bunhac«pski #> Víi a, b, c, d ∈ R tacã: ac + bd ≤ (a 2 + b2 )(c2 + d 2 ) DÊu “=” x¶y ra khi vµ chØ khi a = c b d #> Trêng hîp tæng qu¸t Cho n cÆp sè Ta cã: a1,a 2 ,...,a n b1,b2 ,...,bn (a1b1 + a 2b2 + ... + a n bn )n ≤ (a12 + a 22... + a n 2 )(b12 + b22 + ... + bn 2 ) a a a DÊu “=” x¶y ra khi vµ chØ khi 1 = 2 = ... = n b1 b2 bn Bµi to¸n 6.1. Sinh viªn: NguyÔn Xu©n L ¬ng Líp C§SP To¸n – Tin K48 24 §Ò tµi: Chøng minh bÊt ®¼ng thøc vµ c¸c øng dông Víi c¸c sè a, b, c > 0 tho¶ m·n ®iÒu kiÖn abc = 1. Chøng minh r»ng : a b c 1 + + ≥ . 2 2 2 a + b + c ( ab + a + 1) ( bc + b + 1) ( ca + c + 1) Lêi gi¶i: VT = = = = a ( ab + a + 1) a ( ab + a + abc ) 1 a ( bc + b + 1) 2 2 + + 2 + b ( bc + b + 1) b ( bc + b + 1) b ( bc + b + 1) 2 2 2 + + + c ( ca + c + 1) 2 cb 2 ( abc + bc + b ) 2 b 2c ( bc + b + 1) 2 1 2  . + b + b c÷  2  ( bc + b + 1)  a 1 ¸ p dông bÊt ®¼ng thøc Bunhia - copxki cho hai bé sè : 1 ; b;b c ta cã : a  1  2 2 2 b + c  .  +   a ÷   a; b; c vµ   ( ) ( ) ( ) a 2 + 2 2 + b c  ≥ ( 1 + b + bc ) hay  ( ) ( b 2 ) ( a + b + c )  1 2  + b + b 2 c ÷ ≥ ( bc + b + 1) a  ⇔ 1 1 2  . + b + b c ≥ ÷ 2   ( a + b + c) ( bc + b + 1)  a 1 Bµi to¸n 6.2. Cho n sè thùc a1,a 2 ,...,a n vµ n sè d¬ng ( n ≥ 1 ). Chøng minh r»ng (§PCM) b1,b2 ,...,bn Sinh viªn: NguyÔn Xu©n L ¬ng Líp C§SP To¸n – Tin K48 25 §Ò tµi: Chøng minh bÊt ®¼ng thøc vµ c¸c øng dông a12 + a 22 + ... + a n 2 ≥ (a1 + a 2 +...+ a n ) b1 b2 bn b + b +...+ b 1 Ph©n tÝch: BÊt ®¼ng thøc trªn t¬ng ®¬ng víi 2 2 n   (a1 + a 2 + ... + a n )2 ≤  ( a1 )2 + ... + ( a n )2  .  ( b1 )2 + ... + ( b n )2  . bn     b1 VËy ta cã lêi gi¶i sau. Lêi gi¶i: ¸p dông bÊt ®¼ng thøc Bunhac«pski cho n cÆp sè: a12 a 22 an2 , ,…, Vµ b1 b2 bn     a1 b1 b1 + a 2 b2 b1 , b2 , …, bn ta cã: b2 + ... + a n bn  bn ÷÷≤  a1 + ... + a n ÷(b1 + ... + bn ) bn   b1  2 2   a12 a n 2 (a1 +...+ a n ) 2 ⇔ + ... + ≥ b1 bn b1 +...+ b n DÊu “=” x¶y ra ⇔ Bµi to¸n 6.3. a1 a2 an b1 b2 bn = = ... = ⇔ a1 = ... = a n . b1 bn b1 b2 bn Cho a,b,c ≥ − 3 ,a + b + c = 3 . Chøng minh r»ng 4 4a + 3 + 4b + 3 + 4c + 3 ≤ 3 7 Lêi gi¶i: ¸p dông bÊt ®¼ng thøc Bunhac«pski cho 3 cÆp: ( 4a + 3 ;1), ( 4b + 3 ; 1), ( 4c + 3 ; 1) Ta cã 4a + 3 + 4b + 3 + 4c + 3 ≤ 3(4a + 3 + 4b + 3 + 4c + 3) = 3 7 Khai th¸c bµi to¸n: B»ng c¸ch xÐt c¸c cÆp sè nh trªn ta cã thÓ gi¶i c¸c bµi to¸n sau: 1) Cho x 2 + y2 + z2 = 1, chøng minh r»ng x + 2y + 3z ≤ 14 2) Cho a,b,c ≥ 0 , chøng minh r»ng: a + b + c = 1⇒ a + b + b + c + c + a ≤ 6 Sinh viªn: NguyÔn Xu©n L ¬ng Líp C§SP To¸n – Tin K48 26 §Ò tµi: Chøng minh bÊt ®¼ng thøc vµ c¸c øng dông 3) Cho a,b,c ≥ 0 , chøng minh r»ng: c(b − c) + c)a − c) ≤ ab 4) Chøng minh r»ng: (a1 + a 2 + ... + a n )2 ≤ n(a 22 + a 22 + ... + a n 2 ) 5) Cho ax + by = C , chøng minh r»ng: 2 2 2 A2x 2 + B2 y2 ≥ 2A2B C2 2 a B +b A Bµi to¸n 6.4. Chøng minh r»ng sin( x )+cos( x ) ≤ Lêi gi¶i: ¸p dông bÊt ®¼ng thøc Bunhac«pski: 2 2 ∀x ∈ R 2 2 2 1.sin(x)+1.cos(x ) ≤ (1 +1 )(sin(x) + cos(x) ) = 2 ⇒ sin( x )+cos( x ) ≤ 2 ∀x ∈ R Bµi to¸n 6.5. Cho a,b,c lµ c¸c sè d¬ng. chøng minh bÊt ®¼ng thøc: Lêi gi¶i: a 2 + b 2 + c2 ≥ a + b + c 2 b+c c+a a +b ¸p dông bÊt ®¼ng thøc Bunhac«pski. 7. Ph¬ng ph¸p ph¶n chøng. - KiÕn thøc : Gi¶ sö ph¶i chøng minh bÊt ®¼ng thøc nµo ®ã ®óng , ta h·y gi¶ sö bÊt ®¼ng thøc ®ã sai , sau ®ã vËn dông c¸c kiÕn thøc ®· biÕt vµ gi¶ thiÕt cña ®Ò bµi ®Ó suy ra ®iÒu v« lý . §iÒu v« lý cã thÓ lµ tr¸i víi gi¶ thiÕt , hoÆc lµ nh÷ng ®iÒu tr¸i nhîc nhau , tõ ®ã suy ra ®¼ng thøc cÇn chøng minh lµ ®óng . - Mét sè h×nh thøc chøng minh bÊt ®¼ng thøc : + Dïng mÖnh ®Ò ®¶o + Phñ ®Þnh råi suy ra ®iÒu tr¸i víi gi¶ thiÕt . + Phñ ®Þnh råi suy ra tr¸i víi ®IÒu ®óng . + Phñ ®Þnh råi suy ra hai ®iÒu tr¸i ngîc nhau . + Phñ ®Þnh råi suy ra kÕt luËn . - C¸c vÝ dô : Bµi to¸n 7.1. Sinh viªn: NguyÔn Xu©n L ¬ng Líp C§SP To¸n – Tin K48 27 §Ò tµi: Chøng minh bÊt ®¼ng thøc vµ c¸c øng dông Cho a + b = 2 , Chøng minh r»ng: 3 a + 3 b ≤ 2 Lêi gi¶i: §Æt 3 a = m , 3 b = n th× a = m3, b = n 3 . Ta cã m3 + n 3 =2 CÇn chøng minh m + n < 2 . Gi¶ sö m + n > 2 th× (m + n)3 > 8 ⇒ m3 + n3 + 3mn(m + n) > 8 ⇒ 2 + 3mn(m + n) > 8 ⇒ mn(m + n) > 2 ⇒ mn(m + n) > m3 + n 3 Chia 2 vÕ cho sè d¬ng m + n (theo gi¶ thiÕt ph¶n chøng) mn > m2 − mn + n 2 ⇒ 0 > (m − n)2 (v« lÝ) VËy ph¶i cã m + n < 2 Bµi to¸n 7.2. Cho 25 sè tù nhiªn a1 ,a 2 ,...,a 25 tho¶ m·n ®iÒu kiÖn 1 + 1 + ... + 1 = 9 . a1 a2 a 25 Chøng minh r»ng trong 25 sè tù nhiªn ®ã, tån t¹i hai sè b»ng nhau. Lêi gi¶i: Chøng minh b»ng ph¶n chøng. Gi¶ sö trong 25 sè tù nhiªn ®¶ cho, kh«ng cã hai sè nµo b»ng nhau. Kh«ng mÊt tÝnh tæng qu¸t, gi¶ sö a1 < a 2 < ... < a 25 Suy ra a1 > 1,a 2 > 2,...,a 25 > 25 1 + 1 + ... + 1 ≤ 1 + 1 + ... + 1 ThÕ th× a1 a2 a 25 1 2 25 (1) Ta l¹i cã 1 + 1 + ... + 1 (2) < 2 25 −1 = 9 1 2 25 1 + 1 + ... + 1 Tõ (1) vµ (2) suy ra < 9, tr¸i víi gi¶ thiÕt. VËy a1 a2 a 25 tån t¹i hai s« b»ng nhau trong 25 sè a1,a 2 ,...,a 25 . Bµi to¸n 7.3. Chøng minh r»ng kh«ng cã 3 sè d¬ng a, b, c nµo tho¶ m·n c¶ 3 bÊt ®¼ng thøc: a+ 1 < 2; b 1 1 b + < 2; c + c a Lêi gi¶i: Gi¶ sö tån t¹i ba sè d¬ng a, b, c tho¶ m·n c¶ 3 bÊt ®¼ng thøc: Sinh viªn: NguyÔn Xu©n L ¬ng Líp C§SP To¸n – Tin K48 28 §Ò tµi: Chøng minh bÊt ®¼ng thøc vµ c¸c øng dông a+ 1 < 2; b 1 1 b + < 2; c + < 2 c a Céng theo tõng vÕ ba bÊt ®¼ng thøc trªn, ta ®îc: 1 1 1 +b+ +c+ 0 nªn ta dÓ dµng chøng minh ®îc 1 1 1 > 2; b + > 2; c + > 2 b c a 1  1  1  Nh vËy  a + ÷+  b + ÷+  c + ÷ ≥ 6 ®iÒu nµy m©u thuÈn a  b  c  víi (1). VËy kh«ng tån t¹i c¸c sè d¬ng a, b, c tho¶ m·n c¶ 3 bÊt ®¼ng a+ thøc ®· cho. Khai th¸c bµi to¸n: T¬ng tù nh bµi to¸n trªn ta cã thÓ chøng minh ®îc bÊt ®¼ng thøc sau Chøng minh r»ng kh«ng cã 3 sè d¬ng a, b, c tho¶ m·n c¶ 3 bÊt ®¼ng Thøc sau: 4a(1 − b) > 1; 4b(1 − c) > 1 ; 4c(1 − a) > 1 Bµi 7.4 Cho a 4 + b 4 < a 3 + b 3 Chøng minh r»ng: a + b < 2 Lêi gi¶i Ph¬ng ph¸p ph¶n chøng: Gi¶ sö a + b ≥ 2 .§Æt a = 1 + x víi x + y ≥ 0  b = 1 + y  XÐt hiÖu: a 4 + b4 − a3 − b3 = ( 1 + x ) + ( 1 + y ) − ( 1 + x ) − ( 1 + y ) 4 4 3 3 = ( x + y ) + 3 ( x2 + y 2 ) + 3 ( x3 + y3 ) = ( x + y ) + 3 ( x 2 + y 2 ) + 3 ( x + y ) ( x 2 − xy + y 2 ) ≥ 0 ∀x; y Sinh viªn: NguyÔn Xu©n L ¬ng Líp C§SP To¸n – Tin K48 29 §Ò tµi: Chøng minh bÊt ®¼ng thøc vµ c¸c øng dông hay a 4 + b 4 − a 3 − b 3 ≥ 0 → víi a + b ≥ 2 Th×: a 4 + b 4 ≥ a 3 + b 3 Tr¸i víi gi¶ thiÕt . VËy a + b < 2 : a 3 + b3 < 2 Chøng minh r»ng: a + b < 2 Lêi gi¶i : Ph¬ng ph¸p ph¶n chøng. Bµi 7.5. cho Gi¶ sö a + b ≥ 2 a = 1 + x b = 1 + y ta ®Æt  víi x+y≥0 Ta cã: a 3 + b3 = ( 1 + x ) + ( 1 + y ) = 2 + 3 ( x + y ) + 3 ( x 2 + y 2 ) + x3 + y 3 3 3 ( ) ( = 2 + 3 ( x + y ) + 3 x 2 + y 2 + ( x + y ) x 2 − xy + y 2 ≥ 2 ) V× x + y ≥ 0 Suy ra a 3 + b 3 ≥ 2 Tr¸i gi¶ thiÕt.VËy a + b < 2 8. Phong ph¸p h×nh häc. Nãi chung ta sö dông c¸c bÊt ®¼ng thøc trong tam gi¸c: *> Tæng hai c¹nh trong tam gi¸c bao giê cñng lín h¬n c¹nh cßn l¹i. *> HiÖu hai c¹nh trong tam gi¸c lu«n bÐ h¬n c¹nh cßn l¹i. *> ∆ABC A+B+C = π *> Lu ý ®Õn tÝnh chÊt c¸c cung liªn kÐt: §èi, phô, bï, kh¸c π Bµi to¸n 8.1. Chøng minh bÊt ®¼ng thøc sau víi a,b,c,d > 0 (a 2 + c2 )(b2 + c2 ) + (a 2 + d 2 )(b2 + d 2 ) ≥ (a + b)(c + d) Lêi gi¶i: XÐt tø gi¸c ABCD Cã AC ⊥ BD,O lµ giao ®iÓm cña hai ®êng chÐo; OA = a,OC = b,OB = c,OD = d víi a,b,c,d > 0 . Theo ®Þnh lý Pitago: AB = a 2 + c2 ,BC = b2 + c2 ,AD = a 2 + d 2 ,CD = b2 + d 2 AC = a + b,BD = c + d CÇn chøng minh AB.BC + AD.CD ≥ AC.BD ThËt vËy, ta cã AB.AC ≥ 2S ABC AD.CD ≥ 2S ADC Sinh viªn: NguyÔn Xu©n L ¬ng Líp C§SP To¸n – Tin K48 30 §Ò tµi: Chøng minh bÊt ®¼ng thøc vµ c¸c øng dông Suy ra AB.BC + AD.CD ≥ 2SABCD = AC.BD VËy (a 2 + c2 )(b2 + c2 ) + (a 2 + d 2 )(b2 + d 2 ) ≥ (a + b)(c + d) C¸ch gi¶i kh¸c. ¸p dông bÊt ®¼ng thøc Bunhac«pski (m2 + n 2 )(x 2 + y2 ) ≥ (mx + ny)2 víi m = a,n = c,x = c, y = b ta ®îc ®iÒu cÇn chøng minh. Bµi to¸n 8.2. Chøng minh bÊt ®¼ng thøc sau b»ng ph¬ng ph¸p h×nh häc: a 2 + b2 b2 + c2 ≥ b(a + c) víi a,b,c lµ nh÷ng sè d¬ng Lêi gi¶i: §Æt c¸c ®o¹n BH = a,HC = c trªn mét ®êng th¼ng. KÎ ®o¹n HA = b vu«ng gãc víi BC . DÓ thÊy AB.AC ≥ 2SABC = BC.AH Khai th¸c bµi to¸n: T¬ng tù nh trªn ta cã thÓ chøng minh ®¬c bÊt ®¼ng thøc sau Chứng minh bất đẳng thức sau bằng phương ph¸p h×nh học : a + b . b + c ≥ b(a + c) với a, b, c > 0 2 2 2 2 Bµi to¸n 8.3. Cho tam tam gi¸c ABC cã c¸c c¹nh a, b, c vµ c¸c ®êng cao t¬ng øng h a , h b , h c . Gäi r lµ b¸n kÝnh ®êng trßn néi tiÕp tam gi¸c. Chøng minh r»ng r , r , r 0 b+c−a >0 c+a−b>0 Vµ abc > 0 VËy bÊt ®¼ng thøc ®· ®îc chøng minh 9. Ph¬ng ph¸p sö dông tÝnh ®ång biÕn, nghÞch biÕn cña hµm sè Bµi to¸n 9.1. chøng minh r»ng víi mäi sè thùc x > 2 3x + 4x < 5x (1) Ph©n tÝch: 2 2 §Ó ý r»ng 3, 4, 5 lµ bé sè Pitago: 3 + 4 = 5 ⇔  3 ÷ +  4 ÷ = 1 vµ 5 5 x x c¸c hµm sè y =  3 ÷ , y =  4 ÷ nghÞch biÕn, ta cã lêi gi¶i sau: 5 5 2 Lêi gi¶i: a   BÊt ®¼ng thøc (1) ⇔  3 ÷ 5 2 a   + 4 ÷ 5 2 < 1. x x Do c¸c hµm sè mò y =  3 ÷ , y =  4 ÷ nghÞch biÕn (c¬ sè bÐ h¬n 1) Nªn víi x > 2 ta cã: 5 5 Sinh viªn: NguyÔn Xu©n L ¬ng Líp C§SP To¸n – Tin K48 33 §Ò tµi: Chøng minh bÊt ®¼ng thøc vµ c¸c øng dông x 2 x 2 3 3  4 4  ÷<  ÷,  ÷<  ÷. 5 5 5 5 x 3  ÷ 5 VËy x 2     + 4 ÷ <  3÷ 5 5 2   +  4 ÷ =1 ⇔ 5 3x + 4x < 5x . Khai th¸c bµi to¸n: BÊt ®¼ng thøc x a + ya < z a ( a > 2 ) ®óng víi mäi bé sè Pitago ( x, y,z ∈ R ®îc gäi lµ bé sè Pitago nÕu x 2 + y2 < z2 ) 10. Ph¬ng ph¸p lµm tréi, lµm gi¶m. Dïng tÝnh chÊt cña B§T ®Ó ®a mét vÕ cña B§T cÇn chøng minh vÒ d¹ng ®Ó tÝnh tæng h÷u h¹n hoÆc tÝch h÷u h¹n. Bµi to¸n 10.1. Chøng minh bÊt ®¼ng thøc sau víi n ∈ N,n ≥ 2 . Lêi gi¶i: 2 n − 3 < 1 + 1 + ... + 1 < 2 n − 2 n 2 3 1 + 1 + ... + 1 . n 2 3 a) Chøng minh A > 2 n − 3 b»ng c¸ch lµm gi¶m mæi sè h¹ng cña A : 1= 2 2 > = 2( k +1 − k ) víi mäi k k+ k k +1 + k k ∈ N* A= §Æt Do ®ã A > 2  ( n +1 − n ) + ... + ( 4 − 3) + ( 3 − 2)  =   2( n +1 − 2) = 2 n +1 − 2 2 > 2 n +1 − 3 > 2 n − 3. b) Chøng minh A < 2 n − 2 b»ng ph¬ng ph¸p lµm tréi mæi sè h¹ng cña A: 1= 2 2 > = 2( k − k −1) víi mäi k ∈ N* k k+ k k + k −1 Do ®ã A < 2  ( n − n − 1) + ... + ( 3 − 2) + ( 2 − 1)  = 2( n − 1) = 2 n − 2 Sinh viªn: NguyÔn Xu©n L ¬ng Líp C§SP To¸n – Tin K48 34 §Ò tµi: Chøng minh bÊt ®¼ng thøc vµ c¸c øng dông Bµi to¸n 10.2. Chøng minh c¸c B§T sau víi n ∈ N* : 1 + 1 + ... + 1 < 2 − 1 n 12 22 n2 5 b) 1 + 1 + ... + 1 < 12 22 n2 3 a) Lêi gi¶i: a) Víi k > 1 ta cã 1 < 1 = 1 −1 k 2 k(k −1) k −1 k LÇn lît thay k = 2,3,...,n råi céng l¹i ta cã: 1 + 1 + ... + 1 < 1− 1 ⇒ ®pcm 2 2 2 n 1 2 n b) Víi k > 1 ta cã: 1 < 2 1 − 1   ÷ k2  2k −1 2k + 1  LÇn lît thay k = 2,3,...,n råi céng l¹i ta cã: 1 + 1 + ... + 1 < 1+ 2(1 + 1 ) < 1+ 2 = 5 12 22 n2 3 2n +1 3 3 Khai th¸c bµi to¸n: T¬ng tù nh trªn ta chøng minh ®îc bÊt ®¼ng thøc sau: 1 + 1 + 1 + ... + 1 2 2x + x + 1 3 1) 2x 12. 2 víi mäi x Sử dụng phương ph¸p đ¸nh gi¸: §©y là PP tương đối khã trong việc chøng minh BĐT, tuú từng dạng bµi mµ cã cã c¸ch ®¸nh gi¸ kh¸c nhau.Cần chú ý điều kiện đề bài để cã hướng đi phï hợp nhất cho bài to¸n Bµi to¸n 12.1. Cho x, y,z là ba số thay đổi, nhận gi¸ trị thuộc đoạn [0 ; 2]. 2(x + y + z) − (xy + yz + zx) ≤ 4 Chứng minh rằng: Lêi gi¶i: Do giả thiết x, y,z là ba số thay đổi, nhận gi¸ trị thuộc đoạn [0 ; 2]. ⇒ (2 − x)(2 − y)2 − z) ≥ 0 ⇒ 8 − 4xyz + 2(xy + yz + xz) − xyz ≥ 0 1 ⇒ 4 ≥ 2(x + y + z) − (xy + xz + yz) + xyz 2 ≥ 2(x + y + z) − (xy + xz + yz) Suy ra §PCM. Đẳng thức xảy ra chẳng hạn khi X = 2, Y = Z = 0 Bµi to¸n 12.2. Chứng minh rằng với mọi số nguyªn n ≥ 3 ta đều cã : n n +1 > ( n + 1) n Lêi gi¶i: Ta cã 34 = 81, 43 = 64 ⇒ 34 > 43 ⇒ BÊt ®¼ng thøc cÇn chøng minh ®óng víi n = 3 n n n +1  1 Víi n > 3 , §PCM ⇔ n >  ⇒ ÷ 1 + ÷ < n  n   n (1) Ta l¹i cã Sinh viªn: NguyÔn Xu©n L ¬ng Líp C§SP To¸n – Tin K48 36 §Ò tµi: Chøng minh bÊt ®¼ng thøc vµ c¸c øng dông n 1 n n(n − 1) 1 n(n − 1)...(n − n + 1) 1  1 k . 2 + ... + . 2 1 + ÷ = ∑ C n . k 1 + + n n 2! n n! n  n 1  1 1  1  2  = 1 + 1 + 1 − ÷+ ... + 1 − ÷1 − ÷ 2!  n  n!  n  n  1 1 1 1 < 1 + 1 + + ... + < 1 + 1 + + ... + n −1 2! n! 2 2 1 1 1 < 1 + 1 + + ... + n −1 + ... = 1 + =3 1 2 2 1− 2 n  1 ⇒ 1 + ÷ < 3 < n ⇒ (1)  n Bµi to¸n 12.3. Cho Chứng minh r»ng Lêi gi¶i: x; y;z ∈ [ 0;1] . 2(x 3 + y3 + z 3 ) − (x 2 y + y 2 z + z 2 x) ≤ 3 x; y;z ∈ [ 0;1] ⇒ 1 − x 2 ;1 − y 2 ;1 − z 2 ;1 − x;1 − y;1 − z ≥ 0 x 3 > x 2 ; y3 > y 2 ;z 3 > z 2 ⇒ 1- y - x 2 − x 2 y + 1 − z − y 2 + zy 2 + 1 − x − z 2 + z 2 x ≥ 0 ⇒ 3 − (x 2 + y 2 + z 2 ) − (x + y + z) + x 2 y + y 2z + z 2 x ≥ 0 ⇒ 3 ≥ x 2 + y2 + z 2 + x + y + z − ( x 2 y + y2z + z 2x ) Dấu “ ” xảy ra khi vµ chØ khi hoặc 2 trong 3 số x; y;z bằng 1, số cßn lại bằng 0 BiÕn ®æi riªng tng vÕ råi so s¸nh kÕt qu¶ suy ra ®iÒu chøng minh Bµi to¸n 12.4. Chøng minh r»ng Lêi gi¶i: Ta cã 200300 > 300200 Sinh viªn: NguyÔn Xu©n L ¬ng Líp C§SP To¸n – Tin K48 37 §Ò tµi: Chøng minh bÊt ®¼ng thøc vµ c¸c øng dông 200300 = (2003 )100 = 8000000100 300200 = (3002 )100 = 90000100 ⇒ 200300 > 300200 (®pcm) Bài to¸n 12.5. CMR: 2 x1 ×x 2 + x 2 ×x 3 + ×××+ x 2002 ×x 2003 + x 2003 ×x1 ≤ x12 + x 22 + ×××+ x 2003 Lêi gi¶i: XÐt bÊt ®¼ng thøc: 2 2 2 2 0 ≤ ( x1 − x 2 ) + ( x 2 − x 3 ) + ×××+ ( x 2002 − x 2003 ) + ( x 2003 + x1 ) 2 2 0 ≤ ( x12 − 2x1x 2 + x 22 ) + ( x 22 − 2x 2 x 3 + x 32 ) + ×××+ ( x 2002 − 2x 2002 x 2003 + x 2003 )+ + ( x 22003 − 2x 2003 x1 + x12 ) 2 2x1x 2 + 2x 2 x 3 + ×××+ x 2003 x1 ≤ 2x12 + 2x 22 + ×××+ 2x 2003 Tõ ®ã suy ra ®pcm 13. Ph¬ng ph¸p dïng tÝnh chÊt tØ sè Cho 3 sè d¬ng a, b, c : a < (a + c) b (b + c) a > (a + c) b (b + c) a c a (a + c) < c NÕu b,d > 0 th× tõ < ⇒ < b d b (b + d) d a < 1 th× b a NÕu > 1 th× b NÕu Bµi to¸n 13..1. Cho 3 sè d¬ng a, b, c . Chøng minh r¼ng Lêi gi¶i: Do c > 0 T¬ng tù ta cã Vµ 1< a + b + c < 2 a +b c+b a +c a (1) < a < a +c a +b+c a +b a +b+c ⇒ a < b < a + b (2) a +b+c c+b a +b+c ⇒ ⇒ c < c < b+c a +b+c a +c a +b+c (3) Céng vÕ theo vÕ cña 3 bÊt ®¼ng thøc kÐp trªn ta ®îc: Sinh viªn: NguyÔn Xu©n L ¬ng Líp C§SP To¸n – Tin K48 38 §Ò tµi: Chøng minh bÊt ®¼ng thøc vµ c¸c øng dông Bµi to¸n 13.2. 1< a + b + c < 2 a +b c+b a +c (®pcm) Chøng minh r»ng : 1 1 1 1 + + ≤ a 2 + 2b 2 + 3 b 2 + 2c 2 + 3 c 2 + 2a 2 + 3 2 trong dã a, b, c lµ c¸c sè thùc d ong tho¶ m·n: abc = 1 Lêi gi¶i a 2 + b 2 ≥ 2ab; b 2 + 1 ≥ 2b ⇒ a 2 + 2b 2 + 3 ≥ 2 ( ab + b + 1) 1 1 1 ⇒ 2 ≤ . 2 a + 2b + 3 2 ab + b + 1 T- ¬ng tù ta 1 1 1 ≤ . 2 b + 2c + 3 2 bc + c + 1 1 1 1 ≤ . c 2 + 2a 2 + 3 2 ac + a + 1 2 L¹i cã : 1 1 1 1 ab b + + = + 2 + ab + b + 1 bc + c + 1 ac + a + 1 ab + b + 1 ab c + abc + ab abc + ab + b 1 ab b 1 + ab + b + + = =1 ab + b + 1 ab + b + 1 ab + b + 1 ab + b + 1 1 1 1 1 VËy + + ≤ . a 2 + 2b 2 + 3 b 2 + 2c 2 + 3 c 2 + 2a 2 + 3 2 = 14. Ph¬ng ph¸p sö dung c¸c bÊt ®¼ng thøc trÞ tuyÖt ®èi * Mét sè bÊt ®¼ng thøc th«ng dông 2 1> A ≥ 0( A = 0 ⇔ A = 0); A = A 2 2> A ≤ B ⇔ − B ≤ A ≤ B (B ≥ 0) A ≥ B 3> A ≥ B ⇔   A ≤ −B 4> A + B ≤ A + B . DÊu “=” x¶y ra khi vµ chØ khi A, B Cïng Sinh viªn: NguyÔn Xu©n L ¬ng Líp C§SP To¸n – Tin K48 39 §Ò tµi: Chøng minh bÊt ®¼ng thøc vµ c¸c øng dông dÊu 5> A − B ≤ A − B . DÊu “=” x¶y ra khi vµ chØ khi A ≥ B ≥ 0 hoÆc A≤B≤0 6> A > B ⇔ A 2 > B2 * a) Cho c¸c sè thùc a1 ,a 2 ,...,a n thÕ th× hiÓn nhiªn a1 + a 2 + ... + a n ≤ a1 + a 2 + ... + a n b) Cho c¸c sè thùc kh¸c kh«ng bÊt k× a, b thÕ th×: a b + ≥ 2 dÊu “=” x¶y ra khi vµ chØ khi a = ± b b a ThËt vËy: 2 2 2 2 2 a b a  b  − ÷ ≥0⇔  ÷ + ÷ ≥ 2 b a b a a  b ⇔  ÷ + ÷ +2≥ 4 b a  2 a b a b ⇔  + ÷ ≥ 22 ⇔ + ≥ 2 b a b a Bµi to¸n 14.1. Cho tam gi¸c ABC cã ®é dµi c¸c c¹nh lµ a, b, c . BiÕt r»ng a < b < c Chøng minh r»ng: Lêi gi¶i: a b c b c a + + − − − ab = (1 + x 2 ) 2 (1 + y 2 ) 2 1 1 Ta cã dÔ thÊy víi mäi a, b th× : - (a − b) 2 ≤ ab ≤ (a + b) 2 4 4 2 2  2 Mµ : ( a − b ) = 1 −  x 2 + 1  2 2   2 ( a + b ) = 1 − 2   y + 1 1 1 Suy ra : − ≤ ab ≤ . 4 4 Bµi 15.3. Cho a,b,c > 0; a + b + c ≤ 1 . Chøng minh r»ng : 1 1 1 + + ≥9 a 2 + 2bc b 2 + 2ca c 2 + 2ab Lêi gi¶i: §Æt : a 2 + 2bc = x;b 2 + 2ac = y;c 2 + 2ab = z Khi ®ã x + y + z = a 2 + 2bc + b 2 + 2ac + c 2 + 2ab = ( a + b + c) ≤ 1 2 Sinh viªn: NguyÔn Xu©n L ¬ng Líp C§SP To¸n – Tin K48 42 §Ò tµi: Chøng minh bÊt ®¼ng thøc vµ c¸c øng dông Bµi to¸n trë thµnh : Cho x, y,z > 0; x + y + z ≤ 1 Chøng minh r»ng : 1 1 1 + + ≥9 x y z 1 1 1 Ta chøng minh ®îc : ( x + y + z ) ( + + ) ≥ 9 x y z (Theo bÊt ®¼ng thøc C«si ) Mµ : x + y + z ≤ 1 nªn suy ra Bài to¸n 15.4. Chøng minh r»ng Lêi gi¶i: §Æt 1 1 1 + + ≥9 . x y z 10000001 20000001 < 10000002 20000002 x = 10000002, y = 20000002 1− y 1− x vµ y x 1 1 x x y 1− x 1 1 1− y =1− 4R 43 Sinh viªn: NguyÔn Xu©n L ¬ng Líp C§SP To¸n – Tin K48 §Ò tµi: Chøng minh bÊt ®¼ng thøc vµ c¸c øng dông V× ∆ ABC lµ mét tam gi¸c nhän nªn t©m ®êng trßn ngo¹i tiÕp tam gi¸c n»m trong tam gi¸c ABCnÕu G lµ träng t©m tam gi¸c ABC th× t©m 0 n»m ë mét trong ba tam gi¸c tam gi¸c GAB, tam gi¸c GAC ,tam gi¸c GBC . Gi¶ sö t©m O n»m trong tam gi¸c GAB th× OA +OB=2R vµ GA+ GB > 2R mµ GA= 2 3 2 3 2 3 2 3 AA1= ma ,GB= BB1 = mb 2 3 Nªn GA+GB > 2R ⇒ (ma+mb) >2R ⇒ ma+mb >3R Mµ trong tam gi¸c OCC1 cã CC1 >OC ⇒ mc >R Do ®ã ma+ mb+ mc > 3R+R=4R . VËy ma+mb+ mc >4R Bµi 16. 2: Mét ®êng trßn tiÕp xóc víi hai c¹nh cña mét tam gi¸c vu«ng ®Ønh A t¹i hai ®iÓm B vµ C , kÎ mét tiÕp tuyÕn víi ®êng trßn c¾t c¸c c¹nh AB vµ AC t¹i M vµ N , chøng minh r»ng AB + AC 2 AB + AC < MB+NC< 3 Lêi gi¶i: A N C l M 0 B Gäi I lµ tiÕp ®iÓm cña tiÕp tuyÕn MN víi ®êng trßn t©m O tÝnh chÊt tiÕp tuyªn cho ta MB=MI ,NC=NI Tõ ®ã MN=MB+NC nhng tam gi¸c vu«ng AMN th× MN< AM+AN Nªn 2MN < AM+AN +BM+ CN =AB +AC Sinh viªn: NguyÔn Xu©n L ¬ng Líp C§SP To¸n – Tin K48 44 §Ò tµi: Chøng minh bÊt ®¼ng thøc vµ c¸c øng dông ⇒ MN< AB + AC 2 Ngoµi ra trong tam gi¸c vu«ng AMN ta còng cã c¹nh huyÒn MN>AM vµ MN> AN ⇒ 2MN > AM+AN V× MN=BC+CN Nªn 3MN > AM+AN +BM+CN do ®ã 3MN > AB+AC ⇒ MN > AB + AC 3 AB + AC AB + AC VËy < MB+NC< 3 2 17. Ph¬ng ph¸p dïng bÊt ®¼ng thøc tæng qu¸t chøa luü thõa c¸c sè tù nhiªn Bµi to¸n : Cho a>b>0 CMR: a1996 − b1996 > a1995 − b1995 a1996 + b1996 a1995 + b1995 L¬i gi¶i: §Ó chøng minh bÊt ®¼ng thøc trªn , ta chøng minh bÊt ®¼ng thøc trung gian sau nÕu a > b > 0 vµ m, n lµ hai sè tù nhiªn mµ m>n th× a m − b m a n − b n (1) > am + bm a n + bn ThËt vËy ta dïng phÐp biÕn ®æi t¬ng ®¬ng ®Ó chøng minh m m m n n n a + b − 2 b a + b − 2 b (1) ⇔ > am + bm a n + bn m n m n 2 b 2 b 2 b 2 b ⇔ 1> 1− n ⇔− m >− n a m + bm a + bn a + bm a + bn Sinh viªn: NguyÔn Xu©n L ¬ng Líp C§SP To¸n – Tin K48 45 §Ò tµi: Chøng minh bÊt ®¼ng thøc vµ c¸c øng dông bm bm bn bn bm bn ⇔ m < ⇔ m < a bm a n bn a + bm a n + bn + + bm bm bn bn 1 1 ⇔ m < n am an ⇔ m +1 > n +1 a a +1 +1 b b m n b b am an a a ⇔ m > n ⇔ ( ) m > ( ) n (2) b b b b a BÊt ®¼ng thøc (2) lu«n ®óng v× a > b > 0 nªn > 1 vµ m > n vËy b bÊt ®¼ng thøc (1) lu«n ®óng m m n n a − b a − b ¸p dông bÊt ®¼ng thøc trung gian vèi a> b > 0 > n m m n a +b a +b vµ m > n nªn khi m=1996, n=1995 th× bÊt ®¼ng thøc ph¶i chøng 1996 1996 1995 1995 a − b a − b minh lu«n ®óng > 1996 1996 a1995 + b1995 a +b Ngoµi ra cßn cã mét sè ph¬ng ph¸p kh¸c ®Ó chøng minh bÊt ®¼ng thøc nh : Ph¬ng ph¸p Lîng gi¸c ho¸ , sö dông tÝnh chÊt cña bÊt ®¼ng thøc, kÜ thuËt C«si ngîc dÊu... ta ph¶i c¨n cø vµo ®Æc thï cña mçi bµi to¸n mµ sö dông ph¬ng ph¸p cho phï hîp . Trong ph¹m vi nhá cña ®Ò tµi nµy kh«ng hÖ thèng ra nh÷ng ph¬ng ph¸p ®ã . PhÇn III øng dông cña c¸c bÊt ®¼ng thøc ®Ó gi¶i c¸c bµi to¸n kh¸c Chñ ®iÓm 1: øng dông cña bÊt ®¼ng thøc c«si ®Ó t×m gi¸ trÞ lín nhÊt, nhá nhÊt cña biÓu thøc Bµi to¸n 1.1 . T×m gi¸ trÞ nhá nhÊt cña A= Lêi gi¶i: 2 1 + víi 0 < x < 1 1− x x Sinh viªn: NguyÔn Xu©n L ¬ng Líp C§SP To¸n – Tin K48 46 §Ò tµi: Chøng minh bÊt ®¼ng thøc vµ c¸c øng dông §Ó ¸p dông bÊt ®¼ng thøc C«-si, ta xÐt biÓu thøc B= 2x 1 − x + 1− x x ¸p dông bÊt ®¼ng thøc C«-si víi hai sè d¬ng 2x 1 − x . =2 2 1− x x  2x 1 − x =  B = 2 2 ⇔ 1 − x x 0 < x < 1 2x 1− x vµ cã 1− x x B≥2 Gi¶i (1) ta ®îc (1) (2) 2x 2 = (1 − x) 2 ⇔ x 2 = 1 − x Do 0 < x < 1 nªn x 2 = 1 − x ⇔ x = Nh vËy min B = 2 2 ⇔ x = B©y giê ta xÐt hiÖu A − B 2 −1. 1 = 2 −1 2 +1 1   2x 1 − x  2 − 2x 1 − 1 + x  2 A−B= + ÷−  + + ÷= x  1− x x 1− x x  1− x = 2 +1 = 3 Do ®ã min A = 2 2 + 3 khi vµ chØ khi x = 2 − 1 Khai t¸c bµi to¸n: T¬ng tù nh trªn ta cã thÓ t×m ®îc gi¸ trÞ nhá nhÊt cña A= (x + a)(x + b) . x Híng dÈn: ( x + a )( x + b) x + ax + bx +ab  ab  A= = =  x + ÷+ ( a + b) x x x   ¸p dông bÊt ®¼ng thøc C«si. Bµi to¸n 1.2. Cho c¸c sè d¬ng a,b,c tho¶ m¶n a + b + c = 1. T×m gi¸ trÞ lín nhÊt cña 2 Sinh viªn: NguyÔn Xu©n L ¬ng Líp C§SP To¸n – Tin K48 47 §Ò tµi: Chøng minh bÊt ®¼ng thøc vµ c¸c øng dông Ph©n tÝch: 1 1 1 P = (1 + )(1 + )(1 + ). a b c 1 Tõ a, b, c > 0 suy ra 3 abc ≤ ⇔ 3 1 3 abc ≥ 3 . Do ®ã cã thÓ khai triÓn biÓu thøc P råi íc lîng theo bÊt ®¼ng thøc C«si. Lêi gi¶i: C¸ch 1: P = 1+ 1 + 1 + 1 + 1 + 1 + 1 + 1 . a b c ab ac bc abc ¸p dông bÊt ®¼ng thøc C«si cho 3 sè d¬ng ta cã: a + b + c ≥ 3 abc ⇔ 1 ≥ 3 abc ⇔ abc ≤ 1 33 ⇔ 1 ≥ 33 (1) abc MÆt kh¸c: 1 + 1 + 1 ≥ 3 3 ( 1 )2 ≥ 32 ; VËy C¸ch 2. ab ac bc abc 1 + 1 + 1 ≥ 3 3 1 ≥ 3. a b c abc P ≥ 1 + 3 + 32 + 33 = (1 + 3)3 = 64. P = a +1. b +1. c +1 = 1 (a +1)(b +1)(c +1) = a c b abc = 1 (a + a + b + c)(b + a + b + c)(c + a + b + c) abc 34 4 4 4 3 ≥ 4 a b c = 4 = 64 abc Khai th¸c bµi to¸n: Ta cã bµi to¸n tæng qu¸t: Cho 3 sè d¬ng a, b, c cã S = a + b + c . T×m gi¸ trÞ lín nhÊt cña, f (a,b,c) = (1 + 1 )(1+ 1 )(1 + 1). Bµi to¸n 1.3. T×m gi¸ trÞ lín nhÊt cña a b c Sinh viªn: NguyÔn Xu©n L ¬ng Líp C§SP To¸n – Tin K48 48 §Ò tµi: Chøng minh bÊt ®¼ng thøc vµ c¸c øng dông B= Lêi gi¶i: y−2 x −1 + x y BÊt ®¼ng thøc C«si cho phÐp lµm tréi m«t tÝch ab ≤ a+b víi a 2 x − 1, y − 2 lµ c¸c tÝch vµ b kh«ng ©m. Ta xem c¸c biÓu thøc 2(y − 2) . x − 1 = 1(x − 1) vµ y − 2 = 2 Theo bÊt ®¨ng thøc C«-si 1(x − 1) 1 + x − 1 1 x −1 = ≤ = x x 2x 2 y−2 2(y − 2) 2 + y − 2 1 2 = ≤ = = y y 4 2y 2 2 2 max B = x − 1 = 1 x = 2 1 2 2+ 2 + = ⇔ ⇔ 2 4 4 y − 2 = 2 y = 4 Bµ to¸n 1.4 x y z + + T×m GTNN của A = biết x, y, z > 0 , x+y y+z z+x 2 Lêi gi¶i: Ta cã: 2 2 xy + yz + zx = 1 x y z x+y+z + + ≥ . x+y y+z z+x 2 Theo bất đẳng thức Cauchy : x+y y+z z+x ≥ xy ; ≥ yz ; ≥ zx . 2 2 2 xy + yz + zx 1 x+y+z Nªn ≥ = 2 2 2 1 1 VËy min A = ⇔ x=y=z= 3 2 xy yz zx + + Bµ to¸n 1.5. T×m GTNN của A = với z x y 2 2 2 Sinh viªn: NguyÔn Xu©n L ¬ng Líp C§SP To¸n – Tin K48 49 §Ò tµi: Chøng minh bÊt ®¼ng thøc vµ c¸c øng dông x, y,z > 0; Lêi gi¶i: x + y + z =1 xy yz xy yz + ≥2 . = 2y . z x z x yz zx zx xy + ≥ 2z ; + ≥ 2x . Tương tự : x y y z 2A > 2(x + y + z) Suy ra 1 min A = 1 với x = y = z = . 3 Chñ ®iÓm II øng dông cña bÊt ®¼ng thøc bunhac«pski ®Ó t×m gi¸ trÞ lín nhÊt, nhá nhÊt Theo bất đẳng thức Cauchy : Bµi to¸n 2.1. T×m gi¸ trÞ nhá nhÊt cña A= 2 1 + víi 0 < x < 1 1− x x Lêi gi¶i: ¸p dông bÊt ®¼ng thøc Bunhac«pski cho 2 cÆp sè (a 2 + b 2 )(m 2 + n 2 ) ≥ (am + bn) 2 , ta cã 2 2    1   2  ÷ + ÷ .  1 − x   x      ( (1 − x) 1  2 ⇒ + ÷( 1 − x + x ) ≥ 1− x x  ( ) +( x) 2 2+ 1 ) 2 2   2 1 ≥ . 1 − x + . x  ÷  x  1− x  2 ⇒ A.1 ≥ 3 + 2 2 X¶y ra ®¼ng thøc khi vµ chØ khi: ⇔ x( 2 + 1) = 1 ⇔ x = 2 − 1 VËy min A = 3 + 2 2 khi vµ chØ khi x = 2 − 1 Bµi to¸n 2.2. Gi¶ sö x, y ≥ 0 tho¶ m·n : x 2 + y 2 = 1 a ) Chøng minh r»ng 1 ≤ x + y ≤ 2 b ) TÝnh gi¸ trÞ lín nhÊt, nhá nhÊt cña biÓu thøc : P = 1 + 2x + 1 + 2 y Lêi gi¶i: Sinh viªn: NguyÔn Xu©n L ¬ng Líp C§SP To¸n – Tin K48 50 §Ò tµi: Chøng minh bÊt ®¼ng thøc vµ c¸c øng dông 0 ≤ x ≤ 1 ⇒ x 2 ≤ x Ta cã x, y ≥ 0; x + y = 1 ⇒  2 0 ≤ y ≤ 1 ⇒ y ≤ y 2 2 ⇒ x 2 + y 2 ≤ x + y hay x + y ≥ 1 ( x + y) 2 ≤ ( 12 + 12 ) ( x 2 + y 2 ) = 2 ⇒ x + y ≤ 2 ⇒ x + y ≤ 2 1≤ x + y ≤ 2 VËy ( b) 1 + 2x + 1 + 2 y ) 2 ≤ 2( 2 + 2x + 2 y ) ≤ 4 + 4 2 ⇔ 1 + 2x + 1 + 2 y ≤ 2 1 + 2 1 DÊu “=” x¶y ra khi vµ chØ khi x = y = 2 Nªn gi¸ trÞ lín nhÊt cña p lµ 2 1 + 2 khi x = y = Bµi to¸n 2.3. 1 2 6 6 6 a b c Timg gi¸ trÞ nhá nhÊt cña trong ®ã + 3 + 3 3 3 3 3 b +c c +a a +b a,b,c lµ nh÷ng sã thùc d¬ng tho¶ m ·n ®iÒu kiÖn a + b + c = 1 Lêi gi¶i: ¸p dông bÊt ®¼ng thøc Bunhac«pski   Hay ( b3 + c3 ) ( 2 + c3 + a 3 ) ( 2 + ) 2 a 3 + b3  .  2 2 2  a   b   c   3 3 3 2  3 3 ÷ +  3 +  ≥ b + a + c ( ) ÷  ÷ 3 3 3  b + c   c + a   b + a   b6 c6   a6 3 3 3 2 2 ( a + b + c ) . 3 3 + 3 + ≥ a + b + c ( ) 3 3 3 ÷ b +c c +a a +b  3 3 3 b6 c6  ( a + b + c )  a6 ⇔ 3 3 + 3 + 3 ≥ ( I) 3 3 ÷ b + c c + a a + b 2   3 3 3 Sinh viªn: NguyÔn Xu©n L ¬ng Líp C§SP To¸n – Tin K48 51 §Ò tµi: Chøng minh bÊt ®¼ng thøc vµ c¸c øng dông L¹i Cã ( ) + ( b ) + ( c ) ( a ) + ( b ) + ( c )  ( a + b + c ) =  a 3 ≥ ( a 2 + b 2 + c2 ) (II) 3 3 3 ( 1 + 1 + 1) ( a 2 2 3 2 3 2    2 2 + b 2 + c2 ) ≥ ( a + b + c ) = 1 ⇒ ( a 2 + b 2 + c2 ) ≥ 2 2 1 (III) 9 b6 c6  1  a6 Tõ (I), (II), (III) ⇒  3 + 3 + 3 ≥ 3 3 3 ÷ b + c c + a a + b   18 1 Khi a = b = c = 3 6 6 6 b c   a 1 V©y  3 khi + + = 18 a = b = c = 3 3 3 3 3 ÷ 3  b + c c + a a + b  min Chñ ®iÓm III. Sö dung bÊt ®¼ng thøc ®Ó gi¶i ph¬ng tr×nh - KiÕn thøc : Nhê vµo c¸c tÝnh chÊt cña bÊt ®¼ng thøc , c¸c ph¬ng ph¸p chøng minh bÊt ®¼ng thøc , ta biÕn ®æi hai vÕ ( VT , VP ) cña ph¬ng tr×nh sau ®ã suy luËn ®Ó chØ ra nghiÖm cña ph¬ng tr×nh . NÕu VT = VP t¹i mét hoÆc mét sè gi¸ trÞ nµo ®ã cña Èn ( tho¶ m·n TX§) => ph¬ng tr×nh cã nghiÖm . NÕu VT > VP hoÆc VT < VP t¹i mäi gi¸ trÞ cña Èn . => ph¬ng tr×nh v« nghiÖm . - C¸c vÝ dô : Bµi to¸n 3.1. Gi¶i ph¬ng tr×nh x − 3 + x − 5 = x 2 − 8x +18. Ph©n tich: XÐt thÊy x 2 − 8x +18 = (x − 4)2 + 2 ≥ 2 . V× vËy chóng ta cã thÓ Sinh viªn: NguyÔn Xu©n L ¬ng Líp C§SP To¸n – Tin K48 52 §Ò tµi: Chøng minh bÊt ®¼ng thøc vµ c¸c øng dông t×m gi¸ trÞ lín nhÊt cña vÕ tr¸i råi so s¸nh gi¸ trÞ bÐ nhÊt cña vÕ ph¶i. Lêi gi¶i: §iÒu kiÖn: 3 ≤ x ≤ 5 VÕ ph¶i (VP ): x 2 − 8x +18 = (x − 4)2 + 2 ≥ 2 DÊu “=” x¶y ra ⇔ x = 4 §¸nh gi¸ vÕ tr¸i (VT ) theo bÊt ®¼ng thøc Bunhac«pski: ( x − 3.1+ 5 − x.1)2 ≤ (x − 3 + 5 − x)(12 +12 ) = 4 ⇒ x -3 + 5 − x ≤ 2. DÊu “=” x¶y ra ⇔ x − 3 = 5 − x ⇔ x − 3 = 5 − x ⇔ x = 4 1 1 Ta cã VP ≥ 2; VT ≤ 2  VP = 2 ⇔x=4 VËy VP = VT ⇔   VT = 2  KÕt luËn: Ph¬ng tr×nh cã nghiÖm duy nhÊt x = 4 . Khai th¸c bµi to¸n: Ta cã bµi to¸n t¬ng tù nÕu xuÊt ph¸t tõ vÕ ph¶i lµ mét b×nh ph¬ng “thõa”, ch¼ng h¹n gi¶i ph¬ng tr×nh: x − 2 + 4 − x = x 2 − 6x +11. Tæng qu¸t: Gi¶i ph¬ng tr×nh sau víi 0 < a < b : x − a + b − x = x2 − 2 a +b a +b b −a x+ +2 . 2 2 2 Bµi to¸n 3.2. Chøng tá ph¬ng tr×nh v« nghiÖm v× cã mét vÕ lu«n nhá h¬n vÕ kia Gi¶i ph¬ng tr×nh (1) x −1 − 5x −1 = 3x − 2 Lêi gi¶i: §iÒu kiÖn x¸c ®Þnh cña (1) lµ x ≥ 1 . Víi ®iÒu kiÖn nµy th× x < 5x , do ®ã x −1 < 5x −1 suy ra vÕ tr¸i cña (1) lµ sè ©m, cßn vÕ ph¶i kh«ng ©m. Ph¬ng tr×nh v« nghiÖm. Bµi to¸n 3.3. Sö dông tÝnh ®èi nghÞch ë 2 vÕ Gi¶i ph¬ng tr×nh: 3x 2 + 6x + 7 + 5x 2 +10x +14 = 4 − 2x − x 2 (1) Lêi gi¶i: Sinh viªn: NguyÔn Xu©n L ¬ng Líp C§SP To¸n – Tin K48 53 §Ò tµi: Chøng minh bÊt ®¼ng thøc vµ c¸c øng dông 3( x +1)2 + 4 + 5( x +1)2 + 9 ≥ 4 + 9 = 5 VÕ ph¶i 4 − 2x − x 2 = 5 − (x +1)2 ≤ 5 VËy hai vÕ cña (1) ®Òu bµng 5, khi ®ã x = −1 . KÕt luËn: x = −1 VÕ tr¸i Khai th¸c bµi to¸n. T¬ng tù nh bµi to¸n trªn chóng ta cã thÓ chøng minh ®îc c¸c bÊt ®¼ng thøc sau 1) x − 7 + x − 9 = x 2 −16x + 66 2) 2x −1 + x − 2 = x +1 3) x −1 + x + 3 + 2 (x −1)(x 2 − 3x + 5) = 4 − 2x Bµi to¸n 3.4. Sö dông tÝnh ®¬n ®iÖu cña hµm sè gi¶i ph¬ng tr×nh 3 (1) 2x +1 + 3 x = 1 Lêi gi¶i: Ta thÊy x = 0 lµ nghiÖm ®óng cña ph¬ng tr×nh (1) Víi x > 0 th× 3 2x +1 > 1 , 3 x > 0 nªn vÕ tr¸i cña (1) lín h¬n 1 Víi x < 0 th× 3 2x +1 < 1 , 3 x < 0 nªn vÕ tr¸i cña (1) nhá h¬n 1 VËy x = 0 lµ nghiÖm ®óng cña ph¬ng tr×nh (1) Bµi to¸n 3.5. Gi¶i ph¬ng tr×nh : 13 x − 1 + 9 x + 1 = 16x Lêi gi¶i: §iÒu kiÖn : x ≥ 1 (*) C¸ch 1 : ¸p dông bÊt ®¼ng thøc C«si ta cã : 13 x − 1 + 9 x + 1 = 13.2. 1 3 x − 1 + 3.2. x +1 2 2 ≤ 13( x - 1 + DÊu '' = '' x¶y ra 1 9 ) + 3(x + 1 + ) = 16x 4 4 1  x − 1 =  2 ⇔ x = 5 tho¶ m·n (*) ⇔+  4  x +1 = 3  2 Ph¬ng tr×nh (1) cã nghiÖm ⇔ dÊu '' = '' ë (2) x¶y ra Sinh viªn: NguyÔn Xu©n L ¬ng Líp C§SP To¸n – Tin K48 54 §Ò tµi: Chøng minh bÊt ®¼ng thøc vµ c¸c øng dông VËy (1) cã nghiÖm x = Chñ ®iÓm IV. 5 . 4 Dïng bÊt ®¼ng thøc ®Ó gi¶i hÖ ph¬ng tr×nh - KiÕn thøc : Dïng bÊt ®¼ng thøc ®Ó biÕn ®æi tõng ph¬ng tr×nh cña hÖ , suy luËn vµ kÕt luËn nghiÖm . Lu ý : Mét sè tÝnh chÊt : a2 + b2 ≥ 2ab a + c < ; c > 0 => a < b a > 1 nÕu a > b > 0 . b - C¸c vÝ dô : Boµi to¸n 4.1. Gi¶i hÖ ph¬ng tr×nh : (1)  x3 + 2 y 2 − 4 y + 3 = 0  2 2 2  x + x y − 2y = 0 ⇔ x3 = - 1 - 2(y - 1)2 ⇔ x3 (2) ⇔ x2 ≤ 2y ≤ 1 1+ y2 ≤ -1 ⇔x ( v× 1 + y2 ≥ 2y) ≤ - 1 . (*) ⇔ -1 ≤ x ≤ 1 (**) Tõ (*) vµ (**) => x = -1 . Thay x = -1 vµo (2) ta cã : y = 1 . => HÖ ph¬ng tr×nh cã nghiÖm duy nhÊt : x = -1 ; y = 1 . - KiÕn thøc : BiÕn ®æi mét ph¬ng tr×nh cña hÖ , sau ®ã so s¸nh víi ph¬ng tr×nh cßn l¹i , lu ý dïng c¸c bÊt ®¼ng thøc quen thuéc . Bµi to¸n 4.2 : Gi¶i hÖ ph¬ng tr×nh :  x + y + z =1  4 4 4  x + y + z = xyz Lêi gi¶i: ¸p dông : B§T : A2 + B2 ≥ 2AB dÊu '' = '' x¶y ra khi A = B Ta cã : x4 + y4 ≥ 2x2y2 ; y4 + z4 ≥ 2y2z2 ; z4 + x4 ≥ 2z2x2 . => x4 + y4 + z4 ≥ x2y2 + y2z2 + z2x2 M¾t kh¸c : x2y2 + y2z2 ≥ 2x2yz (*) Sinh viªn: NguyÔn Xu©n L ¬ng Líp C§SP To¸n – Tin K48 55 §Ò tµi: Chøng minh bÊt ®¼ng thøc vµ c¸c øng dông y2z2 + z2x2 ≥ 2xy2z x2y2 + z2x2 ≥ 2xyz2 => 2(x2y2 + y2z2 + z2x2 ) ≥ 2xyz(x + y + z) = 2xyz . => x2y2 + y2z2 + z2x2 ≥ xyz . (**) 4 4 4 Tõ (*) vµ (**) => x + y + z ≥ xyz DÊu '' = '' x¶y ra khi : x = y = z mµ x + y + z = 1 nªn : x=y=z= 1 3 VËy hÖ ph¬ng tr×nh cã nghiÖm : x = y = z = 1 3 C¸ch 2: ¸p dông B§T C«si ; - KiÕn thøc : Dïng ph¬ng ph¸p thÕ Bµi to¸n 4.3. Gi¶i hÖ ph¬ng tr×nh x + y 2 + z 3 = 14   1 1 x y z  1 ( + +  2 x 3 y 6 z )( 2 + 3 + 6 ) = 1  (víi x, y, z > 0) Lêi gi¶i: ¸p dông : NÕu a, b > 0 th× : 3 x a b + ≥2 b a 2 1 + )(3 x + 2 y + z ) = 36 y z x y x z y z ⇔ 6 ( + ) + 3( + ) + 2( + ) = 22 y x z x z y x y MÆt kh¸c : v× x, y, z > nªn 6 ( + ) ≥ 12 y x x z ; 3( + ) ≥ 6 z x z y 2( + ) ≥ 4 y z (2) ⇔ ( + Sinh viªn: NguyÔn Xu©n L ¬ng Líp C§SP To¸n – Tin K48 56 §Ò tµi: Chøng minh bÊt ®¼ng thøc vµ c¸c øng dông x y x z y z ( + ) + 3( + ) + 2( + ) ≥ 22 y x z x z y DÊu '' = '' x¶y ra khi x = y = z , thay vµo (1) ta ®îc : x + x2 + x3 = 14 (x - 2)(x2 + 3x + 7) = 0 x - 2 = 0 x = 2 . VËy hÖ ph¬ng tr×nh cã nghiÖm duy nhÊt : x = y = z = 2 . chñ ®iÓm VI. Dïng bÊt ®¼ng thøc ®Ó gi¶i ph¬ng tr×nh nghiÖm nguyªn Ngoµi ra cßn cã mét sè nh÷ng øng dông kh¸c cña bÊt ®¼ng thøc , ®ßi hái häc sinh ph¶i linh ho¹t vµ s¸ng t¹o trong khi gi¶i , häc sinh ph¶i n¾m ch¾c ®îc c¸c kiÕn thøc vÒ bÊt ®¼ng thøc th× míi vËn dông ®îc . VÝ dô : Dïng bÊt ®¼ng thøc ®Ó gi¶i ph¬ng tr×nh nghiÖm nguyªn . Bµi 1 : T×m nghiÖm nguyªn d¬ng cña ph¬ng tr×nh : 1 1 1 + + =2 x y z Lêi gi¶i: Kh«ng mÊt tÝnh tæng qu¸t , ta gi¶ sö x ≥ y ≥ z , ta cã : 2= 1 1 1 + + x y z ≤ 3 z => 2z ≤ 3 , mµ z nguyªn d¬ng VËy z = 1 . Thay z = 1 vµo ph¬ng tr×nh ta ®îc : 1 1 + =1 x y Theo gi¶ sö , x ≥ y , nªn 1 = 1 1 + x y ≤ 2 y Y nguyªn d¬ng nªn y = 1 hoÆc y = 2 . Víi y = 1 kh«ng thÝch hîp Víi y = 2 ta cã : x = 2 . Sinh viªn: NguyÔn Xu©n L ¬ng Líp C§SP To¸n – Tin K48 57 §Ò tµi: Chøng minh bÊt ®¼ng thøc vµ c¸c øng dông VËy (2 ; 2 ; 1) lµ mét nghiÖm cña ph¬ng tr×nh . Ho¸n vÞ c¸c sè trªn , ta ®îc nghiÖm cña ph¬ng tr×nh lµ : (2 ; 2 ; 1) ; (2 ; 1 ; 2) ; (1 ; 2 ; 2) Chñ ®iÓm VII. ¸p dông c¸c bÊt ®¼ng thøc vµ c¸c mÖnh ®Ò kh¸c ®Ó t×m gi¸ trÞ lín nhÊt, nhá nhÊt *> C¸c ®Þnh lý MÖnh ®Ò 1. NÕu tæng cña c¸c sè thùc d¬ng x1 , x 2 ,..., x n b»ng mét sè thùc d¬ng cho tríc th× tÝch cña chóng lín nhÊt khi x1 = x 2 = ... = x n Chøng minh: Theo bÊt ®¼ng thøc C«si, ta cã: S x1 + x 2 + ... + x n ≥ n n n x1x 2 ...x n n S ⇔ x1x 2 ...x n ≤   n  DÊu “=” x¶y ra khi vµ chØ khi x1 = x 2 = ... = x n . Víi S ®· cho, khi ®ã tÝch cã gi¸ trÞ lín nhÊt. Tæng qu¸t h¬n, ta cã ®Þnh lý sau: §Þnh lÝ 2. NÕu n sè thùc d¬ng x1 , x 2 ,..., x n cã tæng S kh«ng ®æi th× tÝch P = x m1 x m2 ...x mn cã gi¸ trÞ lín nhÊt khi 1 2 n x1 x 2 x = = ... = n m1 m 2 mn Trong ®ã mi lµ c¸c sè h÷u tØ d¬ng cho tríc Mét c¸ch ®èi ngÈu ta cã: MÖnh ®Ò 3: NÕu tÝch c¸c sè d¬ng x1 , x 2 ,..., x n b»ng mét sè cho tríc th× tæng cña chóng bÐ nhÊt khi x1 = x 2 = ... = x n Chøng minh hoµn toµn nh ®èi víi mÖnh ®Ò 1 Sinh viªn: NguyÔn Xu©n L ¬ng Líp C§SP To¸n – Tin K48 58 §Ò tµi: Chøng minh bÊt ®¼ng thøc vµ c¸c øng dông Mét c¸ch tæng qu¸t, ta cã ®Þnh lý sau §Þnh lý 4. NÕu n sè thùc d¬ng x1 , x 2 ,..., x n cã tÝch trÞ nhá nhÊt khi P = x1m1 x 2 m2 ...x n mn kh«ng ®æi th× tæng S = x1 + x1 + ... + x n x x x cã gi¸ 1 = 2 = ... = n m1 m 2 mn (trog ®ã m1,m2 ,...,m N lµ c¸c sè höu tØ d¬ng) Bµi to¸n 4.1. Trong tÊt c¶ c¸c h×nh chö nhËt cã chu vi cho tríc, h×nh nµo cã diÖn tÝch lín nhÊt ? Lêi Gi¶i: Gi¶ sö x, y lµ ®é dµi hai c¹nh kÒ nhau cña h×nh chö nhËt, 2S lµ chu vi cña nã, P lµ diÖn tÝch. Ta cã x + y = S,xy = P . Víi tæng ®· cho, tÝch P sÎ cã gi¸ trÞ lín nhÊt nÕu x = y = S , tøc lµ nÕu h×nh 2 chö nhËt lµ mé h×nh vu«ng Bµi to¸n 4.2. gi¸ trÞ nhá nhÊt cña hµm sè sau ®¹t ®îc khi nµo ? Lêi gi¶i: y = (a + x)(b + x) , a, b, x ∈ R + . x Ta cã y = ab + x + a + b bÐ nhÊt khi ab + x bÐ nhÊt. x x TÝch ab .x = ab kh«ng ®æi, nªn y cã gi¸ trÞ bÐ nhÊt khi x ab = x tøc lµ ab = x . x Bµi to¸n 5.3. Tõ mét h×nh vu«ng c¹nh a , h·y c¾t ë bèn gãc nh÷ng h×nh vu«ng c¹nh x sao cho khi gÊp l¹i th× ®îc mét c¸i hép cã thÓ tÝch lín nhÊt (kh«ng cã mÐp) Lêi gi¶i: DiÖn tÝch mÆt ®¸y cña h×nh hép b»ng (a − 2x)2 , ®é cao b»ng x , vËy thÓ tÝch c¸i hép lµ: V = x(a − 2x) 2 . §Æt a − 2x = y th× thu ®îc 2V = (2x)y 2 . Ta cã 2x + y = a kh«ng ®æi. VËy 2V b 2 − 4ac (do ®ã, V ) sÎ cã gi¸ trÞ lín nhÊt khi Sinh viªn: NguyÔn Xu©n L ¬ng Líp C§SP To¸n – Tin K48 59 §Ò tµi: Chøng minh bÊt ®¼ng thøc vµ c¸c øng dông Tõ ®ã x = a 6 2x = y tøc lµ 2x = a − 2x 1 2 2 Bµi to¸n 4.3. T×m gi¸ trÞ lín nhÊt, nhá nhÊt cña A = x − 2 + y − 1 trong ®ã x + y = 5 Lêi gi¶i: a) ¸p dông a + b ≤ a + b ®îc A ≤ x + 2 + y +1 = 6 + 2 max A = 6 + 2 khi ch¼ng h¹n x = −2, y = −3 b) ¸p dông chÊt trÞ tÝnh tuyÖt ®èi x − y ≥ x − y ®îc A ≥ x − 2 + y −1 = 4 − 2 min A = 4 − 2 khi ch¼ng h¹n x = 2, y = 3 Khai th¸c bµi to¸n. B»ng c¸ch ¸p dông bµi to¸n trªn ta cã thÓ tÝnh ®îc gi¸ trÞ nhá nhÊt cña a) A = x 2 − 2x + 1 + x 2 − 6x + 9 b) B = x − 2 x − 1 + x + 2 x − 1 c) C = x − 2 + 2x − 3 + 4x − 1 + 5x − 10 Bµi 4.4. T×m gia trÞ nhá nhÊt cña biÓu thøc 2 1  x10 y10  1 16 Q =  2 + 2 ÷+ ( x + y16 ) − ( 1 + x 2 y 2 ) . 2 y x  4 Lêi gi¶i: 1  x10 y10  4 4 12 12  2 + 2 ÷ ≥ x y dÊu b»ng khi x = y 2 y x  1 16 1 x + y16 ) ≥ x 8 y8 dÊu b»ng khi x16 = y16 ( 4 2 2 1 ⇒ Q ≥ x 8 y8 + x 4 y 4 − ( 1 + x 2 y 2 ) 2 2 1 1 = ( x 8 y8 + 2x 4 y 4 + 1) − ( 1 + x 2 y 2 ) − 2 2 Sinh viªn: NguyÔn Xu©n L ¬ng Líp C§SP To¸n – Tin K48 60 §Ò tµi: Chøng minh bÊt ®¼ng thøc vµ c¸c øng dông 2 2 1 4 4 1 x y + 1) − ( x 2 y 2 + 1) − ( 2 2 2 l¹i cã: 12 + 12  x 2 y 2 + 12  ≥ x 2 y 2 + 1   = ( )( ) ( ( ) ( ) dÊu b»ng khi x y 1 + 1) ≥ ( x y + 1) 8 hay 2 x 4 y 4 + 1 ≥ x 2 y 2 + 1 ⇒ ( 1 4 4 x y 2 2 ) 2 2 2 2 2 2 =1 4 2 4 2 2 1 2 2 1 1 2 2 5 5 2 2  ⇒ Q ≥ x y +1 − x y +1 − = x y +1 − 4 − ≥ −  2 8 2 8  2 ( ) ( ) ( ) dÊu b»ng khi x 2 y 2 = 1 5 VËy Q min = − khi x 2 = y 2 = 1 2 phÇn iv Bµi t©p tæng hîp I> Chøng minh bÊt ®¼ng thøc 1> Chøng minh c¸c bÊt ®¼ng thøc: a) x 2 + x 2 +1 > 0 ; b) x 2 + x 2 +1 > 0 ; 2> Chøng minh c¸c bÊt ®¼ng thøc: a) (x + y + z)2 ≤ 3(x 2 + y2 + z2 ) ; b) a 3 + b3 + abc ≥ ab(a + b + c) víi a, b, c > 0 ; 3> Cho a = 2004 − 2003,b = 2005 − 2004 So s¸nh a vµ b , sè nµo lín h¬n. 1 1 1 1 + + + ... + 1.1999 2.1998 3.1997 1999.1 Chøng minh r»ng A ≥ 1,999 5> Chøng minh r»ng víi mäi sè nguyªn d¬ng n : a) A ≥ 1,999 1 + 1 + 1 + ... + 1 Chøng min r»ng víi mäi sè tù nhiªn n ≥ 2 ®Òu cã n < 1 + 1 + 1 + 1 + ... + 1 < 2 n n 1 2 3 4 4> Cho A = Sinh viªn: NguyÔn Xu©n L ¬ng Líp C§SP To¸n – Tin K48 61 §Ò tµi: Chøng minh bÊt ®¼ng thøc vµ c¸c øng dông 7> Chøng minh r»ng víi mäi sè nguyªn d¬ng n 1 + 2 + 3 + ... + n ≤ n n +1 2 8> Cho a = 2 − 1 + 3 − 2 + ... + 25 − 24 1+ 2 2+3 24 + 25 Chøng minh r»ng a < 2 5 9> ) Cho A = 1 . 3 . 5 ..... 2n −1 2 4 6 Chøng minh r»ng: 2n (n ∈ N,n ≥ 2) 1 ; 2n +1 1 ; b) A < 3n +1 a) A < 10> Chøng minh c¸c bÊt ®¼ng thøc sau víi c¸c sè d¬ng a, b, c, d : (a + b)(c + d) ≥ ac + bd 11> Chøng minh b¸t ®¼ng thøc sau a 2 + b 2 + c2 + d 2 ≥ a + b + c + d 2 a +b b+c c+d d+a Víi c¸c sè d¬ng a, b, c, d 12> Chøng min r»ng nªu c¸c ®o¹n th¼ng cã ®é dµi a, b, c lËp ®îc thµnh mét tam gi¸c th× c¸c ®o¹n th¼ng cã ®é dµi a , b, c cñng lËp ®îc thµnh mét tam gi¸c. 13> Cho a, b, c lµ ®é dµi 3 c¹nh cña mét tam gi¸c. Chøng minh r»ng 2(a + b + c) ≤ a 2 + b2 + b2 + c2 + a 2 + c2 < 3(a + b + c) 14> 108) Cho c¸c sè d¬ng a, b, c . Chøng minh bÊt ®¼ng thøc a + b + c >2 a +c b+c a+b 15> Cho c¸c sè d¬ng a, b, c, d . Chøng minh a + b + c + d ≥2 b+c c+d d+a a +b 16> Chøng minh bÊt ®¼ng thøc Sinh viªn: NguyÔn Xu©n L ¬ng Líp C§SP To¸n – Tin K48 62 §Ò tµi: Chøng minh bÊt ®¼ng thøc vµ c¸c øng dông x + y + 4 ≥ 3 x + y   ÷ 2 2 y2 x2  y x 17> Chøng minh bÊt ®¼ng thøc a+b ( a − b) 2 víi a,b > 0 − ab < 2 8b a b c d 18> Cho c¸c sè d¬ng a, b, c, d . BiÕt + + + ≤1 1+ a 1+ b 1+ c 1+ d 1 Chøng minh r»ng abcd ≤ 81 19> Chøng minh bÊt ®¼ng thøc x 2 y2 z 2 x y z + + ≥ + + víi c¸c sè d¬ng x, y, z y2 z2 x 2 y z x B»ng c¸ch vËn dông c¸c bÊt ®¼ng thøc C«-si vµ Bunhac«pski. 20> Cho a = 3 3 + 3 3 + 3 3 − 3 3, b = 2 3 3 . Chøng minh r»ng a < b 21> Chøng minh a) b) 2 + 2 + 2 + ... + 2 + 2 < 2 2 − 2 + 2 + ... + 2 + 2 2 − 2 + 2 + ... + 2 + 2 > (vÕ tr¸i cã 100 dÊu c¨n) 1 ( tö cã 100 dÊu c¨n, mÈu cã 4 99 dÊu c¨n) 22> a) Chøng minh r»ng víi mäi sè nguyªn d¬ng n , ta cã n  1 1 + ÷ < 3  n b) Chøng minh r»ng trong c¸c sè cã d¹ng n n ( n lµ sè tù nhiªn, n ≥ 2 ) sè 3 3 cã gi¸ trÞ lín nhÊt. 23> . Cho a, b, c lµ c¸c sè thùc kh«ng ©m vµ a + b + c = 1 . Chøng minh r»ng a) a + 1 + b + 1 + c + 1 < 3,5 ; Sinh viªn: NguyÔn Xu©n L ¬ng Líp C§SP To¸n – Tin K48 63 §Ò tµi: Chøng minh bÊt ®¼ng thøc vµ c¸c øng dông b) a + b + b + c + c + a ≤ 6 24> ) Cho a, b, c lµ 3 c¹nh cña mét tam gi¸c cßn x, y, z lµ 3 sè tho¶ m¶n ®iÒu kiÖn ax + by + cz = 0 . Chøng minh r»ng xy + yz + xz ≤ 0 (1) 25> . Cho a 3 > 36 vµ abc = 1. Chøng minh r»ng a2 + b 2 + c 2 > ab + bc + ca 3 26> Cho (a + c)(a + b + c) < 0 . Chøng minh r»ng (b − c) 2 > 4a(a + b + c) 27> Cho c¸c sè a,b,c,d,m,n tho¶ m·n: m 2 + n 2 − a 2 − b 2 − c 2 − d 2 > 0 . Chøng minh r»ng (m 2 + a 2 − b 2 )(n 2 − c 2 − d 2 ) ≤ (pq − ac − bd) 2 28> Chøng minh r»ng víi mäi a,b,c ta ®Òu cã: 19a 2 + 54b 2 + 16c 2 + 36ab − 24bc − 16ca ≥ 0 29> Chøng minh r»ng víi mäi a, b, c ta ®Òu cã: a) a + b + c ≥ 3 (a,b,c > 0) b c a b)a 2 + b2 + c2 ≥ ab + bc + ca 30> Chøng minh c¸c bÊt ®¼ng thøc a) a + b ≤ 1 + ab trong ®ã a ≤ 1, b ≤ 1 b) Víi a > b > 1 th× 1 1 2 + ≥ 1 + a 2 1 + b 2 1 + ab 31> Chøng minh c¸c bÊt ®¼ng thøc (a + b)(b + d) ≥ ab + cd ( a, b, c, d > 0) 32> Chøng minh c¸c bÊt ®¼ng thøc m m n n a − b a − b a) víi a > b > 0, m, n∈ N > n m m n a +b a +b n b) (1 + x) + (1 − x) n < 2 n víi x < 1 vµ n ∈ N, n >1 33> Chøng minh c¸c bÊt ®¼ng thøc sau víi mäi a, b, c Sinh viªn: NguyÔn Xu©n L ¬ng Líp C§SP To¸n – Tin K48 64 §Ò tµi: Chøng minh bÊt ®¼ng thøc vµ c¸c øng dông a) a 2 + b 2 + 1 ≥ ab + a + b b) a 2 + b 2 + 4 ≥ ab + 2(a + b) 2 a c) + b + c2 ≥ ab − ac + 2bc 4 34> Chøng minh r»ng víi mäi x. y ta cã x 2 + 5y 2 − 4xy + 2x − 6y + 3 > 0 35> NÕu a + b = 1 Chøng minh r»ng a 4 + b 4 ≥ 1 8 36> Chøng minh r»ng: x2 + 2 ≥ 2 ∀x ∈ R a) x2 + 1 x+8 b) ≥ 6 , ∀x > 1 x −1 c) (a + b)(ab + 1) ≥ 4ab, ∀a, b > 0 37> Chøng minh r»ng a) (a + b)(b + c)(c + a) ≥ 8abc b) a 2 (1 + b 2 ) + b 2 (1 + c 2 ) + c2 (1 + a 2 ) ≥ 6abc 38> . a) Cho x 2 + y 2 = 1 Chøng minh r»ng x + 2y ≤ 5 b) Cho 2x2 + 3y 2 = 5 Chøng minh r»ng 2x + 3y ≤ 5 39> Chøng minh r»ng 1 1 25 2 ) + cos x + ) ≥ 5 sin 2 x cos2 x 2 2 2 2 40> Cho a + b + c + d = 1 Chøng minh r»ng (x 2 + ax + b)2 + (x 2 + cx + d 2 ) ≤ (2x 2 + 1)2 ∀x ∈ R (sin 2 x + 41> Chøng minh r»ng 1 1 1 1 1 + + + ... + > n +1 n + 2 n + 3 2n 2 1 1 1 n −1 b) + + ... + < n 2 2 32 n2 42> Cho n sè d¬ng a1 ,a 2 ,a 3 ,...a n . Chøng minh r»ng a) Sinh viªn: NguyÔn Xu©n L ¬ng Líp C§SP To¸n – Tin K48 65 §Ò tµi: Chøng minh bÊt ®¼ng thøc vµ c¸c øng dông 1 1 1 1 + + ... + a1 a 2 an ≤ a a1a 2 ...a n ( VÕ tr¸i gäi lµ trung b×nh ®iÒu hoµ cña a1 ,a 2 ,a 3 ,...a n ) 1 1 1 + + =4 43> Cho x, y, z lµ c¸c sè d¬ng tho¶ m·n: x y z CMR: 1 1 1 + + ≤ 1 2x + y + z x + 2y + z x + y + 2z 44> CMR: nÕu x − y ≤ 10 th× x - 2y ≤ 200. 45> TÝch cña 2 sè d¬ng lín h¬n tæng cña chóng. CMR: tæng cña chóng lín h¬n 1 4 46> / C¸c sè d¬ng x, y tho¶ m·n: x 2 + y3 ≥ x 3 + y 4 CMR: x 3 + y3 ≤ 2 . 47> CMR: a 4 + b 4 + c 4 + abc ( a + b + c ) ≥ 2 ( ab + bc + ca ) 3 48> T×m tØ sè lín nhÊt cña sè cã 3 ch÷ sè víi tæng c¸c ch÷ sè cña nã. 49> Gi¶ sö a, b, c lµ c¸c sè d¬ng, biÕt abc = 1. CMR: a + b + c ≥ 3. 1 1 1 1 1 1 37 − + − + ×××+ − > 2 4 6 8 98 100 120 51> Gi¶ sö a, b, c tho¶ m·n bÊt ®¼ng thøc: a + b + c > 0  ab + ac + bc > 0 abc > 0  CMR: a > 0; b > 0; c > 0. 50> CMR: 52> Cã thÓ cã c¸c c¹nh x, y, z cña 1 tam gi¸c, tháa m·n bÊt ®¼ng thøc: x 3 + y3 + z3 + 2xyz ≥ x 2 ( y + z ) + y 2 ( z + x ) + z 2 ( x + y ) 53> CMR: ( a + b ) ×( b + c ) ×( c + a ) abc >5 Víi a, b, c lµ ®é dµi c¸c c¹nh tam gi¸c. Sinh viªn: NguyÔn Xu©n L ¬ng Líp C§SP To¸n – Tin K48 66 §Ò tµi: Chøng minh bÊt ®¼ng thøc vµ c¸c øng dông 54> CMR: x12 + x 22 + x 32 + x 24 + x 52 ≥ x1 ( x 2 + x 3 + x 4 + x 5 ) 55> Gi¶ sö a, b, c lµ 3 c¹nh cña mét tam gi¸c, chøng minh bÊt ®¼ng thøc: a3+b3+3abc > c3 56> Chøng minh bÊt ®¼ng thøc: ab + bc + ca < c víi a ,b lµ c¹nh gãc 2 2 vu«ng cña tam gi¸c ABC, cßn c lµ c¹nh huyÒn. 57> CMR: Víi c¸c sè d¬ng a, b, c kh«ng vît qu¸ 1 ta cã bÊt ®¼ng thøc: 58> a b c + + ≤2 bc + 1 ac + 1 ab + 1 Cho c¸c sè x, y, z > 0 vµ x + y + z = 1. Chøng minh r»ng : x + 2y + z ≥ 4(1 - x )(1 − y )(1 − z ) 59> Cho a + b + c + d = 2. Chøng minh r»ng a 2 + b2 + c2 + d 2 ≥ 1 60> Cho hai sè d¬ng a,b tho¶ m¶n a + b = 1. Chøng minh r»ng 2 2 1  1  25  a + + b +  ÷  ÷ ≥ a  b 2  60> Cho ba sè d¬ng a,b,c tho¶ m·n a + b + c = 1 . Chøng minh r»ng a + b ≥ 16abc. 61> Cho 3 sè d¬ng a,b,c tho¶ m¶n a + b + c = 2  2 2 2 a + b + c = 2 4 3 62> Chøng minh r»ng víi mäi a, b, c ∈ [ 0;1] ta lu«n cã Chøng minh r»ng 0 ≤ a, b,c ≤ ( 1 + a + b + c) 2 ≥ 4 ( a 2 + b 2 + c2 ) Sinh viªn: NguyÔn Xu©n L ¬ng Líp C§SP To¸n – Tin K48 67 §Ò tµi: Chøng minh bÊt ®¼ng thøc vµ c¸c øng dông 63> Cho a,b,c lµ 3 sè d¬ng. Chøng minh r»ng 64> 65> 66> 67> b c a+b+c + ≥ 3 c a abc Cho c¸c sè d¬ng a,b,c tho¶ m·n abc = 1. Chøng minh r»ng a3 b3 c3 3 + + ≥ ( 1 + b) ( 1 + c) ( 1 + a ) ( 1 + c) ( 1 + a ) ( 1 + b) 4 Cho a,b tho¶ m·n a + b = 1. Chøng minh r»ng 1 2 ab.( a + b ) ≤ 64 Cho x ∈ R tho¶ m·n x > 1. Chøng minh r»ng x4 +1 ≥2 2 x3 − x Cho 0 < a, b, c, d 1 3b(1 - c) > 2 8c(1 - d) > 1 32d(1 - a) > 3 69> Cho tam gi¸c ABC cã chu vi 2p = a + b + c (a, b , c lµ ®é dµi c¸c c¹nh cña tam gi¸c ) . Chøng minh r»ng : 1 1 1 1 1 1 + + ≥2( + + ) p −a p −b p −c a b c II> T×m gi¸ trÞ lín nhÊt nhá nhÊt 70> T×m gi¸ trÞ nhá nhÊt cña A = x 2 + x + 1 + 71> T×m gi¸ trÞ nhá nhÊt, lín nhÊt cña: a) A = 1 − x + 1 + x x2 − x −1 b) B = x − 2 + 6 − x 72> T×m gi¸ trÞ lín nhÊt cña A = 2x + 5 − x 2 73> T×m gi¸ trÞ nhá nhÊt, lín nhÊt cña: Sinh viªn: NguyÔn Xu©n L ¬ng Líp C§SP To¸n – Tin K48 68 §Ò tµi: Chøng minh bÊt ®¼ng thøc vµ c¸c øng dông 74> T×m gi¸ trÞ nhá nhÊt cña A = x + y biÕt x, y lµ c¸c sè d¬ng tho¶ m·n a b + = 1 (a,b > 0) x y 75> T×m gi¸ trÞ nhá nhÊt cña A = x 4 + y 4 + z 4 biÕt r»ng xy + yz + xz = 1 76> T×m gi¸ trÞ lín nhÊt cña A = x − y biÕt r»ng x 2 + 4y 2 = 1 b c + c+d a+b với b + c > a + d ; b,c > 0; a,d ≥ 0 T×m giá trị nhỏ nhất của A = x 2 + y 2 với x + y = 4 77> T×m GTNN của A = 78> T×m gi¸ trÞ nhá nhÊt cña biÓu thøc P( x) = x2 + x + 2 x ( x + 1) + 1 víi x ∈ R 79> Gi¶ sö x, y,z lµ c¸c sè d¬ng thay dæi vµ tho¶ m·n ®iÒu kiÖn xy 2 z 2 + x 2 z + y = 3z 2 . H·y t×m gi¸ trÞ lín nhÊt cña biÓu thøc z4 P= 1 + z 4 ( x 4 + y4 ) 80> T×m gi¸ trÞ nhá nhÊt cña biÓu thøc (x P= 3 + y3 ) − ( x 2 + y 2 ) ( x − 1) ( y − 1) Trong ®ã x, y lµ nh÷ng sè d¬ng lín h¬n 1 1 2 f ( x ) = 2x 2 + 5x + 2 + 2 x + 3 − 2x. 81> Víi nh÷ng gi¸ trÞ cña x tho¶ m·n: x ≥ − , h·y tÝnh gi¸ trÞ lín nhÊt cña biÓu thøc Sinh viªn: NguyÔn Xu©n L ¬ng Líp C§SP To¸n – Tin K48 69 §Ò tµi: Chøng minh bÊt ®¼ng thøc vµ c¸c øng dông 82> T×m gi¸ trÞ nhá nhÊt cña biÓu thøc T = ( x + y ) ( x + z ) Trong ®ã x, y,z la nhömg sè d¬ng tho¶ m¶n ®iÒu kiÖn ( x + y + z ) xyz = 1. 83> T × m gi¸ trÞ nhá nhÊt cña biÓu thøc : 2 1  x10 y10  1 16 16 Q =  2 + 2 ÷+ x + y - 1+ x 2 y 2 . 2 y x  4 84> Gi¶ sö a,b,c lµ nh÷ng sè thùc d¬ng tho¶ m·n ®iÒu kiÖn ( ) ( ) b 2 + c2 ≤ a 2 T×m gi¸ trÞ nhá nhÊt cña biÓu thøc P= 1 2 2 1 2 1 b + c + a + ( )  2 2 ÷ a2 b c  85> Cho x ∈ R tho¶ m·n x > 1 . Chøng minh r»ng x4 + 1 ≥2 2 x3 − x 86> T×m gi¸ trÞ nhá nhÊt cña biÓu thøc A = x + 1 + x − 2x + 5 2 2 87> T×m gi¸ trÞ nhá nhÊ, lín nhÊt cña biÓu thøc 89> A = x −1 + y − 2 T×m gi¸ trÞ nhá nhÊt cña biÓu thøc A = x − 2 x −1 + x + 2 x −1 90> T×m gi¸ trÞ nhá nhÊt cña biÓu thøc A = − x + 4x + 12 − − x + 2x + 3 2 91> 2 T×m gi¸ trÞ lín nhÊt cña c¸c biÓu thøc biÕt x + y = 4 a) A = x −1 + y − 2 b) B= y−2 x −1 + x y 92> T×m gi¸ trÞ nhá nhÊt cña biÓu thøc A = 3x + 3y víi x + y = 4 93> T×m gi¸ trÞ nhá nhÊt cña biÓu thøc Sinh viªn: NguyÔn Xu©n L ¬ng Líp C§SP To¸n – Tin K48 70 §Ò tµi: Chøng minh bÊt ®¼ng thøc vµ c¸c øng dông A= b c + c+d a+b víi b + c > a + d; b,c >; a,d ≥ 0 iii. gi¶i ph¬ng tr×nh vµ hÖ ph¬ng tr×nh 94> a) T×m gi¸ trÞ lín nhÊt cña L = 2 x − 3 + 5 − 2x b. Gi¶i ph¬ng tr×nh : 95> Gi¶i ph¬ng tr×nh : 6− x + 2 x − 3 + 5 − 2x - x2 + 4x - 6 = 0 (*) 2 x + 2 = x - 6x + 13 96> Gi¶i ph¬ng tr×nh : 3 x 2 − 12 x + 16 + y 2 − 4 y + 13 = 5 PhÇn V Híng dÈn gi¶i bµi tËp tæng hîp I. Chøng minh c¸c bÊt ®¼ng thøc. 1> a) x 2 + x 2 +1 > 0 ; a Híng dÈn: 2 b) BiÕn ®æi  x + 1  + 1 > 0  ÷ 2 2  a + b + a − b < 2 a víi a, b > 0 Híng dÈn: BiÕn ®æi t¬ng ®¬ng 2> Híng dÈn: XÐt hiÖu 3(x 2 + y2 + z 2 ) − (x + y + z)2 3> Híng dÈn: 1 1 = 2004 + 2003, = 2005 + 2004 a b 1 1 So s¸nh vµ a b C¸ch 1: C¸ch 2: Ta cã 2003 + 2005 < 2004 2 Nªn 2003 + 2005 < 2004 + 2004 Do ®ã 2005 − 2004 < 2004 − 2003 Tøc lµ b < a Sinh viªn: NguyÔn Xu©n L ¬ng Líp C§SP To¸n – Tin K48 71 §Ò tµi: Chøng minh bÊt ®¼ng thøc vµ c¸c øng dông 4> Híng dÈn: Dïng bÊt ®¼ng thøc 5> 1 2 chóng minh mæi sè h¹ng cña > ab a + b A > 0,001 a) 1 1 1 1 + + + ... + > ... > < n 1 2 n n §Æt a = Sinh viªn: NguyÔn Xu©n L ¬ng Líp C§SP To¸n – Tin K48 72 §Ò tµi: Chøng minh bÊt ®¼ng thøc vµ c¸c øng dông 1 2 2 = < = 2( n − n − 1) n n+ n n + n −1 Nªn a < 2 n MÆt kh¸c 7> Híng dÈn: Dïng ph¬ng ph¸p quy n¹p to¸n häc Víi n = k + 1 . Sk +1 = Sk + k + 1 ≤ k. k +1 + k +1 2 CÇn chøng minh: k. k +1 k+2 + k + 1 ≤ (k + 1). ⇔ 2k 2 ≤ 3k 2 2 C¸ch 2: Víi n = 1 bÊt ®¼ng thøc ®óng Víi n ≥ 2 ¸p dung bÊt ®¼ng thøc Bunhac«pski 8> Híng dÈn: XÐt n +1 − n n +1 − n n +1 − n n +1 − n = = < n + (n + 1) 2n + 1 4n 2 + 4n + 1 4n 2 + 4n 1 1 1  =  − ÷ 2 n n +1  9> Híng dÈn: Ta cã 1 3 2n − 1 A = . ... 2 4 2n 2 4 2n A = . ... 3 5 n +1 1 ⇒ A2 < 2n + 1 Suy ra ®iÒu cÇn chøng minh. 10> Híng dÈn: B×nh ph¬ng hai vÕ råi rót gän, ta ®îc bÊt ®¼ng thøc t¬ng ®¬ng (ad − bc) 2 ≥ 0 11> Híng dÈn: ¸p dông C«-si Sinh viªn: NguyÔn Xu©n L ¬ng Líp C§SP To¸n – Tin K48 73 §Ò tµi: Chøng minh bÊt ®¼ng thøc vµ c¸c øng dông 12> Híng dÈn: gi¶ sö a ≥ b ≥ c > 0 . Theo ®Ò bµi: c + b > a suy ra b + c + 2 bc > a ⇒ b + c > a 13> Híng dÈn: LÇn lît chøng minh a 2 + b 2 + b 2 + c2 + c2 + a 2 ≥ 2(a + b + c) (1) a 2 + b 2 + b 2 + c2 + c2 + a 2 < 3(a + b + c) (2) Chøng minh (1) DÓ thÊy (a + b) 2 ≤ 2(a 2 + b 2 ) nªn a + b ≤ 2(a 2 + b 2 ) c + b ≤ 2(c 2 + b 2 ) T¬ng tù a + c ≤ 2(a 2 + c 2 ) 2(a + b + c) ≤ a 2 + b 2 + b 2 + c 2 + a 2 + c 2 Chøng minh (2): Do a, b, c lµ ®é dµi 3 c¹nh cña mét tam gi¸c nªn (a − b) 2 < c2 ⇒ a 2 + b 2 < c 2 + 2ab Suy ra T¬ng tù ⇒ a 2 + b 2 < c2 + 2ab b 2 + b 2 < a 2 + 2cb a 2 + c 2 < b 2 + 2ac Céng vÕ theo vÕ ba bÊt ®¼ng thøc trªn ta ®îc: 2(a + b + c) ≤ a 2 + b2 + b2 + c2 + a 2 + c2 (3) DÓ dµng chøng minh ®îc (x + y + z) 2 ≤ 3(x 2 + y 2 + z 2 ) nªn (x + y + z) ≤ 3(x 2 + y 2 + z 2 ) ¸p dông bÊt ®¼ng thøc trªn ta chøng minh ®îc: a 2 + b2 + b2 + c2 + a 2 + c2 < 3(a + b + c) (4) Tõ (3) vµ (4) suy ra ®iÒu cÇn chøng minh. 14> Híng dÈn: Theo bÊt ®¼ng thøc C«-si Sinh viªn: NguyÔn Xu©n L ¬ng Líp C§SP To¸n – Tin K48 74 §Ò tµi: Chøng minh bÊt ®¼ng thøc vµ c¸c øng dông (b + c) b+c+a b+c  .1 ≤  + 1÷: 2 = a 2a  a  2a b + c a + b+c 2b b ≥ a + c a + b+c 2c c ≥ b + a a + b+c a Do ®ã T¬ng tù ≥ a + b + c >2 a +c b+c a+b a = b + c  DÊu “=” x¶y ra khi vµ chØ khi b = a + c ⇒ a + b + c = 0 tr¸ivíi c = a + b  Céng tõng vÕ ta ®îc gi¶ thiÕt a, b, c lµ c¸c sè d¬ng. VËy ®¼ng thøc kh«ng x¶y ra. 15> Híng dÈn: ¸p dông bÊt ®¼ng thøc 1 4 ≥ víi x > 0, y > 0 th× xy (x + y) 2 a c a 2 + ad + bc + c 2 4(a 2 + ad + bc + c 2 ) (1) + = ≥ 2 b+c d+a (b + c)(a + d) (a + b + c + d) T¬ng tù b d 4(b 2 + ab + cd + d 2 ) (2) + ≥ 2 d+c b+a (a + b + c + d) Céng (1) vµ (2) ta ®îc 2 2 2 2 a + b + c + d ≥ 4(a + b + c + d + ad + bc + ab + cd) ≥ 4B b+c c+d d+a a +b (a + b + c+ d) 2 1 ⇔ (a − c) 2 + (b − d) 2 ≥ 0 ®óng 2 DÊu “=” x¶y ra khi vµ chØ khi a = c vµ b = d Ta chøng minh B ≥ Sinh viªn: NguyÔn Xu©n L ¬ng Líp C§SP To¸n – Tin K48 75 §Ò tµi: Chøng minh bÊt ®¼ng thøc vµ c¸c øng dông 16> Híng dÈn: §Æt x y + =a y x x2 Chøng minh ®îc y 2 + y2 x ≥ 2 nªn a 2 ≥ 4 do ®ã a ≥ 2 hoÆc 2 a ≤ 2 (1) BÊt ®¼ng thøc ph¶i chøng minh t¬ng ®¬ng víi a 2 − 2 + 4 ≥ 3a ⇔ a ≥ 2 hoÆc a ≤ 1 (2) Tõ (1) vµ (2) suy ra ®iÒu cÇn chøng minh 17> Híng dÈn: a+b a+b−2 − ab = 2 2 Ta cã CÇn chøng minh ( a− b 18> Híng dÈn: Tõ gi¶ thiÕt ta cã: 2 ) 2 2 a − b) ( < ab ( = a− b ) 2 2 suy ra ®iÒu cÇn chøng ninh. 8b b c d a 1 + + ≤ 1− = b +1 c +1 d +1 a +1 a +1 ¸p dông bÊt d¼ng thøc C«-si cho 3 sè d¬ng ta cã 1 b c d bcd ≥ + + ≥ 33 a +1 b +1 c +1 d +1 (b + 1)(c + 1)(d + 1) T¬ng tù 1 acd ≥ 33 b +1 (a + 1)(c + 1)(d + 1) 1 abd ≥ 33 c +1 (a + 1)(b + 1)(d + 1) 1 abc ≥ 33 d +1 (a + 1)(c + 1)(b + 1) Nh©n tõng vÕ 4 bÊt ®¼ng thøc, ta ®îc 1 ≥ 81abcd nªn abcd ≤ 19> Híng dÈn: 1 81 Sinh viªn: NguyÔn Xu©n L ¬ng Líp C§SP To¸n – Tin K48 76 §Ò tµi: Chøng minh bÊt ®¼ng thøc vµ c¸c øng dông A= Gäi x2 y2 + y2 z2 + z2 x2 ¸p dông bÊt ®¼ng thøc Bunhac«pski ®îc 2 x y z 3A ≥  + + ÷ y z x (1) ¸p dông bÊt d¼ng thøc C«-si víi 3 sè x, y, z kh«ng ©m, ta ®îc x y z x y z + + ≥ 33 . . = 3 y z x y z x x y z Nh©n tõng vÕ cña (1) vµ (2) ta ®îc A ≥ + + y z x (2) 20> Híng dÈn: 3 3 3 §Æt x = 3 3 + 3 3 , y = 3 − 3 3 th× x + y = 6 B»ng c¸ch xÐt hiÖu b3 − a 3 suy ra ®îc ®iÒu cÇn chøng minh 21> Híng dÈn: a) KÝ hiÖu Ta cã a n = 2 + 2 + ... + 2 + 2 ( cã n dÊu c¨n) a1 = 2 < 2 a 2 = 2 + a1 < 2 + 2 = 2 a3 = 2 + a 2 < 2 + 2 = 2 ....................................... a100 = a 2 + 99 < 2 + 2 = 2 b) Híng dÈn: Víi kÝ hiÖu nh c©u a th× tö lµ 2 − a100 tö lµ 2 − a 99 = 2 − (a 2100 − 2) = 4 − a 2100 2−a 1 §Æt a100 = a , cÇn chøng minh > 4 − a2 4 22> Híng dÈn: Theo quy n¹p Sinh viªn: NguyÔn Xu©n L ¬ng Líp C§SP To¸n – Tin K48 77 §Ò tµi: Chøng minh bÊt ®¼ng thøc vµ c¸c øng dông Víi n ≥ 2 ta cã : n 1 n(n − 1) 1 n(n − 1)(n − 2) 1 n(n − 1).2.1 1  1 . 2+ . 3 + ... + . n 1 + ÷ = 1 + n + n n 2! 3! n!   n n n 1 1 1 < 1 + 1 +  + + ... + ÷ n!   2! 3! DÓ dµng chøng minh ®îc 1 1 1 1 1 1 + + ... + ≤ + + ... + 2! 3! n! 1.2 2.3 n(n − 1) 1 1 1 1 1 1 = 1 − + − + ... + − = 1− Híng dÈn: a) C¸ch 1: Ta nh×n tæng a + 1 díi d¹ng mét tÝch 1( a + 1 ) vµ ¸p dông bÊt ®¼ng thøc C«-si ta cã: a + 1 = 1(a + 1) ≤ (a + 1) + 1 a = +1 2 2 b +1 2 c c +1 ≤ +1 2 Céng vÕ theo vÕ ta ®îc a + 1 + b + 1 + c + 1 ≤ 3,5 b +1 ≤ T¬ng tù DÊu “=” x¶y ra khi vµ chØ khi a + 1 = b + 1 = c + 1 ⇔ a = b = c = 0 tr¸i víi gi¶ thiÕt a + b + c = 1 suy ra a + 1 + b + 1 + c + 1 < 3,5 C¸ch 2: §Æt a + 1 = x; b + 1 = y; c + 1 = z . ¸p dông bÊt ®¼ng thøc (x + y + z) 2 ≤ 3(x 2 + y 2 + z 2 ) (DÓ chøng minh ®îc) Suy ra ®iÒu cÇn chøng minh. b) C¸ch 1: ¸p dông bÊt ®¼ng thøc Bunhac«pski víi hai bé 3 sè Sinh viªn: NguyÔn Xu©n L ¬ng Líp C§SP To¸n – Tin K48 78 §Ò tµi: Chøng minh bÊt ®¼ng thøc vµ c¸c øng dông (1; a + b); (1; b + c); (1; a + c) suy ra ®iÒu cÇn chøng minh. C¸ch 2: §Æt a + b = x; b + c = y; a + c = z . ¸p dông bÊt ®¼ng thøc (x + y + z) 2 ≤ 3(x 2 + y 2 + z 2 ) suy ra ®iÒu ph¶i chøng minh. 24> Híng dÈn: ax + by c ax + by VËy (1) ⇔ xy + xz + yz = xy − (x + y) ≤ 0 c ⇔ ax 2 + xy(a + b + c) + by 2 ≤ 0 (2) NÕu y = 0 th× (2) ⇔ ax 2 ≥ 0 ®óng ⇒ (1) ®óng Tõ ax + by + cz = 0 ⇒ z = − NÕu y ≠ 0 , khi ®ã 2 x x (2) ⇔ a  ÷ + (a + b + c) + b ≥ 0 (3) y y Ta xem vÕ tr¸i cña (3) lµ tam thøc bËc hai cña x cã hÖ sè cña y 2 x  y ÷ lµ a > 0 vµ   (a + b + c) 2 − 4ab = a 2 + b 2 + c 2 − 2ab − 2ac − 2bc Tõ T¬ng tù b − c < a ⇒ b 2 − 2bc + c 2 < a 2 a 2 − 2ac + c 2 < b 2 a 2 − 2ab + b 2 < c 2 . VËy a 2 + b 2 + c 2 < 2ab + 2bc + 2ac ⇒ a 2 + b 2 + c 2 −2ab − 2bc − 2ac < 0 Nªn vÕ tr¸i cña (3) lu«n lín h¬n 0. Suy ra (1) ®îc chøng minh. DÊu “=” x¶y ra khi x = y = z = 0 25> Híng ®Èn: Sinh viªn: NguyÔn Xu©n L ¬ng Líp C§SP To¸n – Tin K48 79 §Ò tµi: Chøng minh bÊt ®¼ng thøc vµ c¸c øng dông Tõ abc = 1 ⇒ bc = Ta cã: 1 vµ do a 3 > 36 nªn ch¾c ch¾n lµ a > 0 a a2 + (b + c)2 − 2bc > bc + a(b + c) 2 a2 ⇔ (b + c) − a(b + c) − 3bc + > 0 (1) 3 a2 XÐt tam thøc bËc hai f (x) = x 2 − ax − 3bc + ta cã hÖ sè cña 3 2 x lµ 1 > 0 4a 2 36 − a 3 2 Vµ a − + 12bc. < 0 . Theo ®Þnh lý thuËn vÒ dÊu cña 3 3a 2 Tam thøc bËc hai th× f (x) >0 víi mäi x Suy ra a2 f (a + b) = (b + c) − a(b + c) − 3bc + > 0 ⇒ (1) ®óng 3 2 Suy ra ®iÒu cÇn chøng minh. 26> Híng dÈn: NÕu a = 0 th× tõ gi¶ thiÕt ta cã c(b + c) < 0 (1) 2 BÊt ®¼ng thøc cÇn chøng minh cã d¹ng (b − c) > 0 (2) Tõ (1) suy ra b ≠ c . VËy (2) ®óng suy ra (1) ®óng. NÕu a ≠ 0 xÐt tam thøc bËc hai f (x) = ax 2 + x(b − c) + a + b + c Tõ f (0) = a + b + c;f ( −1) = 2(a + c) suy ra tõ gi¶ thiÕt ta cã f (0)f (−1) < 0 Theo ®Þnh lý ®¶o vÒ dÊu cña tam thøc bËc hai suy ra ph¬ng tr×nh f (x) = 0 cã hai Nghiªm ph©n biÖt. Hay (b − c) 2 − 4a(a + b + c) > 0 ⇔ (b − c) 2 < 4a(a + b + c) Suy ra ®iÒu cÇn chøng minh. 27> Tù gi¶i 28> Tù gi¶i 29> Híng dÈn: a) Dïng bÊt ®¼ng thøc C«si cho 3 phÇn tö Sinh viªn: NguyÔn Xu©n L ¬ng Líp C§SP To¸n – Tin K48 80 §Ò tµi: Chøng minh bÊt ®¼ng thøc vµ c¸c øng dông b) TÝnh hiÖu a b c , , b c a a 2 + b 2 + c 2 − (ab + bc + ac) = 1 ( a − b ) 2 + ( a − c ) 2 + ( b − c ) 2  ≥ 0 2 30> Híng dÈn: a) B×nh ph¬ng 2 vÕ b) ChuyÓn vÕ, ®a vÒ v× a ≥ b ≥ 1 nªn ab ≥ 1 ( b − a ) 2 ( ab − 1) ( 1 + ab ) ( 1 + a ) 2 ( 1 + b 2 ) ≥ 0 lµ ®óng 31> Híng dÈn: a) ¸p dông hai lÇn bÊt ®¼ng thøc C«si cho ba phÇn tö a, b, c vµ 1 1 1 , , a b c b) B×nh ph¬ng 2 vÕ ®Ó ®i ®Õn kÕt qu¶ ®óng ab + cd ≥ 2 abcd 32> Híng dÈn: a) V× a m + b m > 0,a n + b n > 0 nªn bÊt ®¼ng thøc cÇn chøng minh t¬ng ®¬ng víi (a n + b b )(a m − b m ) > (a m + b m )(a n − b n ) . Khai triÓn ta ®îc bÊt ®¼ng thøc ®óng. b) §Æt a = 1 + x,b = 1 − x khi ®ã a,b > 0 n n n −1 n −1 b +2 ... 4 + nab Ta cã: 2 = (a + b) = na 1 4 44 4 43 + b n A Do a,b > 0 nªn A > 0 . VËy 2 > a n + b n (§PCM) n 33> Híng dÈn: a) Nh©n hai vÕ víi 2, ta cã bÊt ®¼ng thøc ®óng ( a − b ) 2 + ( a − 1) 2 + ( b − 1) 2 ≥ 0. b) Nh©n hai vÕ víi 2, ta cã bÊt ®¼ng thøc ®óng ( a − b ) 2 + ( a − 2 ) 2 + ( b − 2 ) 2 ≥ 0. Sinh viªn: NguyÔn Xu©n L ¬ng Líp C§SP To¸n – Tin K48 81 §Ò tµi: Chøng minh bÊt ®¼ng thøc vµ c¸c øng dông 2 a c) T¬ng ®¬ng víi  − b + c ÷ ≥ 0 , h»ng ®óng 2  34> Híng dÈn: Dïng tam thøc bËc hai víi Èn x cã ∆ lµ tam thøc bËc hai ®èi víi y , Ta cã ∆ y < 0 , tõ ®ã suy ra ®iÒu ph¶i chøng minh 35> Híng dÈn: ChØ cÇn chøng minh cho trêng hîp a, b cã cïng dÊu d¬ng. Trêng hîp tr¸i l¹i th× bÊt ®¼ng thøc hiÓn nhiªn. Ta cã a 2 + b 2 = ( a + b ) − 2ab = 1 − ab 2 a 4 + b 4 = (a + b) 4 − 2a 2 b 2 = (1 − 2ab) 2 − 2a 2 b 2 (1) 1 V× a + b = 1 vµ a + b ≥ 2 ab nªn 0 ≤ ab ≤ 4 1 Thay ab = vµo (1) ta ®îc: 4 2 1 1 1 4 4 2 2 2  a + b = (1 − 2ab) − 2a b ≥ 1 − 2. ÷− 2.  ÷ = 4 8  4 36> Híng dÈn: a) Dïng bÊt ®¼ng thøc C«si cho hai sè x 2 + 1 vµ 1 b) Dïng bÊt ®¼ng thøc C«si cho hai sè x − 1 > 0 vµ 9 c) Dïng bÊt ®¼ng thøc C«si, ta cã a + b ≥ 2 ab vµ ab + 1 ≥ 2 ab Nh©n 2 vÕ, ta cã §PCM 37> Híng dÈn: a) Ta cã a + b ≥ 2 ab , c + b ≥ 2 cb , a + c ≥ 2 ac Nh©n 2 vÕ víi c¸c bÊt ®¼ng thøc trªn, ta ®îc §PCM b) Khai triÓn vÕ tr¸i, dïng bÊt ®¼ng thøc C«si cho 6 sè vµ ®Ó ý 6 6 a 6 b6c6 ≥ 6abc, ∀a, b, c 38> Híng dÈn: a) Dïng bÊt ®¼ng thøc C«si-Bunhac«pski, ta cã ( x + y ) 2 ≤ ( 12 + 22 ) ( x 2 + y2 = 5 ) (do x 2 + y 2 = 1 ) VËy x + 2 y ≤ 5 b) Dïng bÊt ®¼ng thøc C«si-Bunhac«pski, ta cã Sinh viªn: NguyÔn Xu©n L ¬ng Líp C§SP To¸n – Tin K48 82 §Ò tµi: Chøng minh bÊt ®¼ng thøc vµ c¸c øng dông ( 2x + 3y ) 2 ≤ (  2x ) ( 2 ) ( ) ( ) ( ) 2 2 3y   2 +  = 5 2x 2 + 3y 2 + 2 3   Tõ ®ã suy ra §PCM. 39> Híng dÈn: Dïng bÊt ®¼ng thøc C«si-Bunhac«pski cho c¸c sè 1;1;sin 2 x + ( 1 2 sin x ;cos 2 x + 1 2 cos x ta cã 2 2  2 1   2 1   1 + 1  sin x + 2 ÷ +  cos x + ≥ 2 ÷   sin x   cos x    2 2 ) 2 2 1 1   4   ≥  sin 2 x + 2 + cos 2 x + = 1 + ≥ ÷  2 2 ÷  sin x cos x   sin x  ≥ ( 1 + 4 ) = 25 2 Tõ ®ã suy ra §PCM 40> Híng dÈn: Dïng bÊt ®¼ng thøc C«si-Bunhac«pski cho c¸c sè x,a, b vµ x, x, 1 ta cã: ( x 2 + ax + b ( x 2 + cx + d ) ≤ ( x 2 + a 2 + b2 ) ( x 2 + x 2 + 1) (1) ) ≤ ( x 2 + c2 + d2 ) ( x 2 + x 2 + 1) (2) 2 ¸p dông cho x, c, d vµ x, x, 1 ta cã: 2 Céng vÕ víi vÕ (1) vµ (2) ta cã: (x 2 + ax + b ) +( x 2 41> Híng dÈn: a) ta cã 2 + cx + d ) ≤ ( x 2 + 1) ( x 2 + a 2 + b2 + c2 + d 2 ) 2 = ( 2x + ) 2 Sinh viªn: NguyÔn Xu©n L ¬ng Líp C§SP To¸n – Tin K48 83 §Ò tµi: Chøng minh bÊt ®¼ng thøc vµ c¸c øng dông 1 1 ≥ n + 1 2n 1 1 ≥ n + 1 2n ............... 1 1 ≥ n + 1 2n Céng vÕ theo vÕ ta thu ®îc ®iÒu cÇn chøng minh b) ta cã 1 1 22 2 1 1 1 1 < = − 2 2.3 2 3 3 1 1 1 1 < = − 42 3.4 3 4 ......................... < Céng vÕ víi vÕ víi vÕ ta ®îc ®iÒu cÇn chøng minh. 42> Híng dÈn: a) Dïng bÊt ®¼ng thøc C«si cho n sè d¬ng. b) Dïng bÊt ®¼ng thøc C«si cho n sè d¬ng a 2 ,a 2 ,a 2 ,...,a 2 vµ 1 2 3 n chÊt khai c¨n bËc 2 ®îc ®èi víi bÊt ®¼ng thøc cã hai vÕ d¬ng. 43> Híng dÈn: - C¸ch 1 1 1 1 1 1 1 Ta cã : = ≤ ( + )≤ 1 ( + 2x + y + z (x + y) + (x + z) y+z 16 x 4 x+y 1 1 + + 1) y z z T¬ng tù: 1 ≤ 1 −b ± x + 2y + z 16 1 1 1 ≤ ( + x + y + 2z 16 x b 2 − 4ac ( 1 + 1 + 1 + 1 ) y z z x 2a 1 1 1 + + ) y z z Sinh viªn: NguyÔn Xu©n L ¬ng Líp C§SP To¸n – Tin K48 84 §Ò tµi: Chøng minh bÊt ®¼ng thøc vµ c¸c øng dông Céng theo vÕ 3 B§T trªn: 1 1 1 1 1 1 1 1 + + ≤ .4( + + + ) 2x + y + z x + 2y + z x + y + 2z y z z 16 x 1 1 1 1 + + + =4 y z z x 1 1 1 + + ≤ 1 2x + y + z x + 2y + z x + y + 2z Mµ VËy DÊu “=” x¶y ra khi x = y = z = - C¸ch 2: Ta cã 4 3 1 1 1 1 1 1 1 1 1 1 = ≤ ( + )≤ + ( + )= + 2x + y + z 2x + (y + z) 4 2x y + z 8x 16 y z 8x 1 1 + 16y 16z T¬ng tù: 1 1 1 1 ≤ + + x + 2y + z 16x 8y 16z 1 1 1 1 ≤ + + x + y + 2z 16x 16y 8z Céng theo vÕ c¸c B§T: 1 1 1 1 1 1 1 + + ( + + )=1 ≤ x + 2y + z x + y + 2z 2x + y + z y z 4 x 1 1 1 VËy + + ≤1 x + 2y + z x + y + 2z 2x + y + z 44> Hãng dÈn: x − y ≤ 10 x ≤ 10 + y ⇒ x ≤ 100 + y + 20 y XÐt x - 2y ≤ - y + 20 45> Hãng dÈn: Theo gi¶ thiÕt: Ta ®a vÒ d¹ng: y +100 = ( ) 2 y − 10 + 200 ≤ 200 x + y < xy (x - 1).(y - 1) > 1 ⇒ x > 1; y > 1; Sinh viªn: NguyÔn Xu©n L ¬ng Líp C§SP To¸n – Tin K48 85 §Ò tµi: Chøng minh bÊt ®¼ng thøc vµ c¸c øng dông Theo biÓu thøc C«si ta cã: ( x − 1) + ( y − 1) ≥ 2 ( x − 1) .( y − 1) >2 x+y>4 46> Hãng dÈn: Ta cã x + y 2 ≥ x 2 + y3 (∗), gi¶ sö x + y 2 ≤ x 2 + y3 theo gi¶ thiÕt: x 3 + y 4 ≤ x 2 + y3 ⇒ ( x +x ) +( y 3 2 + y 4 ) < 2y 2 + 2y 3 ®iÒu ®ã ngîc víi 2 4 3 x + x 3 ≥ 2x 2 vµ y + y ≥ 2y Tõ (∗) ta cã: x + y ≥ x + y 2 2 3 ≥ x3 + y 4 ⇒ 2 x + 2 y 2 ≥ x 2 + y 3 + x3 + y 4 MÆt kh¸c: ( 1 + x 2 ) + ( 1 + y 4 ) ≥ 2 x + 2 y 2 ≥ x 2 + y 3 + x 3 + y 4 ⇒ ⇒ 2 + x 2 + y 4 ≥ x 2 + y 3 + x 3 + y 4 ⇒ §PCM. 47> Hãng dÈn: BiÕn ®æi biÓu thøc vÒ d¹ng: ) ( 2 2 2 15 2 2 2 2 2 2 2 2 2 2 a − b + b − c + c − a + a − bc + b − ca + c − ab ( ) ( ) ( ) ( ) ( ) ( )  ÷≥ 0 2 2  48> Hãng dÈn: Ta cã: 100a + 10b + c 99a + 9b a + b + c 99a + 9b = + ≤ +1 = a+b+c a +b+c a+b+c a+b 90a 9a + 9b 90a + +1 ≤ + 10 = 90 + 10 = 100 a+b a+b a VËy tØ sè lín nhÊt b»ng 100, dÊu =x¶y ra khi c = 0, b = 0 49> Hãng dÈn: NÕu a = b = c = 1 th× a + b + c = 3. Ta gi¶ sö a < 1,b > 1 Th× tõ ta cã abc = 1 ab − a − b + 1 = ( a − 1) ×( b − 1) < 0 a+d >c+b 50> Hãng dÈn: BiÕn ®æi vÕ tr¸i vÒ d¹ng: Sinh viªn: NguyÔn Xu©n L ¬ng Líp C§SP To¸n – Tin K48 86 §Ò tµi: Chøng minh bÊt ®¼ng thøc vµ c¸c øng dông 1  1 1 1 1  1 1   1  − ÷+  − ÷+  − ÷+ ×××+  − ÷>  2 4   6 8   10 12   98 100  1 37 1 1 1 1  1 1  1 1 >  − ÷+  − ÷+  − ÷ = + + =  2 4   6 8   10 12  4 24 60 120 51> Hãng dÈn: ThËt vËy: a > -b-c > a ×(-b-c) > (-b-c) 2 bc > b 2 + 2bc + c 2 ⇒ b 2 + bc + c 2 < 0 (*) 52> Hãng dÈn: BÊt ®¼ng thøc ®· cho cã thÓ viÕt: ( x + y − z ) ×( x − y + z ) ×( − x + y + z ) ≤ 0 x, y, z lµ 3 c¹nh cña tam gi¸c nªn mçi nh©n tö ®Òu lín h¬n 0 . VËy bÊt ®¼ng thøc kh«ng thÓ x¶y ra . 53> Hãng dÈn: BiÕn ®æi ( a + b ) ×( b + c ) ×( c + a ) − 5abc = = ab ( a + b − c ) + bc ( b + c − a ) + ca ( c + a − b ) > 0 V× a, b, c d¬ng, vµ a + b > c, b + c > a, c + a > b VËy (a + b)(b + c)(c + a) > 5abc 54> Hãng dÈn: 4( x12 + x 22 + x 32 + x 42 + x 52 ) - 4 x1 ( x 2 + x 3 + x 4 + x 5 ) = (x 2 1 − 4x1x 2 + 4x 22 ) + ( x12 − 4x1x 3 + 4x 32 ) + (×××) ( x1 − 2x 2 ) 2 55> Híng dÈn: Ta cã = + ( x1 − 2x 3 ) + ( x1 − 2x 4 ) + ( x1 − x 52 ) ≥ 0 2 2 a+b>c  ⇒ a 2 − ab + b 2 > 0  a3 + b3 + 3.abc = (a - b)(a2 – ab + b2) + 3.abc > c.(a2 – ab + b2) + 3.abc = c.(a2 + 2.ab + b2) = c.(a + b)2 > c.c2 = c3 56> Hãng dÈn: Hay ab + bc + ca < 2.c2 ab + bc + ca < a2 + b2 + c2 Sinh viªn: NguyÔn Xu©n L ¬ng Líp C§SP To¸n – Tin K48 87 §Ò tµi: Chøng minh bÊt ®¼ng thøc vµ c¸c øng dông XÐt: a2 + b2 + c2 – ab – bc – ca = ( ) 1 1 2 2a 2 + 2b 2 + 2c 2 − 2ab − 2bc − 2ca ) = ( a − b ) + (b − c) 2 + (c − a) 2 . ( 2 2 57> Híng dÈn: Ta viÕt : 0 ≤ a ≤ b ≤ c ≤ 1 (1 – a ).(1 – b) ≥ 0 ⇒ a + b ≤ 1 + ab ≤ 1 + 2.ab ⇒ a + b + c ≤ a + b + 1 ≤2 + 2.ab ⇒ 1 + ab ≤ 1 + ac ≤ 1 + bc a b c a +b+c vËy + + ≤ ≤2 bc + 1 ac + 1 ab + 1 1 + ab 58> Híng dÈn: 4( 1 − x ) ( 1 − z) ≤ ( 2 − x − z) = ( 1 + y) 2 ⇒ 4 ( 1 − x ) ( 1 − y ) ( 1 − z ) ≤ ( 1 + y ) ( 1 − y ) = ( 1 − y2 ) CÇn chøng minh: 1 + y ≥ 1-y 2 (®óng) 59> Híng dÈn: C¸ch 1> Ta cã b 2 + c 2 ≥ 2bc;c 2 + d 2 ≥ 2cd;d 2 + a 2 ≥ 2ad;... Céng vÕ víi vÕ ta ®îc 3 ( a 2 + b 2 + c 2 + d 2 ) ≥ 2 ( ab + ac + ad + bc + bd + cd ) ⇔ 4 ( a 2 + b 2 + c 2 + d 2 ) ≥ a 2 + b 2 + c 2 + d 2 + 2 ( ab + ac + ad + bc + bd + cd ) ⇔ 4 ( a 2 + b2 + c2 + d 2 ) ≥ ( a + b + c + d ) = 4 ⇔ ( a 2 + b2 + c2 + d 2 ) ≥ 1 C¸ch 2 2 4 ( a 2 + b 2 + c 2 + d 2 ) = ( 12 + 12 + 12 + 12 ) ( a 2 + b 2 + c 2 + d 2 ) ≥ ( a + b + c + d) = 8 2 60> Híng dÈn: 2 2 1 1 1 1   a + + b + ÷ 2 + + ÷ 2 2  1  1   a b a b = a + ÷ + b + ÷ ≥ a  b 2 2  Sinh viªn: NguyÔn Xu©n L ¬ng Líp C§SP To¸n – Tin K48 88 §Ò tµi: Chøng minh bÊt ®¼ng thøc vµ c¸c øng dông 2 4   2 + ÷ 25 a+b  ≥ = 2 2 61> Híng dÈn: 2 2 Ta cã 1 = ( a + b + c ) ≥ 4 ( a + b ) c ⇔ a + b ≥ 4 ( a + b ) c L¹i cã ( a + b ) ≥ 4ab ⇒ a + b ≥ 16abc 62> Híng dÈn: 2 2 b2 + c2 ⇔ 2 2 − a 2 ≥ ( 2 − a ) Ta cã 4 ⇔ 3a 2 − 4a ≤ 0 ⇔ 0 ≤ a ≤ 3 2 ( T¬ng tù 63> Híng dÈn: ) ( ) 4 4 0≤b≤ ; 0≤c≤ 3 3 ( 1 + a + b + c) 2 ≥ 4( a + b + c) L¹i cã a ≥ a 2 ;b ≥ b 2 ;c ≥ c 2 ( VËy ( 1 + a + b + c ) ≥ 4 a 2 + b 2 + c2 64> Híng dÈn: ¸p dông bÊt ®¼ng thøc C«si 2 T¬ng tù ) 2 a a b a b 3a + + ≥ 33 2 = 3 b b c bc abc b b c 3b + + ≥3 c c a abc c c a 3c + + ≥3 a a b abc Céng vÕ víi vÕ ta ®îc ®iÒu ph¶i chøng minh 65> Híng dÈn: ¸p dông bÊt ®¼ng thøc C«si Sinh viªn: NguyÔn Xu©n L ¬ng Líp C§SP To¸n – Tin K48 89 §Ò tµi: Chøng minh bÊt ®¼ng thøc vµ c¸c øng dông a 3 ( 1 + b ) ( 1 + c ) 3a a3 1+ b 1+ c + + ≥ 3. 3 = 8 64 ( 1 + b ) ( 1 + c ) 4 ( 1 + b) ( 1 + c) 8 b3 1 + a 1 + c 3b + + ≥ T¬ng tù 8 4 ( 1 + a ) ( 1 + c) 8 c3 1 + a 1 + b 3c + + ≥ 8 4 ( 1 + a ) ( 1 + b) 8 Céng vÕ víi vÕ ta ®îc: a3 b3 c3 3 a + b + c 3. 3 abc 3 + + + ≥ ≥ = 2 2 2 ( 1 + b) ( 1+ c) ( 1+ a ) ( 1 + c) ( 1 + a ) ( 1 + b) 4 66> Híng dÈn: 1 1 ⇔ ab ( a + b ) ≤ ⇔ ab   64 8 1 ⇔ ab 1 − 2 ab ≤ . 8 ab.( a + b ) ≤ 2 ( ( a+ b ) 2 ) 1 − 2 ab  ≤  8 ( ¸p dông bÊt ®¼ng thøc C«si cho hai sè kh«ng ©m 2 ab vµ 1-2 ab ( ) 2 ab. 1 − 2 ab ≤ ( ) ab 1 − 2 ab ≤ ) 2 ab + 1 − 2 ab 1 1 = ⇔ 2 ab 1 − 2 ab ≤ hay: 2 2 4 ( ) 1 8 67> Híng dÈn: Gi¶ sö ngîc l¹i c¶ bèn ®¼ng thøc ®Òu ®óng . Nh©n tõng vÒ ta cã : 2.3.8.32a(1 - b)b(1 - c)c(1 - d)d(1 - a) > 2 .3 1 => [ a (1 − a ) ] [ b(1 − b) ] [ c(1 − c) ] [ d (1 − d ) ] > 256 (1) MÆt kh¸c , ¸p dông bÊt ®¼ng thøc C«si ta cã : a (1 − a ) ≤ a +1− a 1 = 2 2 => a(1 - a) ≤ 1 4 Sinh viªn: NguyÔn Xu©n L ¬ng Líp C§SP To¸n – Tin K48 90 §Ò tµi: Chøng minh bÊt ®¼ng thøc vµ c¸c øng dông 1 4 1 c(1 - c) ≤ 4 1 d(1 - d) ≤ 4 T¬ng tù : b(1 - b) ≤ Nh©n tõng vÒ c¸c bÊt ®¼ng thøc ; ta cã : [ a(1 − a)] [ b(1 − b)] [ c(1 − c)] [ d (1 − d )] > 1 256 (2) Tõ (1) vµ (2) suy ra v« lý . §iÒu v« lý ®ã chøng tá Ýt nhÊt mét trong 4 bÊt ®¼ng thøc cho trong ®Çu bµi lµ sai . 69> Híng dÈn: Ta cã : p - a = b+c−a >0 2 T¬ng tù : p - b > 0 ; p - c > 0 ; ¸p dông kÕt qu¶ bµi tËp (3.5) , ta ®îc ; T¬ng tù : 1 1 4 4 + ≥ = p − a p − b ( p − a ) + ( p − b) c 1 1 4 + ≥ p −b p −c a 1 1 4 + ≥ p−a p−c b 1 1 1 1 1 1 + + ) ≥ 4( + + ) => 2( p−a p−c p−c a b c => ®iÒu ph¶i chøng minh . DÊu '' = '' x¶y ra khi : p - a = p - b = p - c ⇔ a = b = c . Khi ®ã tam gi¸c ABC lµ tam gi¸c ®Òu . iii. t×m gi¸ trÞ lín nhÊt, nhá nhÊ 70> Híng dÈn: ¸p dông bÊt ®¼ng thøc C«-si ta ®îc Sinh viªn: NguyÔn Xu©n L ¬ng Líp C§SP To¸n – Tin K48 91 §Ò tµi: Chøng minh bÊt ®¼ng thøc vµ c¸c øng dông min A = 2 khi x = 0 71> Híng dÈn: a) A = 1 − x + 1 + x Híng dÈn: XÐt A 2 = 2 = 2 1 − x 2 ta ®îc min A = 2 víi x = ±1 , max A = 2 víi x=0 b) B = x − 2 + 6 − x Híng dÈn: XÐt B2 = 4 + 2 (x − 2)(6 − x) ta cã x = 2 min B = 2 ⇔  x = 6 ¸p dông bÊt ®¼ng thøc C«-si a + b ≥ 2 ab ta ®îc max B = 2 2 ⇔ x − 2 = 6 ⇔ x = 4 72> Híng dÈn: ¸p dônh bÊt ®¼ng thøc Bunhac«pski ta ®îc max A = 5 ⇔ x = 2 73> Híng dÈn: ¸p dông bÊt ®¼ng thøc C«si vµ Bunhac«pski ta ®îc min A = −100 ⇔ x = −10,max A = 1000 ⇔ x = 10 74> Híng dÈn: C¸ch 1: a b ay bx A = x + y = 1(x + y) =  + ÷(x + y) = a + + +b x y x y   ¸p dông bÊt ®¼ng thøc C«-si cho hai sè d¬ng ta ®îc  x = a + ab 2 min A = a + b + 2 ab = ( a + b) ⇔   y = b + ab C¸ch 2. Dïng bÊt ®¼ng thøc Bunhac«pski 2 a b  a b A = (x + y).1 = (x + y)  + ÷ ≥  x + y ÷ x y x y  75> Híng dÈn: Ta cã x 4 + y 4 ≥ 2x 2 y 2 ; z 4 + y 4 ≥ 2z 2 y 2 ; x 4 + z 4 ≥ 2x 2 z 2 Sinh viªn: NguyÔn Xu©n L ¬ng Líp C§SP To¸n – Tin K48 92 §Ò tµi: Chøng minh bÊt ®¼ng thøc vµ c¸c øng dông Suy ra x 4 + y4 + z4 ≥ x 2 y2 + z 2 y2 + x 2z 2 MÆt kh¸c, ®Ó chøng minh r»ng nÕu a + b + c = 1 th× a 2 + b2 + c2 ≥ Do ®ã VËy 76> Híng dÈn: 1 3 1 x 2 y2 + z 2 y2 + x 2z 2 ≥ . 3 1 3 min A = ⇔ x = y = z = ± 3 3 A = x − y ≥ 0 , do ®ã A lín nhÊt khi vµ chØ khi A 2 lín nhÊt 5 ( Bunhac«pski) A2 ≤ 4 77> Híng dÈn: ¸p dông bÊt ®¼ng thøc C«si cho 2 sè x + y + z vµ t  2 5 1  2x x = −  5  y = −2  5 hoÆc VËy max A = ⇔ ⇔ 2  x 2 + 4y 2 = 1  y = 5   10  2 5  x = 5  y = − 5  10 78> Híng dÈn: P( x) = x +x+2 2 x ( x + 1) + 1 = ( ) 2 x ( x + 1) + 1 + 1 x ( x + 1) + 1 = x ( x + 1) + 1 + 1 x ( x + 1) + 1 ≥2 x = 0 dÊu d¼ng thøc khi x ( x + 1) + 1 = 1 ⇔   x = −1 79> Híng dÈn: Sinh viªn: NguyÔn Xu©n L ¬ng Líp C§SP To¸n – Tin K48 93 §Ò tµi: Chøng minh bÊt ®¼ng thøc vµ c¸c øng dông z4 1 1 P= ⇒ = 4 + ( x 4 + y4 ) 4 4 4 P z 1+ z ( x + y ) Ta cã x2 y xy z + x z + y = 3z ⇒ xy + + =3 z z2 ¸p dông C«si cho 4 sè kh«ng ©m 8 2 1 1; 2 ; x 2 ; x 2 cã 1 + 14 + x 4 + x 4 ≥ 4 4 x4 = 4 x z z z z ¸p dông C«si cho 4 sè kh«ng ©m 4 1 1 4 1 1 y y 4 cã 4 1; 4 ; 4 ; y 1+ 4 + 4 + y ≥ 4 8 = 4 2 z z z z z z ¸p dông C«si cho 4 sè kh«ng ©m 1; x 4 ; y 4 ; y 4 cã 1 + x 4 + y 4 + y 4 ≥ 4 4 x 4 y8 = 4xy 2 Céng vÕ víi vÕ cña 3 bÊt ®¼ng thøc trªn ta ®îc  x2 y  1 1 3 + 3. 4 + 3.x 4 + 3.y 4 ≥ 4.  + 2 + xy 2 ÷ = 12 ⇒ ≥ 3 dÊu b»ng khi x = y = z = 1. z P  z z  1 VËy Pmax = khi x = y = z = 1. 3 80> Híng dÈn: Tõ (x P= 2 2 3 2 + y3 ) − ( x 2 + y 2 ) ( x − 1) ( y − 1) 2 2 x 2 ( x − 1) + y 2 ( y − 1) x2 y2 = = + ≥ y −1 x −1 ( x − 1) ( y − 1) 2xy ( x − 1) ( y − 1) x2 y2 x −1 +1 x dÊu b»ng khi = l¹i cã ( x-1) .1 ≤ = dÊu b»ng khi x = 2 y −1 x −1 2 2 y −1 +1 y = dÊu b»ng khi y = 2 ( y − 1) .1 ≤ 2 2 2xy ⇒P≥ = 8 dÊu b»ng khi x = y = 2 x y . 2 2 VËy Pmin = 8 khi x = y = 2. 81> Híng dÈn: Ta cã f ( x ) = 2x 2 + 5x + 2 + 2 x + 3 − 2x = ( x + 2 ) ( 2x + 1) + Sinh viªn: NguyÔn Xu©n L ¬ng Líp C§SP To¸n – Tin K48 4 ( x + 3 ) − 2x. 94 §Ò tµi: Chøng minh bÊt ®¼ng thøc vµ c¸c øng dông ¸p dông C«si cho 2 sè kh«ng ©m Ta cã ( x + 2 ) vµ ( 2x + 1) x + 2 + 2x + 1 3x + 3 = 2 2 dÊu d¼ng thøc khi: x + 2 = 2x + 1 ⇔ x = 1 ¸p dông C«si cho 2 sè kh«ng ©m 4 vµ ( x + 3) ( x + 2 ) ( 2x + 1) Ta cã ⇒ ≤ 4+x +3 x +7 = 2 2 DÊu d¼ng thøc khi : 4 = x + 3 ⇔ x = 1 4 ( x + 3) ≤ ( x + 2 ) ( 2x + 1) + 4 ( x + 3) − 2x ≤ 3x + 3 x + 7 + − 2x = 5 2 2 dÊu d¼ng thøc khi x = 1 VËy: f ( x ) min = 5 khi x = 1. Sinh viªn: NguyÔn Xu©n L ¬ng Líp C§SP To¸n – Tin K48 95 §Ò tµi: Chøng minh bÊt ®¼ng thøc vµ c¸c øng dông 82> Híng dÈn: Ta cã T = ( x + y ) ( x + z ) = x 2 + xz + xy + yz = x ( x + y + z ) + yz 1 Tõ: ( x + y + z ) xyz = 1 ⇒ x ( x + y + z ) = thay vµo T ta cã: yz T= 83> Lêi gi¶i 1 + yz ≥ 2 dÊu d¼ng thøc khi yz = 1 yz 1  x10 y10  Ta cã + 2 ÷ ≥ x 4 y 4 DÊu “=” x¶y ra khi vµ chØ khi x12 = y12  2 2 y x  1 16 1 x + y16 ) ≥ x 8 y8 dÊu b»ng khi x16 = y16 ( 4 2 2 2 1 1 1 ⇒ Q ≥ x 8 y8 + x 4 y 4 − ( 1 + x 2 y 2 ) = ( x 8 y8 + 2x 4 y 4 + 1) − ( 1 + x 2 y 2 ) − 2 2 2 2 2 1 1 = ( x 4 y 4 + 1) − ( x 2 y 2 + 1) − 2 2 2 l¹i cã: ( 12 + 12 ) ( x 2 y 2 ) + 12  ≥ ( x 2 y 2 + 1)   hay 2 ( x 4 y 4 + 1) ≥ ( x 2 y 2 + 1) dÊu b»ng khi x 2 y 2 = 1 2 84> Híng dÈn: Ta cã 1 2 1 1 2 a 2  1 1  3a 2  1 1  2 2 1 2 P = 2 ( b + c ) + a  2 + 2 ÷ = 2 ( b + c ) +  2 + 2 ÷+  + ÷ a 4  b c  4  b2 c2  b c  a Cã 1 2 a2  1 1  1 2 a2  1 1  2 2 ( b + c ) + 4  b2 + c 2 ÷≥ 2 a 2 ( b + c ) . 4  b2 + c2 ÷ a2     1 2 a2 4 1 2 a2  1 1  2 2 ≥ 2 2 (b +c ). . 2 = 2 dÊu b»ng khi 2 ( b + c ) =  2 + 2 ÷ a 4 b + c2 a 4 b c  ⇔ 4b 2 c 2 = a 4 ; b 2 = c 2 Sinh viªn: NguyÔn Xu©n L ¬ng Líp C§SP To¸n – Tin K48 96 §Ò tµi: Chøng minh bÊt ®¼ng thøc vµ c¸c øng dông a2  1 1  a2 4 L¹i cã 3.  2 + 2 ÷ ≥ 3. . 2 = 3 dÊu “=” x¶y ra khi vµ chØ 4 b c  4 b + c2 khi b 2 = c 2 a2 a2 2 2 2 2 ⇒ P ≥ 5 dÊu b»ng khi b = c = . VËy Pmin = 5 khi b = c = . 2 2 85> Híng dÈn: x +1 = 3 x −x 4 ( x 2 − 1) + 2x 2 86> Híng dÈn: Áp dụng bất đẳng thức : 2 x ( x 2 − 1) x2 −1 2x = + 2 ≥2 2 x x −1 a2 + b2 + c2 + d 2 ≥ (a + c)2 + (b + d)2 A = x 2 + 12 + (1 − x) 2 + 2 2 ≥ ( x + 1 − x ) 2 + (1 + 2) 2 = 10 1− x 1 min A = 10 ⇔ =2 ⇔ x= x 3 87> a) Điều kiện : x ≥ 1, y ≥ 2 . Bất đẳng thức Cauchy cho phÐp làm giảm một tổng : a+b ≥ ab 2 Ở đ©y ta muốn làm tăng một tổng. Ta dùng bất đẳng thức : a + b ≤ 2(a 2 + b 2 ) A = x − 1 + y − 2 ≤ 2(x − 1 + y − 3) = 2 x − 1 = y − 2  x = 1,5 max A = 2 ⇔  ⇔  x + y = 4  y = 2,5 C¸ch kh¸c : XÐt A2 rồi dïng bất đẳng thức Cauchy. b) Điều kiện : x ≥ 1, y ≥ 2 Bất đẳng thức Cauchy cho phÐp làm trội một tÝch : Ta xem c¸c biểu thức ab ≤ a+b 2 x − 1 , y − 2 là c¸c tÝch : Sinh viªn: NguyÔn Xu©n L ¬ng Líp C§SP To¸n – Tin K48 97 §Ò tµi: Chøng minh bÊt ®¼ng thøc vµ c¸c øng dông x − 1 = 1.(x − 1) , y − 2 = 2(y − 2) 2 x − 1 1.(x − 1) 1 + x − 1 1 Theo bất đẳng thức Cauchy : = ≤ = x x 2x 2 y−2 2.(y − 2) 2 + y − 2 1 2 = ≤ = = y 4 y 2 2y 2 2 2 x − 1 = 1 x = 2 1 2 2+ 2 M ax B = + = ⇔  ⇔  2 4 4 y − 2 = 2 y = 4 88> Híng dÈn: Điều kiện x ≤ 2 Đặt 2 2 − x = y ≥ 0, ta cã : y = x − 2 2 1 9 9 9 1 7  2 a = 2 − y + y = −  y − ÷ + ≤ ⇒ max A = ⇔ y = ⇔ x = 2 4 4 4 2 4  89> Híng dÈn Áp dụng | A | + | B | ≥ | A + B | . min A = 2 ⇔ 1 ≤ x ≤ 2 . ¸p dông c¸c bÊt ®¼ng thøc ®¶ biÕt ¸p dông xÐt hiÖu 90> Híng dÈn − x 2 + 4 x + 12 ≥ 0 ( x + 2)(6 − x) ≥ 0 ⇔  ⇔ − 1 ≤ x ≤ 3{ Tập x¸c định :  2 − x + 2 x + 3 ≥ 0 ( x + 1)(3 − x) ≥ 0 (1) XÐt hiệu : (- x2 + 4x + 12)(- x2 + 2x + 3) = 2x + 9. Do (1) nªn 2x + 9 > 0 nªn A > 0. XÐt : A2 = ( ) } 2 ( x + 2)(6 − x) − ( x + 1)(3 − x) . Hiển nhiªn A2 ≥ 0 nhưng dấu “ = ” kh«ng xảy ra (víi A > 0). Ta biến đổi A2 dưới dạng kh¸c : A2 = (x + 2)(6 – x) + (x + 1)(3 – x) - 2 ( x + 2)(6 − x)( x + 1)(3 − x) = = (x + 1)(6 – x) + (6 – x) + (x + 2)(3 – x) – (3 – x) - 2 ( x + 2)(6 − x)( x + 1)(3 − x) = (x + 1)(6 – x) + (x + 2)(3 – x) – 2 ( x + 2)(6 − x)( x + 1)(3 − x) + 3 = ( ( x + 1)(6 − x) − ( x + 2)(3 − x) ) 2 + 3. Sinh viªn: NguyÔn Xu©n L ¬ng Líp C§SP To¸n – Tin K48 98 §Ò tµi: Chøng minh bÊt ®¼ng thøc vµ c¸c øng dông 91> Lêi gi¶i: A2 ≥ 3. Do A > 0 nªn min A = Trước hết ta chứng minh : a + b ≤ Áp dụng (*) ta cã : S = 3 với x = 0. 2(a 2 + b 2 ) (*) (a + b ≥ 0) x − 1 + y − 2 ≤ 2(x − 1 + y − 2) = 2 3  x=  x − 1 = y − 2  2 maxS = 2 ⇔  ⇔ x + y = 4 y = 5  2 2 • Cã thể tÝnh S rồi ¸p dụng bất đẳng thức Cauchy. 92> Lêi gi¶i: A = 3x + 3y ≥ 2. 3x.3y = 2 3x + y = 2. 34 = 18 . min A = 18 với x = y = 2. 93. Lêi gi¶i: Kh«ng mất tÝnh tổng qu¸t, giả sử a + b ≥ c + d. Từ giả thiết suy ra : a+b+c+d . 2 b c b+c  c c  a +b+c+d c+d c+d  A= + = − − − − ÷≥ ÷ c+d a +b c+d c+d a+b 2(c + d) c + d a+b  Đặt a + b = x ; c + d = y với x ≥ y > 0, ta cã : b+c≥ A≥ x+y y y x 1 y  x y 1 x y 1 1 − + = + −1+ =  + ÷− ≥ 2. . − = 2− 2y y x 2y 2 x  2y x  2 2y x 2 2 1 ⇔ d =0 , x = y 2 , b+c≥a +d ; 2 a = 2 + 1,b = 2 − 1,c = 2,d = 0 min A = 2 − Chẳng hạn khi II. gi¶i ph¬ng tr×nh 94> Híng dÈn : a. Tãm t¾t : ( 2 x − 3 + ⇔ 5 − 2x )2 ≤ 2(2x - 3 + 5 - 2x) = 4 2 x − 3 + 5 − 2x ≤ 2 => MaxL = 2 khi x = 2 . Sinh viªn: NguyÔn Xu©n L ¬ng Líp C§SP To¸n – Tin K48 99 §Ò tµi: Chøng minh bÊt ®¼ng thøc vµ c¸c øng dông 3 5 ≤x≤ 2 2 (*) ⇔ 2 x − 3 + 5 − 2x = x2 - 4x + 6 VP = (x - 2)2 + 2 ≥ 2 , dÊu '' = '' x¶y ra khi x = 2 . => víi x = 2 ( tho¶ m·n TX§ ) th× VT = VP = 2 . b. TX§ : => ph¬ng tr×nh (*) cã nghiÖm x = 2 . 95> Híng dÈn: TX§ : -2 ≤ x ≤ 6. VP = (x - 3)2 + 4 ≥ 4 . DÊu '' = '' x¶y ra khi x = 3 . VT2 = ( 6 − x .1 + x + 2 .1)2 ≤ (6 - x + x + 2)(1 + 1) = 16 => VT ≤ 4 , dÊu '' = '' x¶y ra khi 6− x = x+2  x=2. => kh«ng cã gi¸ trÞ nµo cña x ®Ó VT = VP => Ph¬ng tr×nh v« nghiÖm 96> Híng dÈn: 2 3 x 2 − 12 x + 16 ≥ 2 ; y − 4 y + 13 ≥ 3 => VT ≥ 5 . x − 2 = 0 x = 2  y − 2 = 0 y = 2 DÊu '' = '' x¶y ra khi :  => ph¬ng tr×nh cã nghiÖm : x = 2 ; y = 2 c. kÕt luËn C¸c bµi tËp vÒ bÊt ®¼ng thøc thêng lµ t¬ng ®èi khã ®èi víi häc sinh , nhng khi híng dÉn häc sinh xong ®Ò tµi (mét sè ph¬ng ph¸p chøng minh bÊt ®¼ng thøc vµ øng dông cña bÊt ®¼ng thøc ), häc sinh sÏ thÊy r»ng viÖc lµm bµi to¸n vÒ bÊt ®¼ng thøc sÏ rÔ h¬n . §ång thêi ®øng tríc bµi to¸n khã cho dï ë d¹ng bµi tËp nµo häc sinh còng cã híng suy nghÜ vµ tËp suy luËn , c¸c em sÏ cã tù tin h¬n . Chuyªn ®Ò cßn cã thÓ cßn nhiÒu thiÕu sãt , rÊt mong ®îc sù ñng hé cña c¸c ThÇy, C« gi¸o Vµ c¸c b¹n ®Ó ®Ò tµi ngµy cµng hoµn thiÖn h¬n. Nh©n ®©y, t«i xin göi lêi c¶m ¬n ch©n thµnh tíi Ban gi¸m hiÖu nhµ trêng, ban Sinh viªn: NguyÔn Xu©n L ¬ng Líp C§SP To¸n – Tin K48 100 §Ò tµi: Chøng minh bÊt ®¼ng thøc vµ c¸c øng dông chñ nhiÖm Khoa To¸n – Tin, ®Æc biÖt lµ gi¶ng viªn.Th.S.NCS.NguyÔn Quang HoÌ ®· t¹o ®iÒu kiÖn vµ trùc tiÕp híng dÉn , gióp ®ì t«i hoµn thµnh ®Ò tµi nµy. §ång Híi, ngµy 12 th¸ng 12 n¨m 2008. Sinh viªn: NguyÔn Xu©n L¬ng Môc lôc Môc lôc........................................................................................................101 ************************************************************ tµI LIÖU THAM KH¶O 1> 2> 3> 4> 5> 6> 7> 8> §¹i sè s¬ cÊp vµ thùc hµnh gi¶i to¸n Hoµng Kú( chñ biªn) Bµi tËp c¬ b¶n vµ n©ng cao ®¹i sè 8 (Phan V¨n §øc-NgyÔn Th¸I Hoµ - NguyÔn ThÕ Thù¬ng NguyÔn Anh Dòng) Bµi tËp to¸n chän läc vÒ B§T (GS: Phan Huy Kh¶i) N©ng cao vµ ph¸t triÓn to¸n 8 (Vò H÷u B×nh) To¸n n©ng cao ®¹i sè 8 (NguuyÔn VÜnh CËn) BÊt ®¼ng thøc (TrÇn §øc Huyªn) To¸n n©ng cao vµ c¸c chuyªn ®Ò ®¹i sè 8 (Vò D¬ng Thôy: Chñ biªn) NguyÔn Ngäc §¹m N©ng cao vµ ph¸t triÓn to¸n 8 Vò H÷u B×nh). Sinh viªn: NguyÔn Xu©n L ¬ng Líp C§SP To¸n – Tin K48 101 [...]... toán học , ta tiến hành : + Kiểm tra bất đẳng thức đúng với n = 1 (n = n0) + Giả sử bất đẳng thức đúng với n = k > 1 (k > n0) + Chứng minh bất đẳng thức đúng với n = k + 1 + Kết luận bất đẳng thức đúng với n > 1 (n > n0) Chú ý: Khi chứng minh bất đẳng thức có n số (n N) Thì ta nên chú ý sử dụng phơng pháp quy nạp toán học - Ví dụ : Bài toán 3.1 Chứng minh bất đẳng thức Côsi trong trờng hợp tổng quát... x ) 2 x R Bài toán 6.5 Cho a,b,c là các số dơng chứng minh bất đẳng thức: Lời giải: a 2 + b 2 + c2 a + b + c 2 b+c c+a a +b áp dụng bất đẳng thức Bunhacôpski 7 Phơng pháp phản chứng - Kiến thức : Giả sử phải chứng minh bất đẳng thức nào đó đúng , ta hãy giả sử bất đẳng thức đó sai , sau đó vận dụng các kiến thức đã biết và giả thiết của đề bài để suy ra điều vô lý Điều vô lý có thể là trái với giả... tức là khi và chỉ khi a1 = a 2 = = a n Trong lý thuyết đả có một số bất đẳng thức đợc chúng minh bằng phơng pháp quy nạp (bất đẳng thức Côsi, Becnuli, ) Sau đây ta xét một số bài toán khác Bài toán 3.2 Sinh viên: Nguyễn Xuân L ơng Lớp CĐSP Toán Tin K48 12 Đề tài: Chứng minh bất đẳng thức và các ứng dụng 2 2 2 Tổng quát của bất đẳng thức u + v u + v ữ 2 2 Cho a, b là hai số dơng, chứng minh rằng... L ơng Lớp CĐSP Toán Tin K48 21 Đề tài: Chứng minh bất đẳng thức và các ứng dụng 1 1 2 2 a+b a+b 2 + + = ( a + b) + = + + a b a+b a+b 2 2 a+b a+b a+b 2 = + + ữ ab + 2 = 3 2 a+b 2 Bài toán 5.3 1+1 4 x y x+y Cho x,y >0, chứng minh (1) Phân tích : Do x,y >0 nên bất đẳng thức (1)có thể suy từ bất đẳng thức Côsi hoặc trực tiếp xét hiệu Lời giải : Cách 1: Sử dụng bất đẳng thức Côsicho hai số dơng: x + y... : Với n =2 bất đẳng thức đả đợc chứng minh ở 1 (bất đẳng thức Ơclit) Để chứng minh bất đẳng thức tổng quát, trứơc hết ta hãy xét vài bất đẳng thức phụ Nếu x1, x 2 R + thì x1 < x 2 x1n 1 < x 2n 1 Vậy x1, x 2 R + thì ta luôn có (chuyển một bộ phận sang vế phải, ta đợc) (x1n 1 x 2n 1)(x1 x 2 ) 0 x1n + x 2n x1x 2n 1 + x 2 x1n 1 Lấy n số thực không âm x1,x 2 x n R + , viết các bất đẳng Sinh viên:... minh rằng không có 3 số dơng a, b, c nào thoả mãn cả 3 bất đẳng thức: a+ 1 < 2; b 1 1 b + < 2; c + c a Lời giải: Giả sử tồn tại ba số dơng a, b, c thoả mãn cả 3 bất đẳng thức: Sinh viên: Nguyễn Xuân L ơng Lớp CĐSP Toán Tin K48 28 Đề tài: Chứng minh bất đẳng thức và các ứng dụng a+ 1 < 2; b 1 1 b + < 2; c + < 2 c a Cộng theo từng vế ba bất đẳng thức trên, ta đợc: 1 1 1 +b+ +c+ 0 F(x) 0 0 * a x b (x a)(x b) 0 * F(x) = ax 2 + bx + c 4ac-b 2 4a x R (a > 0) b) Phơng pháp *> Phơng pháp 1: Để chứng minh bất đẳng thức M > N ta biến đổi Sinh viên: Nguyễn Xuân L ơng Lớp CĐSP Toán Tin K48 15 Đề tài: Chứng minh bất đẳng thức và các ứng... 22 Đề tài: Chứng minh bất đẳng thức và các ứng dụng Lời giải: Cách 1: Nhận xét: Do a,b,c là độ dài 3 cạnh của một tam giác nên ta có a + b c > 0; a + c b > 0; b + c a > 0 áp dụng bất đẳng thức Côsi cho các cặp số dơng: (a + b c)(a + c b) a + b c + a + c b = a 2 (a +c-b)(b +c-a) c (b +c-a)(b +a -c) b (1) (2) (3) Để ý rằng cả 2 vế của các bất đẳng thức (1) (2) (3) là các số dơng và ba bất đẳng. .. từ đó suy ra đẳng thức cần chứng minh là đúng - Một số hình thức chứng minh bất đẳng thức : + Dùng mệnh đề đảo + Phủ định rồi suy ra điều trái với giả thiết + Phủ định rồi suy ra trái với đIều đúng + Phủ định rồi suy ra hai điều trái ngợc nhau + Phủ định rồi suy ra kết luận - Các ví dụ : Bài toán 7.1 Sinh viên: Nguyễn Xuân L ơng Lớp CĐSP Toán Tin K48 27 Đề tài: Chứng minh bất đẳng thức và các... ( x 2 ) 0 Dấu đẳng thức xảy ra khi a = b 5 Phơng pháp sử dụng bất đẳng thức Cauchy #>Với hai số a, b 0 ta luôn có: a + b ab 2 Dấu = xảy ra khi và chỉ khi a = b Chứng minh: Cách1: (Phơng pháp biến đổi tơng đơng) 2 a+b a+b a.b ab (a b) 2 0 Bđt hiển nhiên ữ 2 2 đúng Sinh viên: Nguyễn Xuân L ơng Lớp CĐSP Toán Tin K48 19 Đề tài: Chứng minh bất đẳng thức và các ứng dụng Đẳng thức xảy ra a = ... 12.1 Cho x, y,z l ba s thay i, nhn giá tr thuc on [0 ; 2] 2(x + y + z) (xy + yz + zx) Chng minh rng: Lời giải: Do gi thit x, y,z l ba s thay i, nhn giá tr thuc on [0 ; 2] (2 x)(2 y)2 z)

Ngày đăng: 21/10/2015, 22:54

Từ khóa liên quan

Tài liệu cùng người dùng

  • Đang cập nhật ...

Tài liệu liên quan